تغطية شاملة

تم اكتشاف أبعد ثقب أسود هائل حتى الآن

اكتشاف رائع من الكون الشاب: اكتشف الباحثون أبعد ثقب أسود هائل تم رصده حتى الآن، بعد 690 مليون سنة فقط من الانفجار الكبير. وتفاجأ الباحثون بكتلته الهائلة، التي تبلغ 800 مليون مرة كتلة الشمس، وما زالوا لا يفهمون كيف تمكن من جمع كل هذه الكتلة في مثل هذه الفترة المبكرة.

تصور للكوازار J1342+0928، الهالة المجرية النشطة التي تحتوي في مركزها على ثقب أسود هائل كتلته 800 مليون مرة كتلة شمسنا، والذي يلتهم الكثير من المواد من القرص الساخن المحيط به. المصدر: روبن دينيل/معهد كارنيجي للعلوم.
تصور للكوازار J1342 + 0928، نواة المجرة النشطة التي تحتوي على ثقب أسود هائل في مركزها، تبلغ كتلته 800 مليون مرة كتلة شمسنا، والذي يلتهم الكثير من المادة من القرص الساخن المحيط به. مصدر: روبن دينيل / معهد كارنيجي للعلوم.

أعلن فريق دولي من الباحثين، أمس، اكتشاف أبعد ثقب أسود فائق الكتلة معروف حتى الآن، والذي استغرق ضوءه أكثر من 13 مليار سنة ضوئية ليصل إلينا، وذلك منذ أن كان عمر الكون 690 مليون سنة فقط، أي 5% من عمر الكون. عمره الحالي (13.8 مليار سنة). نتائج البحث نشرت أمس في مجلة الطبيعة.

الثقب الأسود، المسمى J1342 + 0928 (اسم رمزي يشير إلى موقعه في السماء)، لديه كتلة هائلة تبلغ 800 مليون مرة كتلة شمسنا. يمتص الثقب الأسود الهائل الكثير من المواد من قرص ساخن يحوم حوله في مركز المجرة - وهي الظاهرة المعروفة باسم كوازار.

"الكوازارات هي من بين ألمع وأبعد الأجسام السماوية، وهي ضرورية لفهم الكون الشاب." قال الباحث برام فانمانز من مركز ماكس بلانك لعلم الفلك في ألمانيا. إن الكوازار المحدد الذي يوجد فيه الثقب الأسود الهائل له لمعان يساوي 40 تريليون مرة سطوع شمسنا.

تم تحديد المسافة الهائلة بين الكوازار والثقب الأسود في مركزه عن طريق القياس الانزياح الأحمر من نوره في ما يسمى بالظاهرة تأثير دوبلر، الطول الموجي للضوء من مصدر يتحرك بعيدًا عن المشاهد، "يذوب" ويتغير إلى طول موجي أطول. وبسبب تسارع تمدد الكون، كلما ابتعد الجسم عنا، كلما ابتعد عنا أيضًا، وزاد انزياح ضوءه نحو الأحمر. يبلغ الانزياح الأحمر للكوازار البعيد حوالي 7.5، مما يعني أن الطول الموجي كما يُرى من الأرض هو 8.5 مرات (7.5+1) عما كان عليه عندما تم قذفه منه منذ أكثر من 13 مليار سنة.

سر كتلة الثقب الأسود البدائي

فاجأت الكتلة الهائلة للثقب الأسود البعيد والقديم الباحثين. إن النظريات الموجودة التي تشرح كيفية تشكل وتطور الثقوب السوداء الهائلة، وامتصاص المادة من الغاز والنجوم في مركز المجرات، تفشل في تفسير كيفية نمو الثقب الأسود بهذه السرعة، وفي وقت قصير جدًا، من الناحية الكونية، بعد الانفجار الكبير.

"إن جمع كل هذه الكتلة في أقل من 690 مليون سنة يمثل تحديًا كبيرًا لنظريات نمو الثقب الأسود الهائل". قال إدواردو بينيدوس، من مركز كارنيجي بالولايات المتحدة الأمريكية، والذي قاد البحث.

رسم توضيحي يحاول توضيح المسافة الشاسعة للثقب الأسود الهائل الذي استغرق ضوءه أكثر من 13 مليار سنة ضوئية ليصل إلى ضوء تلسكوبين ماجلان في تشيلي، ويظهر أحدهما في أسفل الرسم التوضيحي. المصدر: روبن دينيل، بإذن من معهد كارنيجي للعلوم.
رسم توضيحي يوضح المسافة الهائلة التي قطعها الثقب الأسود الهائل الذي استغرق ضوءه أكثر من 13 مليار سنة ضوئية ليصل إلى تلسكوبات ماجلان في تشيلي، ويظهر أحدها في أسفل الرسم التوضيحي. مصدر: روبن دينيل ، بإذن من معهد كارنيجي للعلوم.

"إذا بدأت ببذرة مثل نجم كبير، وتركتها تنمو بأقصى معدل ممكن، وبدأت من لحظة الانفجار الأعظم، فلن تتمكن من خلق شيء كتلته 800 مليون كتلة شمسية - فهذا غير واقعي" , شرح روبرت سيمكو، أستاذ الفيزياء من معهد ماساتشوستس للتكنولوجيا، والذي شارك في الدراسة. "لذلك يجب أن تكون هناك طريقة أخرى تم إنشاؤها بها. وكيف حدث ذلك بالضبط، لا أحد يعرف".

ووفقا للباحثين، من أجل تفسير تكوين مثل هذا الثقب الأسود الهائل في مثل هذه الفترة المبكرة من عمر الكون، يجب أن تظهر نماذج تكوين الثقوب السوداء فائقة الكتلة أن "بذور" الثقوب السوداء التي تبلغ كتلتها ما يصل إلى من الممكن أن تكون كتلته حوالي 10,000 كتلة شمسية موجودة في الكون الشاب، في فترة 65 مليون سنة فقط بعد الانفجار الكبير، أو، بدلا من ذلك، لشرح كيف نمت الثقوب السوداء الأولى بمعدل أكبر مما هو مقبول حاليا.

أدلة من الكون الشاب

وينبع تفرد الثقب الأسود أيضًا من الفترة التي يتكهن فيها الباحثون بوجوده. استغرق الضوء الصادر من الكوازار أكثر من 13 مليار سنة للوصول إلى الأرض، وانبعث منها عندما كان عمر الكون 690 مليون سنة فقط، وذلك خلال الفترة المعروفة باسم "مرحلة إعادة التأين" (Reionization Phase).إعادة التأين)، أو "إعادة التأين" باللغة العبرية.

حتى حوالي 380,000 ألف سنة بعد الانفجار الكبير، كان الكون مظلمًا تمامًا، لأن الفوتونات كانت محاصرة داخل البلازما الكثيفة والساخنة للجسيمات والإلكترونات. لكن حتى بعد أن برد الكون بدرجة كافية تسمح بظهور ذرات الهيدروجين ذات الشحنة الكهربائية المحايدة، وأصبح شفافا للضوء، فقد ظل مظلما حتى أشرقت النجوم الأولى. ثم بدأت الفترة المعروفة باسم "مرحلة إعادة التأين". حدث ذلك منذ ما بين 13.5 و12.5 مليار سنة مضت، عندما تسببت الطاقة المنبعثة من النجوم والمجرات الأولى في تأين معظم ذرات الهيدروجين في الكون الفتي - لتفقد إلكترونها.

وقال بينايدوس: "كانت مرحلة إعادة التأين آخر تغيير مهم في الكون، وتعتبر واحدة من حدود أبحاث الفيزياء الفلكية".

ومن خلال تحليل طيف الضوء المنبعث من البيئة القريبة من الكوازار، تمكن الباحثون من تحديد أن ما بين 38 و77% من الهيدروجين الموجود في بيئته موجود على شكل ذرات محايدة، وليس أيونات. وهذا يعني أن بيئة الكوازار لم يكن لديها الوقت الكافي لتصبح متأينة إلى حد كبير، وأنها كانت موجودة خلال فترة إعادة التأين - الدليل الوحدة الذي تم اكتشافه من هذه الفترة القديمة للكون.

في مخطط مقارنة بين الكوازارات الأكثر ضخامة والأبعد المعروفة حتى الآن. في حين أن J1342+0928 (النجم الأصفر) ليس النجم الزائف الأكثر ضخامة، إلا أنه أبعد نجم تم اكتشافه حتى الآن. المصدر: جينيي يانغ/وا؛ ريدار هان / فيرميلاب؛ م. نيوهاوس/NOAO/AURA/NSF.
يقارن الرسم البياني بين الثقوب السوداء الهائلة وأكبر كتلة ومسافة معروفة حتى الآن. في حين أن J1342+0928 (النجم الأصفر) ليس الأكثر ضخامة، إلا أنه أبعد نجم تم اكتشافه حتى الآن. مصدر: جينيي يانغ / UA؛ ريدار هان / فيرميلاب؛ م. نيوهاوس/NOAO/AURA/NSF.

يوجد الآن عدد من النظريات المتنافسة حول متى حدثت مرحلة إعادة التأين في الكون بالضبط. ووفقا للباحثين، تشير الأدلة الجديدة إلى أن إعادة التأين حدثت في وقت متأخر نسبيا في التسلسل الزمني الكوني.

"هذا يضيف إلى فهمنا الشامل للكون، لأننا حددنا النقطة الزمنية التي يكون فيها الكون في منتصف الطريق خلال التحول السريع للغاية من الحالة المحايدة إلى الحالة المؤينة." قال سيمكو "لدينا الآن القياس الأكثر دقة للتاريخ الذي أضاءت فيه النجوم الأولى."

البحث عن الكوازار الأبعد

من الصعب اكتشاف الكوازارات البعيدة. وفي السنوات الأخيرة، وخاصة منذ عام 2010، تمكن علماء الفلك من مضاعفة عدد الكوازارات المعروفة التي يعود تاريخها إلى حوالي 850 إلى مليار سنة بعد الانفجار الكبير.

من أجل "تحطيم الرقم القياسي" وتحديد موقع كوازار بعيد من فترة سابقة، استخدم فريق الباحثين بقيادة إدواردو بينيدو طريقة معقدة ومنهجية لرسم خريطة لملايين المرشحين.

لتحديد المرشحين للكوازارات، استخدم الباحثون بيانات معقدة من عدد من المسوحات واسعة النطاق للسماء، بهدف تحديد مواقع الأجسام التي تكون ساطعة في نطاق الأشعة تحت الحمراء من ناحية، وغير قابلة للاكتشاف في الضوء المرئي من ناحية أخرى. المدى - وذلك لأنه بسبب تأثير دوبلر، فإن الانزياح الأحمر للضوء من الكوازارات البعيدة يجعلها أكثر سطوعًا في الأشعة تحت الحمراء.

شملت مسوحات السماء التي استخدمها الباحثون تلك التي قاموا بإنتاجها تلسكوب فضائي وايز"تلسكوب الأشعة تحت الحمراء في المملكة المتحدة" (يوكيرت) في مونا كا في هاواي، و"كاميرا الطاقة المظلمة" (ديكام) في شيلي.

ومن بين مئات الملايين من الأهداف التي تمت ملاحظتها في هذه المسوحات السماوية، تمكن الباحثون من الوصول إلى عدة مئات من المرشحين للكوازارات. تم منح هؤلاء المرشحين ملاحظات أكثر تعمقًا باستخدام العديد من التلسكوبات، خاصة اثنين تلسكوب ماجلان في شيلي.

أدى هذا الجهد السيزيفي في النهاية إلى اكتشاف كوازار واحد من هذه الفترة المبكرة من عمر الكون، نظرًا لأن الباحثين المشاهدين إن وجود ما بين 20 إلى 100 من النجوم الزائفة البعيدة والمشرقة مثل J1342 + 0928 في السماء بأكملها، يعد اكتشافًا مهمًا للغاية.

تُظهر الصورتان الصغيرتان أعلاه الفرق في سطوع الكوازار في الضوء المرئي والأشعة تحت الحمراء. بينما في الأشعة تحت الحمراء (يمين) يكون الكوازار مرئيًا بوضوح، في الضوء المرئي لا يمكن رؤيته على الإطلاق، لأن تأثير دوبلر تسبب في تشويه ضوءه المرئي إلى أطوال موجية أطول - الأشعة تحت الحمراء. يوجد في الجزء السفلي من الصورة طيف الكوازار، كما سجله مقياس الطيف في اثنين من تلسكوبات ماجلان في بازيلا، وباستخدام طيف الجسم، تمكن الباحثون من قياس الكتلة الهائلة للثقب الأسود الهائل الموجود في المركز. من الكوازار. المصدر: إدواردو بانيادوس/مراصد كارنيجي وشياوهوي فان/UA.
تُظهر الصورتان الصغيرتان الموجودتان أعلى الصورة الفرق في سطوع الكوازار في الضوء المرئي والأشعة تحت الحمراء. في حين أن الكوازار مرئي بوضوح في صورة الأشعة تحت الحمراء التي التقطها تلسكوب ماجلان (على اليمين)، إلا أنه لا يمكن رؤيته في الضوء المرئي على الإطلاق، لأن تأثير دوبلر تسبب في تحويل ضوءه المرئي إلى أطوال موجية أطول - مثل الأشعة تحت الحمراء. يوجد في الجزء السفلي من الصورة طيف الكوازار، كما تم تسجيله بواسطة أجهزة قياس الطيف في اثنين من تلسكوبات ماجلان في تشيلي. ومن خلال تحليل طيفه، تمكن الباحثون من قياس الكتلة الهائلة للثقب الأسود الهائل الموجود في الكوازار. مصدر: إدواردو بانيادوس/مراصد كارنيجي وشياوهوي فان/UA.

المجرة التي يوجد فيها الثقب الأسود فاجأت الباحثين أيضًا

باستخدام مجموعة التلسكوب الراديوي نويما في جبال الألب الفرنسية ومجموعة كبيرة جدًا (VLT) في نيو مكسيكو بالولايات المتحدة الأمريكية، قام فريق ألماني من الباحثين من معهد ماكس بلانك لعلم الفلك بفحص خصائص المجرة التي يوجد فيها الثقب الأسود الهائل. قاموا بنشر النتائج التي توصلوا إليها في مقال مصاحب للمقالة الرئيسية في مجلة The Astrophysical Journal Letters.

واكتشف الباحثون لدهشتهم أن المجرة، التي تعود بالطبع أيضًا إلى تلك الفترة القديمة التي تلت الانفجار الكبير بـ 690 مليون سنة، تحتوي على كميات عالية جدًا من الغبار والعناصر الثقيلة. بسبب السطوع الهائل للكوازار في مركز المجرة، لا يتمكن الباحثون من مراقبة ضوء النجوم مباشرة، على الأقل ليس باستخدام التلسكوبات الموجودة. ومع ذلك، فإن حقيقة احتوائها على كمية أكبر من العناصر الثقيلة، التي تتشكل في مراكز النجوم، تشير إلى أن المجرة ربما تحتوي على كمية كبيرة من النجوم.

وكتلة جميع نجوم المجرة البعيدة قابلة للمقارنة، بحسب تقديرات الباحثين تحذير لأنها ليست دقيقة تماما، حوالي 20 مليار مرة كتلة شمسنا. إذا قارنت ذلك بمجرة درب التبانة، فإن كتلة كل النجوم الموجودة فيها تتراوح ما بين 40 إلى 60 مليار مرة كتلة الشمس، وهذه كمية كبيرة جدًا من النجوم، بالتأكيد في مثل هذه الفترة الفتية للكون.

وهذا الاكتشاف المتعلق بالمجرة التي تحتوي على الكوازار البعيد سيسمح، بحسب الباحثين، بتنقيح وتنقيح النماذج الموجودة فيما يتعلق بتكوين المجرات.

وقال الباحث برام فانيمانز من معهد ماكس بلانك لعلم الفلك: "نماذج تطور المجرات يجب أن تشرح كيف تكون المجرة قادرة على تشكيل النجوم اللازمة لإنتاج الكميات المرصودة من الغبار والعناصر الكيميائية الثقيلة في مثل هذا الوقت القصير نسبيا".

ماذا بعد؟

سيصبح الثقب الأسود الهائل، وكذلك المجرة التي يقع فيها، هدفًا لملاحظات المتابعة. وقد تم بالفعل تأكيد العديد من الأرصاد، باستخدام تلسكوب هابل الفضائي الذي سيرصد الكوازار في المدى القريب من الأشعة تحت الحمراء، وتلسكوب سبيتزر الفضائي في نطاق الأشعة تحت الحمراء، وتلسكوب شاندرا الفضائي الذي سيرصده بالأشعة السينية. في مجموعة التلسكوبات الراديوية تم تأكيدها ALMA في صحراء أتاكاما في تشيلي، من أجل مواصلة دراسة المجرة المضيفة للكوازار.

المرصد الفضائي الذي يعتبر الخلف المستقبلي لتلسكوب هابل الفضائي، تلسكوب جيمس ويب الفضائي، والذي من المتوقع إطلاقه في عام 2019 إلى الفضاء، سيساعد بشكل كبير في الدراسة المستقبلية لكل من هذا الكوازار والكون الشاب. سيقوم التلسكوب بمراقبة الفضاء في نطاق الأشعة تحت الحمراء للسماح له بالنظر إلى أبعد مسافة ممكنةللكشف عن الأدلة من النجوم والمجرات من فترة سابقة للكون، تصل إلى حوالي 13.5 مليار سنة مضت.

للمزيد حول هذا الموضوع على موقع العلوم:

تعليقات 319

  1. إن المعلومات التي تفيد بوجود مثل هذا الثقب الأسود أمر مخيف حقًا عند التفكير في حجمه الآن لأنه حدث نسبيًا في بداية الكون، لذلك الآن بعد مرور مليارات السنين تمكن بعدها من أكل عدد قليل من النجوم (عندما كان أسودًا) الثقب يأكل نجما فتزداد كتلته) فهو الآن ثقب أسود له كتلة ضخمة وحجمه هكذا

  2. إن المعلومات التي تفيد بوجود مثل هذا الثقب الأسود أمر مخيف حقًا عند التفكير في حجمه الآن لأنه حدث نسبيًا في بداية الكون، لذلك الآن بعد مرور مليارات السنين تمكن بعدها من أكل عدد قليل من النجوم (عندما كان أسودًا) الثقب يأكل نجماً فتزداد كتلته) فهل الآن ثقب أسود له كتلة ضخمة بحجمه حتى أن الكرة في بلادنا لا شيء بالنسبة له؟

  3. لقد عدنا من سانتا باربرا.

    ملخص المناقشة.

    في مقال "التهديد الكمي للنظرية النسبية" في مجلة Scientific American، يُزعم أن عدم الموضعية في التشابك الكمي، والتي تتضمن تأثيرًا فوريًا على مسافة، تتعارض مع النظرية النسبية الخاصة التي بموجبها لا يمكن لأي تأثير أن ينتقل أسرع من الضوء.

    يذكر المقال أنه على الرغم من أن الفيزيائيين والفلاسفة اعتقدوا لسنوات عديدة أن ما يسمى بالتناقض غير موجود، إلا أنه في السنوات الأخيرة ظهرت المزيد والمزيد من الأصوات التي تقول إن التناقض موجود بالفعل.

    تفسير السبب هو، في رأيي، مقال آخر يصف تجربة فايلر، والتي بموجبها يمكن أن يتأثر الماضي بالمستقبل.

    ووفقا للنظرية النسبية الخاصة، فإن السرعة على أوريت تعني رحلة إلى الماضي. على الرغم من أن هذا لا يتعارض بشكل مباشر مع الكوسيلية، إلا أن الرحلة إلى الماضي أمر يصعب قبوله من الناحية المادية. إيلي بات - بواسطة.

    لقد طرحت فكرة أنك لست بحاجة إلى استخلاص التشابك واللامكانية لترى أن الكميات والعلاقات غير متطابقة. في أصل التجربة الفكرية التي تطورت منها النسبية كما هو مكتوب في مقال أينشتاين الأصلي عن النسبية، يوجد فوتون ذو زخم محدد وموقع محدد في لحظة محددة. على الرغم من أن هذه الفكرة كانت منطقية تمامًا في عام 1905، إلا أنها تتناقض مع مبدأ عدم اليقين، وهو المبدأ الأساسي لميكانيكا الكم.

    وتتجلى المشكلة في الفقرة التالية:

    "يمكنك إصدار فوتون من النقطة A وبعد وقت مناسب تستقبله عند النقطة B.

    أما إذا كانت منتشرة في الفضاء من قبل، وباحتمال متساو، فهناك احتمالان:

    1. إن حقيقة الفوتون فيما بعد B ليست شيئًا ماديًا - فهي مجرد وظيفة موجية أو فلسفة أو علم فيروسات الحمار الوحشي، ولكن ليس الفوتون نفسه.

    (غريب بعض الشيء إذا اعتبرنا أن مربع الدالة الموجية هو احتمال العثور على الجسيم عند نقطة معينة).

    2. الجسيم نفسه أيضًا يتجاوز B.

    في الحالة الأولى، لا توجد حقائق بالنسبة للفوتون الذي يتحرك بشكل أسرع من c، وأينشتاين على حق والكم يندم. لكن حسب فهمي، ليس هذا هو الحال.

    في الحالة الثانية، إذا كان الفوتون نفسه يتجاوز B في وقت أقل من ct، فلا مفر من الاستنتاج بأنه يتحرك بشكل أسرع من c وفي الواقع بأي سرعة.

    وما زلت لم أتلق إجابة من أي شخص هنا حول السؤال الذي طرحته في السطر الأخير:

    "في الحالة الثانية، إذا كان الفوتون نفسه يتجاوز B في زمن أقل من ct، فلا مفر من الاستنتاج بأنه يتحرك بشكل أسرع من c وفي الواقع بأي سرعة".

    فهل يتطوع أحد هنا للشرح؟

    آرييل، عندما قرأت التعليقات اكتشفت أن ردك كان معلقًا في ذلك الوقت. ربما ستحاول الإجابة والإثبات بأم عينيك أن هناك أشياء ليس لديها ببساطة إجابة لا لبس فيها بعد؟

    دار نقاش بيني وبين نسيم حول غرابة النسبية وخاصة تطويل الطول، لكننا لم نكمل في النقاط الأخرى:

    إذا كان نموذج ماكسويل خاطئًا ولا يوجد أثير، فكيف تمكن من استخدامه للعثور على سرعة الضوء؟ ففي النهاية هو غير موجود!

    في مفارقة التوأم الأصلية، يمكن لكل توأم أن يدعي أن وقته هو الوقت الحقيقي. لكن إذا كانت نظرية الانفجار صحيحة، فهناك زمن مطلق يتم التعبير عنه أيضًا بدرجة حرارة الكون وفقًا لصيغة فريدمان. لذلك عندما يلتقي التوأم مرة أخرى، فإن وقت التوأم المتبقي هو وقت الكون بينما وقت المسافر هو وقت اصطناعي.

    وتطوير هذه الفكرة يعني أننا إذا عكسنا التكوين وكان التوأم المتحرك ثابتا بالنسبة للإشعاع، فإن زمن التوأم المتبقي يكون أعلى من عمر الإشعاع وبالتالي عمر الكون أيضا.

    فكيف يكون زمنه أعلى من عمر الكون الذي هو جزء منه؟

    أخيرًا، تم توضيح نقطة شخصية يسعدني أن أتناولها في هذا المقال.

    لسنوات عديدة، اتهمت آي ألبينزو بادعاءات مختلفة، أولها أنني كاذب وقح.

    طلبت منه الاعتذار لكنه رفض وقدم أيضًا 3 أمثلة في هذا المقال على أنني كاذب:

    "نعم، لقد كذبت بالتأكيد في المناقشات العديدة التي أجريناها. من المضحك أن تتوقع مني الآن أن أعرف كيف أقتبس لك بالضبط أين وكم ولماذا وكيف مع الأخذ في الاعتبار أن هذه مناقشات كانت بعضها قبل 3 أو 4 سنوات، ولكن لدي بعض الأمثلة في ذهني. أحد الأمثلة على ذلك هو أنك ستعود لفترة طويلة من الزمن وتكتب اقتباسًا لي مرارًا وتكرارًا، والذي سيتم قطعه دائمًا بأعجوبة في منتصف الجملة. لقد أوضحت لك مرات عديدة أنك تستشهد بنصف اقتباس وبالتالي تغير ما قلته بالكامل، لكن ذلك لم يزعجك. يتعلق الجزء الأول من الجملة بحقيقة أن هناك أشخاصًا لا يفهمون ميكانيكا الكم وبالتالي يزعمون أنها خاطئة - وأنت لديك نفس المشكلة. الجزء الذي كنت قد حذفته يوضح أنني لا أتهمك بإنكار ميكانيكا الكم ولكن بنفس النوع من الخطأ - أنت لا تفهم شيئًا ما، وبالتالي تستنتج أنه ربما يكون خطأ. مثال آخر هو أنه خلال مناقشة طويلة حول حل لغز واحد أو آخر من العملات المعدنية بمساعدة التشابك، زعمت أنني قلت أن لديها حلًا كلاسيكيًا على الرغم من أنني كتبت صراحةً عدة مرات أنه ليس لديها حل كلاسيكي. كنت أحاول فقط أن أشرح لك أن الحل الكمي لا يتضمن نقل المعلومات لأنه يتطلب المزامنة المسبقة لعدد كبير من الأزواج المتشابكة - وهو المعادل الكمي للتنسيق المسبق للكود، والغش وفقًا لقواعد اللعبة. ومع ذلك فقد اتهمتني مرارا وتكرارا بالادعاء بأن هناك حلا كلاسيكيا.

    من الصعب بعض الشيء معرفة ما يعنيه لأنه على عكسي الذي يقتبس دائمًا في المصدر، لا توجد اقتباسات هنا وعليك أن تثق في الكاتب بأنه يعرف ما يقوله. ولكن إذا سألني شخص ما، سأكون سعيدًا بإحضار تلك "الاقتباسات التي تم قطعها دائمًا بأعجوبة في منتصف الجملة". أتذكرهم وأعرف أيضًا مكانهم وأعتقد أنهم لن يؤدي إلا إلى تفاقم وضع ألبينزو وادعاءاته الكاذبة.

    "مثال آخر هو أنه خلال مناقشة طويلة حول حل لغز واحد أو آخر من العملات المعدنية بمساعدة التشابك، زعمت أنني قلت أن لديها حلًا كلاسيكيًا على الرغم من أنني كتبت صراحةً عدة مرات أنه ليس لديها حل كلاسيكي. كنت أحاول فقط أن أشرح لك أن الحل الكمي لا يتضمن نقل المعلومات لأنه يتطلب المزامنة المسبقة لعدد كبير من الأزواج المتشابكة - وهو المعادل الكمي للتنسيق المسبق للكود، والغش وفقًا لقواعد اللعبة. ومع ذلك فقد اتهمتني مرارا وتكرارا بالادعاء بأن هناك حلا كلاسيكيا.

    وهذا ما كتبه ألبينزو:

    "لا يوجد فرق بين هذا الحل وأي حل آخر للكود الكلاسيكي المنسق مسبقًا (كما حاول بعض المعلقين هنا اقتراحه)، باستثناء أن هذا الكود يعمل وفقًا لقوانين الكم، وعلى وجه الخصوص فهو موجود في التراكب (كما نعلم فإن سر سحر التشابك هو أنه حالة تراكب، وليس مجرد تراكب خاص لا يمكن تعميمه على أنظمة فرعية خالصة)'.

    أنا على استعداد للمراهنة على أي شخص يريد ذلك بأي مبلغ أنه إذا أخذنا عينة عشوائية من 100 شخص وقرأنا لهم الجملة الأخيرة، فإن الغالبية العظمى ستعتقد أن هذا يعني أنه لا يوجد فرق بين الكلاسيكية المنسقة مسبقًا الكود والحل الكمي، باستثناء بعض التفاصيل الهامشية. بالتأكيد ليس بشكل رئيسي كما يوافق ألبينزو أيضًا - أنه من خلال الحل الكمي من الممكن التأثير من مسافة بعيدة في زمن الصفر، وهو موضوع المناقشة.

    صياغة سيئة من ألبينزو، ليست كذبتي. إذا، كما قال، "فهم القراءة الخاص بك معيب". عندما يقولون "لا يوجد فرق بين موشيه وداني إلا أن موشيه أشقر وداني أحمر"، فهل يعني ذلك أن موشيه وداني هما نفس الشيء؟ عندما تقابلهم، هل ستتمكن من التمييز بينهم أم لا؟ "لكن إذا نسيت أن تذكر أن الجينغهام معروفون بقدرتهم على التأثير من مسافة بعيدة في زمن الصفر، وهو موضوع المناقشة، فقد فهمت ما يسمى بالدكتور لوكا. يبدو الأمر كما لو أنه لا يوجد فرق بين فيلم تيتانيك من عام 1953 وفيلم تيتانيك من عام 1997 إلا أن الأبطال في فيلم 1997 هم أشقر وأحمر الشعر. هؤلاء الأبطال - جاك وروز - هم تقريبًا أهم شيء في الفيلم.

    "هذا مثال ممتاز على التحريف المجنون لكلماتي وحتى الاختراع: "هل تدعي أنهم في النص الفرعي كانوا يقصدون العكس بالفعل وأنه لا يوجد أي تهديد وأنهم على وفاق كبير وأن مؤلفي الكتاب مقال مفبرك شيئا خياليا لزيادة تداوله؟" أنت تعلم جيدًا أنني لم أقل شيئًا كهذا أبدًا. ولم أكتب أنهم اختلقوا أي شيء. لم أكتب أنهم يقصدون العكس.

    وهذا ما كتبه ألبينزو قبل أيام قليلة:

    "بادئ ذي بدء، عليك أن تتذكر أن كل مقال في مجلة - حتى لو كانت علمية، وبالتأكيد في مجلة ساينتفيك أمريكان، التي تخاطب عامة الناس ولديها اهتمام كبير ببيع النسخ - يمكن أن يقع في ميل طبيعي للإثارة أو عناوين منمقة."

    ليست مشابهة جدا؟

    "لقد كذبت مرات عديدة، لأن حججك في أغلب الأحيان فارغة من المضمون، بسبب ميلك القوي للحديث عن أشياء ليس لديك فكرة عنها. تمامًا كما يحدث الآن، عندما تكتب المزيد والمزيد من الأشياء عن المعلومات عندما يكون من الواضح للجميع - بما فيهم أنت - أنك ببساطة لا تعرف أي شيء عن نظرية المعلومات، بما في ذلك التعريفات الأساسية لماهية المعلومات أو ما هو أبعد من المعلومات .'

    هل ادعيت يومًا أنني أعرف نظرية المعلومات؟ أو هل كنت أعلم أن نظرية المعلومات كانت موجودة قبل أن يذكرها أليساندرو؟ كنت أتحدث عن المعلومات كما يتحدث عنها الجميع هنا.

    وبما أن المراجعين وعلماء الأحياء وغيرهم الكثير يستخدمون أيضًا كلمة "معلومات" دون إعطاء تعريفها الدقيق، فلا أشعر بأن الأمر غريب بشكل خاص، أو ربما نكون جميعًا كاذبين وقحين بالفعل.

    خلاصة القول، يمكن لأي شخص أن يقول جملة غير صحيحة وهذا لا يجعله كاذبا. الكاذب هو الذي، في ظل الظروف المعيارية، يفشل في آلة الكذب. وقال ألبينزو جملة غير صحيحة - "إن عدم انتقال المعلومات بين الجزيئات المتشابكة ليس معرفة. هناك دليل رياضي. فهل هذا يجعله كاذبا؟ لا أعتقد ذلك، لأن هذا ما يعتقده، أو على الأقل ما كان يعتقده في ذلك الوقت.

    وهناك أيضًا مقال في مجلة Scientific، والذي يقول بالضبط ما أقوله دائمًا، لذلك إذا كنت كاذبًا، فمن المحتمل أنهم كذلك أيضًا.

    باختصار - لا كذاب ولا كذاب. أنا لا أبحث عن شجارات، وآمل أن يغلق هذا الموضوع للأبد.

  4. قلت: ليس هناك دليل مباشر على تقصير المدة، وهذا بخلاف الأدلة المباشرة الموجودة على تطويل الزمن.

    في الواقع، لم أقل ذلك - يقول فيكي.

    "وكما قلت، "جذر الاحتمال" هو شيء غريب."

    قلت مربع وهذا ليس غريبا، هكذا تجد الاحتمال.

    "الخلل الذي أحاول حله الآن غريب، وهذا لا يعني أنه غير موجود."

    إنه أمر غريب بالفعل، حتى تفسيرك لبلد ألفا. لم أقل أنها غير موجودة، ولكن من المرغوب فيه أن تكون النظرية الفيزيائية واضحة وبسيطة، انظر نظريات نيوتن وماكسويل.

    "هذا يعني أن "المعلومات غير المفيدة" يمكن أن تتحرك بسرعة لا نهائية."

    دورنا في السلطة هو أن نجعلها مفيدة. تريد - الانضمام. لا، سوف نتدبر أمرنا بأنفسنا.

    الذهاب إلى سانتا باربرا.

  5. إسرائيل
    لقد بدأنا بحقيقة أنك زعمت أن أحد تنبؤات النسبية الخاصة - وهو انكماش الطول - غريب، ولم يتم إثباته تجريبيًا.
    لقد أوضحت لكم بعض الظواهر التي لا نعرف كيف نفسرها، دون انكماش الطول. إن القول "إنه غريب" ليس شيئًا أعرف كيفية التعامل معه. الخطأ الذي أحاول حله الآن غريب، وهذا لا يعني أنه غير موجود.

    أنت لا توافق على ما يدعيه أينشتاين بشأن النسبية الخاصة، ولكنك تتفق مع رأيه عندما يكون ذلك مناسبًا لك. ويبدو لي أن فهمه لمعنى EPR خاطئ. يمكن أن تتغير الاحتمالات بسرعة لا نهائية. هناك احتمال معين أن تكون مفاتيحي على الطاولة - لأنني أحيانًا أضعها هناك وأحيانًا في الدرج. لقد تحققت - وها هم في الدرج. في المرة 0، انخفض احتمال وجودهم على الطاولة إلى 0 🙂

    أعلم أن هذا ليس التفسير الكامل، وكما قلت، "الجذر الاحتمالي" شيء غريب. ولكن هذا يعني أن "المعلومات غير المفيدة" يمكن أن تنتقل بسرعة لا نهائية.

  6. عندي؟ أين في المستودع؟ لماذا من أنا رامي ليفي؟

    لكن نعم، لها وجود مستقل حتى بدون النسبية، وحتى منذ ما قبل عام 1905.

    https://en.wikipedia.org/wiki/Olinto_De_Pretto

    لست متأكدًا من أننا متفقان بشأن دورنا في القوة.

    ليس من واجبنا أن نشرح كل شيء. ليس لدينا الأدوات أو التدريب أو الخبرة أو الوقت أو الموهبة اللازمة لذلك.

    مشكلتنا أبسط من ذلك بكثير: كتب أينشتاين ورقة بحثية - EPR - يدعي فيها أن الندم الكمي يرجع إلى أن الجسيمات تؤثر على بعضها البعض بشكل أسرع من الضوء. يقدم تجربة فكرية لإثبات وجهة نظره.

    يتم إجراء نسخة مختلفة من التجربة ويفوز الكم.

    وفقًا للكميين، ليس للفوتون موقع قبل القياس، وينتشر باحتمال متساوٍ في الكون.

    يسأل لطيف:

    "هل تقبل أنه يمكنني إصدار فوتون من النقطة أ وبعد وقت مناسب أستقبله عند النقطة ب؟"

    إسرائيل تجيب:

    "بلا شك. يمكنك إصدار فوتون من النقطة A وبعد وقت مناسب تستقبله عند النقطة B.

    أما إذا كانت منتشرة في الفضاء من قبل، وباحتمال متساو، فهناك احتمالان:

    1. إن حقيقة الفوتون فيما بعد B ليست شيئًا ماديًا - فهي مجرد وظيفة موجية أو فلسفة أو علم فيروسات الحمار الوحشي، ولكن ليس الفوتون نفسه.

    (غريب بعض الشيء إذا اعتبرنا أن مربع الدالة الموجية هو احتمال العثور على الجسيم عند نقطة معينة).

    2. الجسيم نفسه أيضًا يتجاوز B.

    في الحالة الأولى، لا توجد حقائق بالنسبة للفوتون الذي يتحرك بشكل أسرع من c، وأينشتاين على حق والكم يندم. لكن حسب فهمي، ليس هذا هو الحال.

    في الحالة الثانية، إذا كان الفوتون نفسه يتجاوز B في وقت أقل من ct، فلا مفر من الاستنتاج بأنه يتحرك بشكل أسرع من c وفي الواقع بأي سرعة.

    مهمتنا بسيطة نسبيًا: اكتشاف جزء الفوتون الذي يقع بعد B.

    كابيش؟

  7. إسرائيل
    عاشق البساطة؟ ما هو التفسير البسيط للميونات؟ لماذا عندما أضع الساعة على مبنى مرتفع فإنها تعمل بشكل أسرع؟ لماذا ينحني الضوء بالقرب من الشمس؟ لماذا تزداد الكتلة مع السرعة؟

  8. طبعا لا يضر الكوزيلي ولكن تأثير على الماضي؟ ألم نقول أن إيلي فات - بواسطة؟

    ألا ترى أن الأمر أصبح أكثر تعقيدًا، وأكثر تعقيدًا، وأكثر تعقيدًا؟

    وكمحب للبساطة، ألا ترى أن الفوتون المتحرك بكل السرعات يحل معظم المشاكل بما في ذلك التأثير على الماضي؟

  9. https://www.hayadan.org.il/quantun-philospy-part-b-07121

    وهذا الشيء المشترك بين الجزيئات لا ينتقل من واحدة إلى أخرى في زمن صفر؟ ففي النهاية، بدأ بالجسيم الذي قمنا بقياسه وانتقل على الفور إلى الجسيم الثاني.

    قال فاينمان بالفعل إنه لا أحد يفهم الكمات، ولكن على الأقل في مسألة التأثير المباشر عن بعد ثبت صحتها، وهذا يتناقض (على الأقل وفقًا لويكي) مع النسبية.

  10. إسرائيل
    هناك شيء مشترك بين الجزيئات. ولكن أين تأثير المستقبل على الماضي؟

    لا أفهم كيف ينجح شيء "بسيط" مثل تجربة الفتحتين بالنسبة لك...

  11. يوجد أيضًا شرح بسيط للمغناطيسية يظهر في الرابط الذي قدمته.

    وفيما يتعلق بألفا - عند التكبير العادي في مرحلة ما، ستختفي معظم النجوم من الصورة، لكن هذا لا يمكن أن يحدث هنا.

    بالمناسبة، إذا أردت، فأنا مشترك في التلسكوب عبر الإنترنت الذي يمكنك من خلاله رؤية كل نجم ومجرة من اختيارك مباشرة. يمكنني أن أعطيك التفاصيل، هذا جميل جدًا.

    لكنني ما زلت لا أفهم: هل لا تقبل أن الجسيم المتشابك A يؤثر على الجسيم B في 0 مرة؟ حتى لو لم نسميها نقل المعلومات، على الرغم من أنها تقوم بطريقة ما بترتيب الجسيم B في نفس الحالة الكمومية لجسيمه، إلا أنك لا ترى مدى تحديه للنسبية التي بموجبها لا يمكن لأي شيء أن يتحرك بشكل أسرع من الضوء (دعونا اترك التاكيونات في الوقت الحالي).

    ألا ترى أن ذلك يؤدي إلى التأثير على الماضي من المستقبل؟ أنت لا تصدق أن إيلي فات - بواسطة؟

  12. إسرائيل
    وهنا تفسير آخر:
    https://readingfeynman.org/2015/09/06/magnetism-and-relativity/

    لقد تعلمت ذلك منذ وقت طويل، من كتب بيركلي. الكتب ليست معي....

    حول الكاميرا. لا أفهم كيف تعطي فكرتك نتائج مختلفة. لنفترض أنني أقترب من القمر بسرعة قريبة من c. لنفترض أن هناك جهاز إرسال يبث الوقت كل ثانية. هذا يعادل الفوتون مع الساعة، أليس كذلك؟ ووفقًا لطريقتك أيضًا، يتحرك القمر والفوتونات بسرعات متقاربة. أي أنني سأستقبل فوتون الساعة 12:00 قريباً جداً من الساعة القمرية 12:00. ما هو استنتاجي؟ أن القمر قريب جداً.
    أي أنه حتى في اقترابك هناك تقلص في المسافة...

  13. المشكلة هي أنه حسب الفيديو الذي أحضرته فقط من وجهة نظر الشواحن مزدحمة..

    ولم أفهم كيف تعرف كاميرا الفيديو الموجودة على جهاز iPhone الخاص بي أن Alpha Country هي التي أريد التقاط صورة لها وبالتالي تقوم بتكبيرها فقط وليس حوالي 1000 نجم آخر في المنطقة المجاورة.

  14. إسرائيل
    وجهة نظرك لا تؤثر على قيمة القوة. فكر في الحالة المعاكسة - تخيل وجود تيارين من الإلكترونات في الفراغ. لماذا لا يوجد تجاذب مغناطيسي بينهما (على عكس التيار في الأسلاك)؟

  15. إسرائيل
    الكاميرا لا تعرف شيئا. فهو يلتقط الفوتونات التي تنتقل بسرعة الضوء. ولو كانوا يتحركون بسرعة أكبر من سرعة الضوء، لحصلنا على تناقض.

    لنضع سلكين متوازيين في مستوى في اتجاه الشمال. لنفترض أنه في أحدهم يتدفق تيار مستمر نحو الشمال. أي أن الإلكترونات تتحرك جنوبًا.

    1) لا يوجد تيار في السلك الثاني . في هذه الحالة ترى كل من البروتونات والإلكترونات شحنة سالبة في السلك الأول. مجموع القوى - 0.

    2) في السلك الثاني كان هناك تيار أيضًا في اتجاه الشمال. في كلا السلكين - ترى الإلكترونات المزيد من البروتونات، وهذا يخلق جاذبية. وترى البروتونات عددًا أكبر من الإلكترونات، مما يؤدي إلى الجذب مرة أخرى.

    3) عكس التيار في السلك الآخر. ترى الإلكترونات عددًا أكبر من البروتونات، ولكن المزيد من الإلكترونات، وبالتالي تتنافر. ترى البروتونات المزيد من الإلكترونات وهذا يخلق جاذبية معينة. بشكل عام، هناك المزيد من التنافر، وبالتالي فإن الأسلاك تتنافر مع بعضها البعض.

    وينبغي التأكيد على أن هذا لا يعني عدم وجود مجالات مغناطيسية. كلا النوعين من الحقول حقيقيان، ولكن هناك تبعية للنظام المرجعي.

  16. بالنسبة لي إنه أمر غريب.

    كيف تعرف الكاميرا أنني أريد هذا النجم بالذات في الصورة والذي يبعد عني 4 سنوات ضوئية؟ لماذا لا الذي بجانبه؟

    وما أهمية القوة الناتجة عن الشحنات الموجودة في السلك الموصل والتي أحركها بالنسبة إليها؟ حسنًا، من وجهة نظري هم متجمعون معًا، لكن من المفترض أن تؤثر وجهة نظري على القوة البدنية؟

    لم أتحقق، ولكن تم التأكد من أنه إذا تم عكس اتجاه حركة الشحنة أو اتجاه التيار على سبيل المثال، فإن القوة ستنعكس أيضًا، فهل يستمر رفضها؟

  17. إسرائيل
    أعتقد أنك على حق في ذلك. فقط لاحظ أن هذا يعني أنه في ظل الظروف "العادية"، ستظل النسبية الخاصة صحيحة (مثل نظرية نيوتن صحيحة في ظل ظروف معينة).

    بالمناسبة - لا أفهم ما هو الغريب في تشويه الصورة. ومن المنطقي جدًا أن تعتقد أنك إذا اقتربت من سرعة الضوء فسوف ترى أشياء غريبة. وهذا هو الحال أيضًا مع الصوت. تشعر بالإحباط من أمواج السفينة البطيئة، التي تبدو وكأنها تموج من حجر مقذوف، مقارنة بالسفينة السريعة التي تصنع حرف V على الماء. الأمر نفسه ينطبق على الطائرات المقاتلة، فالعين ترى أشياء غريبة جدًا في الهواء الأسرع من الصوت، مثل الماس في غازات العادم، أو خطوط التشقق في الهواء بالقرب من المخاريط. الطيران الأسرع من الصوت يخلق دويًا غريبًا، والجسيمات التي تتحرك فوق سرعة الضوء (في نفس الوسط) تخلق إشعاعًا غريبًا. الغريب لا يعني أن شيئا ما غير موجود.

  18. لا يوجد ادعاء بعدم التناقض هنا، وكما هو مكتوب في مجلة Scientific، فإن التهديد الكمي للنسبية ظهر بعد سنوات عديدة من ديراك.

    لاحظ أيضًا أن ما أقوله هو أنه ربما يتعين مراجعة النسبية وتصبح حالة خاصة لنظرية أكثر عمومية ستشمل اللامكانية كما حدث لنظرية نيوتن التي تعد حالة خاصة من النسبية.

  19. إسرائيل
    سأقتبس مرة أخرى -

    في الواقع، كانت إدارة الجودة ستقدم تنبؤات غير دقيقة إلى حد كبير إذا لم يظهر ديراك وربط إدارة الجودة مع النسبية الخاصة لإنشاء "إدارة الجودة النسبية".

    النسبية العامة من ناحية أخرى

    إنه أمر غريب، لكن هذا لا يعني أنه غير صحيح. تعتقد أنك تقترب من ملصق كبير. محتوى الصورة لا يتغير بل هو مشوه للغاية. لا يختلف الأمر كثيرًا عن التصوير عالي السرعة.

  20. "يزعم المقال أنه لا يوجد تناقض بين النسبية الخاصة ونظرية الكم، بل العكس".

    أين وأين يدعي ذلك؟

    إذن من لديه مقالة أكبر؟

    إلى العلمية. حوالي 30 مرة.

    "سوف ترى كل النجوم التي رأيتها من قبل، ولن ترى نجماً واحداً لم تره من قبل."

    لكنك قلت سابقًا أنه إذا كانت جاما تساوي 10 فإن القمر سيشغل 10 أضعاف الصورة..

    لذا، إذا كانت قيمة جاما تبلغ مليونًا فلن تملأ الصورة بأكملها؟

    ما هذا، كل النجوم سوف تتقلص إلى زاوية الصورة؟

    وكما قلت غريب جداً

  21. إسرائيل
    لقد سبق أن قلت - المقال يؤكد أن المشكلة في النسبية العامة وليست النسبية الخاصة. إذا أصررت على قراءة كل جملة على افتراض أن ما تعتقده صحيح، فلا فائدة من مناقشته.

  22. إسرائيل
    انت كتبت:
    "ولنفترض أنك قمت بتصوير القمر عندما تبلغ قيمة جاما مليونًا.
    ماذا ستشاهد في الصورة مع هذا التكبير النسبي؟ فقط فم تايكو؟
    إذن، أين ذهبت كل النجوم الأخرى في الخلفية؟ ففي نهاية المطاف، تصل الفوتونات الصادرة منها إلى عدسة الكاميرا أيضًا، فأين ذهبت؟

    سوف ترى كل النجوم التي رأيتها من قبل، ولن ترى أي نجوم لم تراها من قبل.

  23. فقط العنوان؟ قرأت الفقرة الأولى كاملة :

    الفيزيائي: ميكانيكا الكم (QM) والنسبية كلاهما دقيقان بنسبة 100%، بقدر ما تمكنا من القياس (وقياساتنا جيدة حقًا). يظهر عدم التوافق عندما تكون تأثيرات إدارة الجودة والتأثيرات النسبية كبيرة بما يكفي ليتم اكتشافها ومن ثم عدم الاتفاق. يعتبر هذا الشرط نظريًا بحتًا اليوم، ولكن في السنوات القليلة المقبلة، يجب أن تؤدي مراقباتنا لبرج القوس A* وفي CERN إلى تسليط الضوء على المشكلات بين إدارة الجودة والنسبية.

    وخاصة الجملة الأخيرة:

    يجب أن يسلط الضوء على المشاكل بين إدارة الجودة والنسبية.

    من هو هذا الفيزيائي بالمناسبة؟

    اقرأ المقال العلمي مرة أخرى، فهو أقل حيرة من هذا المقال بكثير. ويوضح أنه في ذلك الوقت كانوا يعتقدون حقًا أنه لم تعد هناك مشكلة بين النسبية والكميات، ولكن في السنوات الأخيرة (بعد عشر سنوات من ديراك) أصبح الاعتراف بوجود مشكلة أقوى بين الفلاسفة والفيزيائيين.

  24. إسرائيل
    ماذا يكون؟ لقد التقطت كلمة واحدة في عنوان المقال، وتمسكت بها مثل كلب البلدغ...

    ربما ينبغي لي أن أترجم المقطع الذي أحضرته لك. حسنًا، لقد كتب في المقطع أنه لا يوجد تناقض بين النسبية الخاصة ونظرية الكم. ويقول ذلك على العكس من ذلك! أظهر بول ديراك أن النسبية الخاصة مطلوبة لتفسير الظواهر الكمومية.

    أين توجد مشكلة؟ إذا النسبية العامة. أحد الأمثلة على ذلك هو فقدان المعلومات في الثقب الأسود (حرب سسكيند-هوكينغ). والمثال الثاني هو ما أوضحه ألبانزو - مشاكل التفرد.

    إسرائيل، في الحقيقة، ستلتزم بما هو مكتوب. لا تنشغل بكل كلمة لأنك تعتقد أنها تعزز رأيك.

  25. إسرائيل
    بالطبع قرأت. اقتباس:
    في الواقع، كانت إدارة الجودة ستقدم تنبؤات غير دقيقة إلى حد كبير إذا لم يظهر ديراك وربط إدارة الجودة مع النسبية الخاصة لإنشاء "إدارة الجودة النسبية".

    النسبية العامة، من ناحية أخرى،

  26. حسنًا، معجزات حقًا، عليك أن تنتبه لما أكتبه قبل أن ترد عليه، وإلا فسيبدو الأمر تحديًا بعض الشيء..

    "الآن هل تقول أن هناك تناقض؟ اعتقدت أننا اتفقنا على أن القسم 1 كان غريبًا..."

    القسم 1 – الانكماش – غريب .

    القسم الخامس – الكم واللا محلة – ما يسمى بالتناقض.

    كابيش؟

  27. المعجزات

    مرة أخرى gloog gloog؟

    فقط فيما يتعلق بالقسم 1 الذي ذكرته - تقصير الطول - قلت غريبا، ولكن دون تناقض داخلي.

    كان هناك العديد من الأقسام الأخرى بما في ذلك القسم الذي تستمر في تجاهله: المقالة في مجلة Scientific، والتي لسبب ما تقول بالضبط ما أنا عليه.

    كل ما فعلته هو إظهار جذر ما يسمى بالتناقض، وهذه المرة دون تشابك وسرعة الضوء.

  28. إسرائيل
    أين تكلمت هراء؟ حيث لا؟ ؟

    وفي الختام، كل ما عليك قوله هو "هذا غريب". وبغض النظر عما يقولونه لك، بما في ذلك التجارب التي تؤكد النظرية النسبية بشكل مدهش.

    أفهم أنك تحاول التوصل إلى فرضية تناسب الملاحظات النسبية والكمية. أنت لست الوحيد - شموليك وصف فرضية بنروز، وذكر ألبينزو موضوع الأوتار.

    المشكلة هي أن ما تدعيه ضد النظرية النسبية هو في رأيي غير صحيح. أنت تقوم بتجربة فكرية متوهمة، وتتفاجأ بأن التجربة تظهر شيئًا وهميًا. أنت تزعم أنه لا يوجد دليل على انكماش الطول، وتتجاهل عددًا من الأدلة على ذلك، أحدها دليل قوي جدًا (من الأيونات من الفضاء). وهناك المزيد.

  29. معجزات، يجب أن تجتمعوا معًا.

    انتقل إلى جميع التعليقات - أين قلت أنك مخطئ؟ أنت من قلت أن ما أقوله خطأ ولكن عندما فهمت وافقت على أن يبدو النجم كبيرًا وقريبًا.

    بالتأكيد الكم أمر غريب، سأل أينشتاين، لقد حاربه طوال حياته - وخسر.

  30. إسرائيل
    لنفترض أن القمر بزاوية 90 درجة إلى الجانب. سيبدو Alpha Centauri ضخمًا. سيظهر القمر ضيقًا جدًا (لأنه سيبدو منقبضًا فقط على المحور 1).
    كما وصفت من قبل - ارسمها على قطعة من المطاط، ثم قم بمدها/تقليصها بقدر ما تريد.

    لن تأتي بأي شيء غير معقول.

  31. ولنفترض أنك قمت بتصوير القمر عندما تبلغ قيمة جاما مليونًا.

    ماذا ستشاهد في الصورة مع هذا التكبير النسبي؟ فقط فم تايكو؟

    إذن، أين ذهبت كل النجوم الأخرى في الخلفية؟ ففي نهاية المطاف، تصل الفوتونات الصادرة منها إلى عدسة الكاميرا أيضًا، فأين ذهبت؟

    ؟

  32. ولكن لماذا 10 مرات؟ وبعد كل شيء كتبت:

    النجم ألفا سنتوري (يبعد عنا 4 سنوات ضوئية) سريع بما يكفي بحيث يكون عامل جاما:

    يساوي 1000,000، وتلتقط صورة للنجم من الأرض. بما أن انكماش الطول يكون فقط في اتجاه حركتك وليس في الإحداثيات المتعامدة، فماذا ستظهر صورتك؟ — نجم ضخم أقرب من القمر؟

    ألا يقول مليون؟

  33. سلبي.

    أقوم الآن بتصوير القمر من الأرض، فهو يملأ جزءًا من مائة من الصورة.

    فكم سيكون حجمه إذا تم تصويره بنفس الكاميرا من مركبة فضائية تمر فوق الأرض بقوة غاما تساوي 10؟ كم من الصورة سوف تملأ؟

  34. إسرائيل
    كل من النجم والقمر يقتربان. تخيل أنك ترسم كسوف الشمس على وسادة مطاطية. ترسم المخروط الذي يظهر الإخفاء. الآن - قم بتمديد المطاط بالتساوي. بغض النظر عن كيفية القيام بذلك، ستبقى الخطوط المستقيمة مستقيمة.

    من السهل إثبات التقارب - خذ 3 نقاط على خط مستقيم وقم بإجراء التحويل. لكن ……. هنا اقتباس من ويكيبيديا (https://en.wikipedia.org/wiki/Length_contraction)

    في النسبية الخاصة، تحويلات بوانكاريه هي فئة من التحولات التقاربية التي يمكن وصفها بأنها التحولات بين مخططات الإحداثيات الديكارتية البديلة على زمكان مينكوفسكي المقابلة للحالات البديلة للحركة بالقصور الذاتي (واختيارات مختلفة للأصل). تحويلات لورنتز هي تحويلات بوانكاريه وهي تحويلات خطية (تحفظ الأصل)

    وهذا هو - ليس فقط تقاربي، ولكن أيضا خطي.

    هل يمكنك تلخيص النقطة 1؟

  35. لذلك دعونا نقول أن التحول هو تقاربي. هل النجم أقرب أم لا؟

    لأنه كقاعدة عامة عندما يكون الجسم أقرب فإنه يظهر في صورة أكبر. هل توافق على ذلك؟ فلماذا إذا كانت جميع الفضاءات الإقليدية متقاربة فهل سيكون الأمر مختلفًا في هذه الحالة؟

    إذن أنت تدعي أنه في هذه الحالة لن يبدو الأمر أكبر حجمًا بسبب خصوصية الموقف؟ ولا تظن أن الأمر غريب جدًا؟ ولماذا لم يذكر أينشتاين الصلات؟

    هل يمكنك توفير رابط بين تقصير الطول والنباتات الهوائية؟

  36. إسرائيل
    هذا كل شيء، لا. لقد سبق أن شرحت السبب عدة مرات، لذا سأحاول مرة أخرى بطريقة مختلفة. لنفترض أن الحركة تتم على طول المحور السيني بسرعة غاما = 2. هنا لدينا تحول:

    س => س/2
    ص => ذ
    ض => ض

    هذا التحول تقاربي. وهذا يعني أن هناك عدة أشياء تحدث هنا. على سبيل المثال - تظل الخطوط المتوازية متوازية. على سبيل المثال - تظل النقاط المستوية قطرية. فمثلاً - وهذا هو المهم بالنسبة لنا - النقاط الخطية تظل خطية.

    لقد قدمت قصة المركبة الفضائية وكسوف الشمس. وفي حالة الكسوف، إذا نظرنا من وجهة نظر الراصد للكسوف، فإن كل نقطة على محيط القمر تخفي بقعة ميتة على الشمس. وهذا هو - 3 نقاط على خط مستقيم.
    لذلك - عندما تكون المركبة الفضائية بالقرب من المشاهد، سيظل هناك كسوف.

    وهو المطلوب

    إذًا، هل ادعاءك بأن تحويل لورنتز ليس قيمة ذاتية؟

    إسرائيل – انتبه. أنا لا أدعي أن هناك مسافة تتقلص. وأنا أزعم أن ما قلته عن كون النجم أكبر من القمر خطأ.

  37. المعجزات

    المجنون الذي تقوله كما جنون المصري؟

    ما الخطأ عندما تتحرك في اتجاه النجم، أليست المسافة إليه أقصر بالنسبة لك؟ وإذا كانت المسافة أقصر، أليس النجم أقرب؟ أليس الجسم القريب مرئيًا في صورة أكبر؟ هل تريد تجربة المبنى المقابل؟

  38. يا معجزات للمرة الأخيرة، إذا فهمت - سوف تفهم. إذا لم يكن كذلك فلا.

    لا توجد تناقضات في علاقات بروبر - لكن استنتاجاتها غريبة للغاية وربما يكون تقصير الطول هو الأغرب على الإطلاق.

    إذا تحركت بسرعة نحو نجم فإن المسافة إليه تقصر، ولا يقل العرض.

    لذلك، بصرياً، ما هو أقرب يبدو أكبر، وعلى سطحه نرى النجم أكبر.

    هو كذلك ربما. ربما لا. ما يهم؟ وهو أمر غريب للغاية، على عكس نظريات ماكسويل بولتزمان وبرنولي النيوتونية التي ليست غريبة. هذا هو بيت القصيد في هذه المرحلة بالذات. وهذا ليس تناقضا مثل الكميات، بل مجرد تناقض.

    كابيش؟

  39. المعجزات

    حاول أن تتذكر من وقت لآخر أن هناك صلة بين الكلمات والواقع..

    "والآن تقول أنك لم تكتب ذلك؟"

    بالطبع كتبت ذلك. وإلا فمن أين نسخت لصقاً وليس أين كتبت؟

    وكتبت:

    ماذا سوف تظهر صورتك؟ – نجم ضخم أقرب من القمر؟”

    هذا سؤال، وهو يوضح ما أدعيه، وهو غريب جدًا.

    إذا كنت تعتقد أن التفسير الذي قدمته "التحول متقارب. على وجه الخصوص، هذا يعني أن النقاط الثلاث التي تقع على خط مستقيم قبل التحويل ستبقى على خط مستقيم بعد أن يكون صحيحًا، لذلك ليس هذا هو التفسير الوحيد. والشرح الذي قدمته عن الشحنات بأن المسافة بينهما تقصر مما يسبب قوة على الجسم المشحون المتحرك لم يكتمل أيضا، وهذا يظهر أيضا في الرابط الذي قدمته.

    عمل.

  40. إسرائيل
    انت كتبت
    "تخيل أنك تتحرك نحو النجم ألفا سنتوري (على بعد 4 سنوات ضوئية) بسرعة كافية بحيث يكون عامل جاما:

    يساوي 1000,000، وتلتقط صورة للنجم من الأرض. بما أن انكماش الطول يكون فقط في اتجاه حركتك وليس في الإحداثيات المتعامدة، فماذا ستظهر صورتك؟ - نجم ضخم أقرب من القمر؟"

    والآن تقول أنك لم تكتبه؟؟

  41. لا.

    لو كان ذلك صحيحاً لحصلنا على تناقض في العلاقة نفسها، أليس كذلك؟

    هل تعتقد أن أينشتاين كان سيفتقد التناقض في العلاقات الصحيحة؟

    وربما ستجيب بالفعل على سؤال المقال؟ لماذا نقول كلانا نفس الشيء؟

  42. إسرائيل
    وما أقوله هو ما يلي:
    لنفترض أن هناك كسوفاً للشمس. تقترب سفينة فضائية بسرعة قريبة وتمر بالقرب من الأرض وهي تتجه نحو الكسوف. وأزعم أن المركبة الفضائية ستشهد أيضًا الكسوف في لحظة مرورها بالقرب من الأرض. مما أفهمه - أنت تدعي خلاف ذلك.

    هل فهمتك بشكل صحيح؟

  43. دعونا نعمل حقًا بالترتيب، ونحاول رؤية الصورة كاملة.

    لذلك تكتب:

    1) تكتب أن النجم البعيد يجب أن يبدو أكبر من القمر. هذا ببساطة غير صحيح. التحول تقاربي. على وجه الخصوص، هذا يعني أن النقاط الثلاث التي كانت على خط مستقيم قبل التحويل ستبقى على خط مستقيم بعد ذلك.

    ومن هذا يمكن أن نستنتج أنني أدعي أن هناك مشكلة في تقصير الطول، هذا ما قصدته، أليس كذلك؟

    إذن هذا ما كتبته:

    على الرغم من وجود تناقضات رياضية داخل النظرية، إلا أن النسبية الفيزيائية غريبة جدًا لدرجة أن العديد من الفيزيائيين وجدوا صعوبة بالغة في قبولها. لا يزال علماء الفيزياء النظرية مثل أبراهام ولورنتز وبوانكاريه ولانجفين يؤمنون بوجود الأثير.
    على سبيل المثال، يصعب فهم انكماش طول لورينز بشكل خاص وليس له دعم تجريبي مباشر؛ http://math.ucr.edu/home/baez/physics/Relativity/SR/experiments.html#Length_Contraction
    اختبارات انكماش الطول
    في هذا الوقت لا توجد اختبارات مباشرة لانكماش الطول.

    الترجمة: على الرغم من عدم وجود تناقضات رياضية في النظرية النسبية نفسها، إلا أنها غريبة جدًا من الناحية الفيزيائية.

    ثم أنتقل للحديث عن تقصير الطول، وأذكر حقيقة (من ويكيبيديا) أنه ليس لها دليل تجريبي مباشر، وأعطي المثال حيث تتحرك أمام بلد ألفا بسرعة وتسأل عما إذا كانت ستظهر أكبر من القمر.

    فهل أدعي هنا أن تقصير الطول غير موجود؟ بالطبع لا، بعد كل ما قلته من قبل أنه لا يوجد تناقضات في العلاقات في حد ذاتها، المشكلة هي فقط المواجهة مع النظريات الأخرى.

    لقد تم طرح تقصير الطول فقط لإظهار أن النسبية تؤدي إلى استنتاجات تبدو في ظاهرها غريبة وصعبة للغاية على استيعابها. انظر كم وقعنا في مشكلة مفارقة التوأم وكم وقع في مشكلة 4 أساتذة معها وأخيراً توصل كل واحد منهم إلى تفسير معاكس من صديقه.

  44. طيب أوقف خيولك ..

    ربما بدلا من الصراخ "أنت لا تفهم، أنت لا تفهم!" والقول إن إسرائيل تفهم أفضل من أينشتاين، هناك أيضا احتمال أن نسيم لا يفهم تماما ما تقوله إسرائيل؟ 🙂

    أين تكلمت هراء؟ حيث لا؟ 🙁

    من افتتاح المحاضرة:

    في نهاية العرض ستدرك، أعتقد، أنه إذا كان أينشتاين مخطئًا على الإطلاق، فقد كان خطأً معقولًا جدًا ارتكبه.

    ويجب أن أضيف أيضًا أنه من المحتمل أنه لم يكن مخطئًا على الإطلاق، باستثناء ربما فيما يتعلق بموضوع سوء التصرف، وأنا ببساطة لا أفهم القضية جيدًا بما فيه الكفاية.

    سيكون من المفيد أن تجيب على السؤال الذي طرحته سابقًا:

    "لذلك ربما تشرح لي، أنت، المفسر الوطني، كيف يتم نشر مقال يقول بالضبط ما أقوله في مجلة Scientific؟" صدفة؟ حالة مخفضة؟ سحر؟"

    أنا لا أسأل إذا كان هذا صحيحًا أم لا، أو إذا كنت توافق أم لا، فقط كيف حدث أن كلانا يقول نفس الشيء بالضبط؟ لم أقرأ المقال إلا منذ سنوات قليلة وأشعر أنهم أيضًا لم يقرأوا ما كنت أقوله بعناية..

    اذا كيف تحولت؟

  45. إسرائيل
    1) تكتب أن النجم البعيد يجب أن يبدو أكبر من القمر. هذا ببساطة غير صحيح. التحول تقاربي. على وجه الخصوص، هذا يعني أن النقاط الثلاث التي كانت على خط مستقيم قبل التحويل ستبقى على خط مستقيم بعد ذلك.

    2) تكتب أنه لا يوجد دليل على قصر المسافة. وهذا أيضا غير صحيح. تحتوي ويكيبيديا على وصف لـ 5 حالات لديها دليل على ذلك. من الواضح أنه لا يوجد قياس "مباشر" - فليس لدينا اليوم طريقة لتسريع قياس الأجسام إلى سرعات نسبية.

    3) حسابات ماكسويل صحيحة لأنه يستخدم ثابتين يبدو أنهما مستقلان. وتبين في النظرية النسبية أنه يمكن حساب أحدهما من الآخر.

    4) كتبت أن الافتراض بأن قوانين الفيزياء هي نفسها في كل نظام بالقصور الذاتي غير صحيح بسبب إشعاع الخلفية الكونية. لا أفهم التناقض هنا. يمكنك دائمًا اختيار نظام ثالث كمرجع - كيف يؤثر ذلك على الافتراضات الأساسية للنظرية النسبية؟ لدي ساعة يد - ولهذا السبب فإن النظرية النسبية الفيروشية برمتها غير صحيحة؟ بجديه؟ 🙂

    5) أنت حقا تفشل في فهم "المفارقة" التوأم. من فضلك - حاول أن تفهم ما أصفه الآن. أنا على الأرض وأخي على القمر، وكلانا لديه ساعات منسقة. مركبة فضائية بسرعة جاما = 10 تمر بالأرض ثم القمر. لنفترض أن المسافة إلى القمر هي 10 ثواني ضوئية.
    1) تمر المركبة الفضائية بي في الزمن وأخي في الزمن t+10.
    2) ستشاهد المركبة الفضائية الوقت u عندما تعبرني، والوقت u+1 عندما تصل إلى القمر.

    ومن حيث درجة الحرارة - يوفر إشعاع الخلفية الكونية نظامًا مرجعيًا آخر. ونعلم أن سرعتنا بالنسبة لهذا الإشعاع هي 370 كيلومترًا في الثانية باتجاه برج الأسد. وستجد سفينة الفضاء أنها تتحرك بالنسبة للإشعاع الكوني بسرعة هائلة في اتجاه معين. أي أنها سوف ترى انحرافاً كبيراً إلى اللون الأزرق في اتجاه حركتها.

    6) فيما يتعلق بالتشابك. النظرية النسبية لا تحظر الحركة فوق سرعة الضوء. وتقول إن الكتلة الفعلية لا يمكن أن تتسارع فوق سرعة الضوء (لأنها تزيد إلى ما لا نهاية)، وبالطبع يجب ألا يكون هناك تناقضات (تمرير معلومات تؤثر على الماضي).

    ونعم - أعتقد أننا نفهم العلاقات اليوم أكثر مما فهمه أينشتاين. أعطى ألبينزو مثال داروين. ما المشكلة في ذلك؟ أنت تزعم أن إسرائيل تفهم أفضل من أينشتاين...

  46. إسرائيل، ما هو مجال فهمك في الفيزياء؟
    مما أرى، ربما بعض الدورات الأساسية في الفيزياء الكلاسيكية وربما واحدة في مقدمة ميكانيكا الكم.
    هل تقوم بالبحث في الموضوع؟ هل فتحت أي كتاب ليس كتابًا علميًا مشهورًا؟
    في رأيي المتواضع (بناء على مواضيع لعدة سنوات وأيضا من مواقع ليست من العلوم التي تستجيب لها أيضا)
    الجواب ليس لا لبس فيه.
    أنا مثلك مندهش من خفي الكم والنسبية، ولا أفهم لماذا يخطئ حدسي. الفرق بيننا هو أنني أفترض أن افتراضاتي خاطئة، وأتساءل لماذا افتراضاتي خاطئة. من الأسهل بكثير أن تتعلم شيئًا ما إذا افترضت أنك مخطئ، وتحاول معرفة سبب خطأك. لكن تصورك هو أن حدسك هو الصحيح وليس الدراسات والنظريات التي تطورت في القرن الماضي وما زالت تتطور حتى اليوم.

    على أية حال، أنصحك بشدة بعدم محاولة الخلط بين موضوعات مثل النسبية والكم لأنه حتى الفيزيائيون الذين يدرسون هذه المجالات ما زالوا يحاولون العثور على إجابات للأسئلة التي يطرحها الأشخاص مثلك. الفرق هو أن هؤلاء الناس يفعلون ذلك من أجل لقمة العيش. هم من ذوي الخبرة.

    ليس المقصود من الفقرة الأخيرة تهدئة حماسك وفضولك بشأن الفيزياء، وبالتأكيد ليس إهانتك بأي شكل من الأشكال. نرحب بفضولك، ولكن من أجل إشباعه، سيتعين عليك القيام بما هو أكثر قليلاً من مجرد طرح أسئلة على الأشخاص عبر الإنترنت حول موضوعات معقدة مثل النسبية والكم والتي تتطلب أساسًا عميقًا نسبيًا في الرياضيات والفيزياء الكلاسيكية. حاول البدء بالمجلد الأول من ميكانيكا بيركلي. تحتوي الفصول الأخيرة على تغطية متعمقة للنسبية الخاصة. تابع من هناك إلى المجلدين 1 و 2. ثق بي أنك لن تخيب.

    على الرغم من أنني ما زلت طالبًا في الفيزياء، إلا أنني آمل أن تصبح هذه الدرجة أكبر قليلاً في السنوات القادمة. لكن من الواضح أن الطريق إلى هناك هو من خلال طرح أسئلة حول مواضيع تتجاوز فهمي، ولكن من خلال رحلة طويلة ومطولة.

  47. إذا كنت لا تفهم، فربما لا يهم..

    تم الافراج عن الانتظار. لحسن الحظ بالنسبة لك، قام WordPress بقطع جميع الصيغ والرسوم التوضيحية والروابط بحيث لا يستغرق الأمر منك سوى ساعتين لقراءتها.

    الجبال يا قلبي

  48. إسرائيل
    ما هي التجربة التي تظهر عدم التطابق؟ اتركني خارج المقالات - اسأل مؤلفي المقال إذا كان هناك شيء غير واضح. فقط التجارب والحسابات المثيرة للاهتمام هنا.

    لا أفهم سبب الخلط بين الثابت الكوني وميوناتنا الضعيفة.

  49. إطالة الزمن في الميونات:

    "تدعي النظرية النسبية الخاصة أن ساعة مراقب يتحرك بسرعة عالية تتقدم بمعدل منخفض نسبيًا مقارنة بمراقب في حالة الراحة. وتسمى هذه الظاهرة تمدد الزمن النسبي، ويمكن إثباتها بمساعدة جسيمات تسمى الأيونات.

    "لقد قدم ألبرت أينشتاين الثابت الكوني بعد وقت قصير من صياغة النظرية النسبية العامة من أجل تمكين حلها الذي يصف الكون الساكن."

    مما يدل على أنه تم تقديم تفسير خاطئ لتبرير نظرية موجودة ولا يعني أن النظرية خاطئة. هذه هي النقطة - وأنا لا أقول أن Aka قد لا يكون التفسير الصحيح لإطالة الوقت، لكنني لست بحاجة إلى شرحه. وظيفتي هي فقط الإشارة إلى عدم اتساق النسبية مع الكميات.

    وهذا يذكرني - أنك لم تجب بعد على السؤال:

    "لذلك ربما تشرح لي، أنت، المفسر الوطني، كيف يتم نشر مقال يقول بالضبط ما أقوله في مجلة Scientific؟" صدفة؟ حالة مخفضة؟ سحر؟"

    وأما الغلاف الجوي فإن خط الاستواء يدور أسرع من القطبين. هل الهواء الموجود في الطبقات العليا من الغلاف الجوي يدور أيضًا بشكل أسرع من الهواء الموجود في الطبقات العليا فوق القطبين؟

    مساء الخير.

  50. إسرائيل
    لا - هذا هو التفسير من المراقب الأرضي. فيما يتعلق بالميون - التفسير هو تقلص المسافة. بالنسبة له - عمره الافتراضي 2 ميكرون، وسمك الغلاف الجوي 600 متر (وليس 6000).

    إلى ماذا ينتمي الثابت الكوني؟ ولا علاقة لها بالنسبية الخاصة.

    في الأساس، الغلاف الجوي يدور مع الأرض. عندما تطلق صاروخاً إلى الفضاء، ترى دخانه يتصاعد عالياً جداً. لقد رأيت عدة عمليات إطلاق في فلوريدا وكاليفورنيا، وظل الدخان في جميعها كما هو تقريبًا.
    فقط لا تنس أنه على ارتفاعات عالية، لا يوجد الكثير من الغلاف الجوي.

    بالإضافة إلى ذلك - يؤثر الهواء الموجود على الارتفاع على مدارات الأقمار الصناعية المنخفضة. يبدو أنه عند ارتفاع الأقمار الصناعية يتحرك الغلاف الجوي بسرعة كبيرة بسبب الارتفاع، ولكن... تتحرك الأقمار الصناعية بشكل أسرع بكثير. فكر في الحزن. ويتواجد التلسكوب على ارتفاع 600 كيلومتر، مما يعني أن الغلاف الجوي يتحرك بنسبة 10% أكثر من الارتفاع المنخفض. ومن ناحية أخرى، يدور هابل حول الأرض خلال ساعة ونصف.

  51. المعجزات

    تفسير تباطؤ الزمن هو أيضًا تفسير لطول عمر الميونات وهو أيضًا متطرف في العلاقات.

    استخدم أينشتاين الثابت الكوني لشرح التناقض مع نموذجه، واشتكى لاحقًا من أنه كان أكبر خطأ في حياته. هل هذا يعني أن التفسير غير موجود؟ (الانفجار العظيم).

    وبما أنه تم ذكر الأيونات والغلاف الجوي: فهل يعرف أحد ما إذا كان الغلاف الخارجي (الجزء الخارجي من الغلاف الجوي) يدور مع الأرض؟ هل تدور الغازات الموجودة في الغلاف الخارجي فوق القارة القطبية الجنوبية بشكل أبطأ من حيث الميونات من تلك الموجودة فوق خط الاستواء؟

  52. إسرائيل
    تفسيرك ليس أفضل من "لماذا - هكذا". بالنسبة لي، هو أقل جودة بكثير. فهو لا يفسر سبب انحناء الضوء بالقرب من الكتلة، ولا يفسر سبب زيادة الكتلة مع السرعة، ولا يفسر سبب تأثير الجاذبية على الزمن.

    كما أنه لا يشرح سبب تمكن الميونات من اختراق الغلاف الجوي. ماذا تقصد بأن الوقت يتباطأ؟ لماذا لا يفسد الحليب بسرعة؟؟

  53. "الأرض في حالة حركة بالنسبة إلى إشعاع الخلفية"

    ضئيلة تتحرك الميونات بسرعة c تقريبًا، بينما تتحرك الشمس بسرعة بضع مئات من الكيلومترات/السنة بالنسبة للإشعاع.

    "ما هو إشعاع الخلفية؟ كم عدد الفوتونات لكل سم3؟ ماذا عن الغلاف الجوي نفسه؟

    عَرَضِيّ. لقد طلبت تفسيراً بديلاً - لقد حصلت عليه. هل تريد الآن أسبوعين آخرين من التفسيرات غير ذات الصلة لمناقشة التناقض المفترض بين النسبية والكم؟

    "المشكلة الأكبر هي أنها لا تفسر أي شيء." ما هو إشعاع الخلفية؟ كم عدد الفوتونات لكل سم3؟ وماذا عن الجو نفسه؟
    ولماذا يصبح الوقت أطول بسبب التوتر؟

    لم أفهم مدى أهميتها. نحن نحاول أن نرى ما هو التناقض المفترض بين النسبية والكم. أنت تسأل عن المشاهدات، وأنا أحاول أن أوضح لك أنه يمكن أن يكون هناك تفسير بديل وأشير إلى أنه ليس بالضرورة صحيحا، فلماذا يجب أن أخوض في التفاصيل؟ وما الذي أحاول بيعه بالضبط؟ حبوب ذرة؟

    يشير أولبرز إلى مفارقة أولبرز لإظهار أن ظلام سماء الليل يتناقض مع افتراض وجود كون ثابت لا نهائي وأبدي. فشل في التوصل إلى تفسير (التفسير الصحيح: توسع الكون). فهل عدم إحضاره بالتفسير يعني عدم وجود المفارقة؟

    أحاول أن أبين أن النسبية لا تتوافق مع الكميات (وبعض الأشياء الأخرى، في انتظار أن يفرج والدي عن الانتظار)، فهل هذا يعني أن لدي تفسيرًا بديلاً لكل ظاهرة؟

    لذا ربما تشرح لي، أيها المفسر الوطني، كيف يتم نشر مقال يقول بالضبط ما أقوله في مجلة Scientific؟ صدفة؟ حالة مخفضة؟ سحر؟

  54. إسرائيل
    في رأيي نعم. لعدة أسباب.
    إن الأرض في حالة حركة بالنسبة إلى إشعاع الخلفية، لذا كان من المفترض أن نرى اعتماداً على الاتجاه. تتحرك الأرض بسرعة عدة مئات من الكيلومترات في الثانية، وهو أمر لا يستهان به. ولذلك، أتوقع الاعتماد على امتصاص الميونات والوقت.

    المشكلة الأكبر هي أنه لا يفسر شيئا. ما هو إشعاع الخلفية؟ كم عدد الفوتونات لكل سم3؟ وماذا عن الجو نفسه؟
    ولماذا يصبح الوقت أطول بسبب التوتر؟ لأنه يناسب ما تبيعه؟ 🙂

  55. ما هو سيء

    وهنا تكمن مشكلة الميونات كما أفهمها: عمرها القصير لا يسمح لها بقطع المسافة من مكان تكوينها إلى الأرض قبل أن تتفكك، ومع ذلك فإنها تتمكن من الوصول إلى الأرض. وكيف يتم ذلك دون إطالة الأزمنة أو تقصير الأزمنة. أنا أفهم ذلك أليس كذلك؟

    وهذا هو التفسير الذي قدمته لك: بسبب التسارع والحركة مقابل إشعاع الخلفية، تكون الساعة الداخلية للميون في حلقة وبالتالي تتباطأ، تمامًا كما تتباطأ عمليات التحلل بالتبريد وبالتالي العمر الافتراضي التفاحة في الثلاجة أطول من التفاحة في الثلاجة. تفسير ليس بالضرورة صحيحا، ولكن سيئة؟

  56. إسرائيل
    لقد حصلت على تفسير سيء للغاية. يتم التقاط Muyons في كل من لوس أنجلوس وسيدني. كيف يعمل

    ما تقوله النظرية النسبية يظهر أيضًا في التجربة: إطالة الزمن، تقصير المسافة، زيادة الكتلة، انحناء الضوء، موجات الجاذبية، امتصاص الميونات من الإشعاع الكوني، تنافر الجاذبية وما إلى ذلك.

  57. المعجزات

    "النسبية بسيطة، خاصة وعامة. نظرية الكم معقدة للغاية. لماذا تفضل المعقد؟

    لقد ذهبنا إلى هناك من قبل، أتذكرين؟ 1935، إي بي آر؟

    لأن هذا ما يظهر في التجربة.

  58. المعجزات

    عكا - غير متظلم. الفيزياء - وليس علم النفس.

    الميون لا يرى شيئًا، وله نظام مرجعي مفضل. لقد طلبت تفسيراً - لقد حصلت عليه.

    ألبانزو

    فماذا تقول بحسب المادة النسبية الأصلية وحسبها فقط – هل التناقض مع الكمات التي أشرت إليها موجود أم لا؟ تذكر أنه لا يمكنك التحدث عن تقديرات تقريبية لمبدأ عدم اليقين لأن تحويلات لورنتز المستمدة من النسبية ليست تقريبية.

    علاوة على ذلك، إذا كنت تريد التحدث كمعلم إلى أحد الطلاب، فسيتعين عليك الإجابة على بعض الأسئلة قبل أن تحصل على السلطة.

  59. حسنا، ماذا سيحدث؟

    https://www.hayadan.org.il/researchers-discover-the-most-distant-supermassive-black-hole-07121704/comment-page-6/#comment-723358

    خصوصا الفقرة الأخيرة.

    في دورة هادفا، أحد الأشياء الأولى التي تتعلمها (الأول عادةً) هو تعريف الحدود. يتم تعريف الاستمرارية بمساعدة النهاية، ومن هناك إلى المشتقات. لكن إذا حاولت أن تتعلم الهدوة فقط من كتابات نيوتن الأصلية، فستجد أن مفهوم الحد غير موجود هناك على الإطلاق. لقد أجرى نيوتن كل البراهين بشكل هندسي وتعجب وتعجب - وأغلبها خطأ. الاستنتاجات صحيحة، والمسار ليس كذلك، وغالباً ما يحتوي على تناقض داخلي. وبعد مرور 150 عامًا فقط، اكتشف نيوتن طرقًا لإثبات جميع مبادئ ونظريات الهدوة بدقة. فماذا تقول - هل هناك مشكلة في هادفا أم لا؟ إذا اقتصرت على الكتابات الأصلية فقط (بعضها موجود في كتاب المبادئ، وبعضها نُشر بشكل منفصل)، فإنني أجد جملًا تحتوي على أدلة كاذبة. ماذا يعني هذا بالنسبة لهدوة؟

    لا يمكنك أن تتعلم النسبية الخاصة من إحدى أوراق أينشتاين فقط، مثلما لا يمكنك أن تتعلم التطور من أصل الأنواع فقط. ليست هناك حاجة لمعرفة مسار الفوتون لصياغة النظرية النسبية الخاصة. حقيقة أن أينشتاين فعل ذلك (لأنه وسع الميكانيكا الكلاسيكية وفي الميكانيكا الكلاسيكية لا يوجد سبب للاعتقاد بأننا لا نعرف مسار الفوتون) لا تعني أن هناك مشكلة في العلاقات الخاصة. بل يعني أن هناك مشكلة مع الطالب الذي يظن أنه يستطيع أن يتعلم التوراة كاملة من مادة واحدة، وعندما يوضح له أنه لا يستطيع، يتجاهل ذلك.

  60. إسرائيل
    إذا كانت ساعة الميون أبطأ (بالنسبة له) فسوف يرى الأرض تقترب منه بسرعة أعلى بكثير من سرعة c.

    إن تغير الوقت في ساعات GPS هو نتيجة لعدة عوامل. أحدها، وهو العامل الرئيسي، هو قلة الجاذبية على ارتفاع القمر الصناعي. لا توجد مشكلة السرعة هنا. العنصر الثاني في تغير الزمن هو السرعة النسبية، ونقيس أيضًا أن التباطؤ متماثل، مما يعني عدم وجود نظام مرجعي مفضل.
    إنه لا يتطابق مع الضيق الذي تصفه.

  61. في هذه الأثناء، دعونا نذهب.

    تتعرض الميونات للإجهاد - إجهاد الأيائل - الذي ينتج عن التسارع إلى السرعة العالية والحركة ضد إشعاع الخلفية. يؤدي الانكماش إلى إبطاء الساعة الداخلية للميون.

    وهذا أيضًا هو السبب وراء إبطاء ساعات أي نظام متسارع، مثل الأقمار الصناعية لنظام تحديد المواقع العالمي (GPS).

    وهذا تفسير محتمل، ولكنه ليس ضروريا. الحل الأمثل هو دمج النسبية كحالة خاصة في نظرية تتضمن اللامكانية، فنظرية نيوتن هي حالة خاصة للنسبية عند السرعات المنخفضة ونظرية ماكسويل هي اختزال النسبية.

  62. المعجزات

    لماذا تفعل هذا بنفسك؟

    مرحبًا بالجميع وأشكركم على حضوركم العرض الذي قدمته بعنوان "مشكلة النسبية".

    ربما تنتظر مني أن أكمل جملة "هل كان أينشتاين مخطئا؟" هذا مثل قول "هل كان نيوتن مخطئًا" أو "هل كان ماكسويل مخطئًا؟" في نهاية العرض ستدرك، أعتقد، أنه إذا كان أينشتاين مخطئًا على الإطلاق، فقد كان خطأً معقولًا جدًا ارتكبه.

    ويجب أن أضيف أيضًا أنه من المحتمل أنه لم يكن مخطئًا على الإطلاق، باستثناء ربما فيما يتعلق بموضوع سوء التصرف، وأنا ببساطة لا أفهم القضية جيدًا بما فيه الكفاية. أحد الأسباب التي دفعتني إلى تقديم هذا العرض اليوم هو أنه ربما يقوم أحدكم بتوضيح القضايا التي سأثيرها اليوم حتى أفهمها بشكل أفضل. لقد أثرت كل هذه المواضيع في العديد من المنتديات وكنت على اتصال بالعديد من الخبراء، بما في ذلك أساتذة الفيزياء، لكن لم تنهي أي من إجاباتهم تساؤلاتي. كما أنني أستثمر قدرًا كبيرًا من الوقت - وأيضًا المال - لمحاولة العثور على إجابة لتلك المحاجر من خلال البحث وجميع أنواع التجارب.

    أود أن أقسم العرض إلى 4 أجزاء:

    1. الطريقة التي كانت تبدو بها الفيزياء حتى نهاية القرن التاسع عشر.

    2. شرح سبب توصل أينشتاين إلى النسبية الخاصة بما في ذلك شرح مختصر عن ماهية النسبية ولماذا لدينا تمدد الزمن حسب النسبية.

    3. المشاكل التي أراها مع النسبية.

    4. مناقشة مفتوحة للمجموعة.

    الموضوع واسع، والوقت قصير، لذا بطبيعة الحال لن نتمكن من التوسع كثيرًا في كل جزء. على أية حال، ما زلت أود أن أروي بالترتيب الزمني تاريخ العلماء المشاركين في الطريق إلى النسبية وآمل أن تستمتعوا بالقصة وكذلك الفيزياء..

    لذلك دعونا نبدأ بالجزء الأول - تاريخ موجز للنسبية.

    سأبدأ بوصف للشكل الذي كانت تبدو عليه الفيزياء قبل سنة أينشتاين المعجزة 1905.

    أعطى قانون نيوتن للميكانيكا وصفًا ممتازًا للطريقة التي تتفاعل بها الأشياء المادية مع القوى، بما في ذلك جميع الصياغة الرياضية. وفي نهاية القرن الثامن عشر وبداية القرن التاسع عشر، بدأت قوانين البصريات والكهرباء والمغناطيسية في الظهور. أظهر مايكل فاراداي أن الكهرباء تؤثر على المغناطيسية والعكس صحيح. بالمناسبة، لم يحصل فاراداي على أي تعليم رياضي، ولم يكن يعرف الجبر إلا بالكاد، لكنه كان مجربًا عظيمًا، أظهر العلاقة بين الكهرباء والمغناطيسية.

    العام 1861 - الحرب الأهلية تدور رحاها في أمريكا - وفي اسكتلندا ينشر عالم فيزياء اسمه جيمس كليرك ماكسويل بحثًا بعنوان: "على خط القوة الفيزيائية".

    ربما سمع الكثير منكم عن ماكسويل، دعنا نقول فقط أنه إلى جانب نيوتن وأينشتاين، يعتبر ماكسويل أحد أعظم علماء الفيزياء في كل العصور.

    حاول ماكسويل صياغة معادلات مايكل فاراداي للكهرومغناطيسية رياضيًا وجسديًا. دعاه:

    نظرية الدوامات الجزيئية المطبقة على الظواهر المغناطيسية.
    ربما يكون معظمكم على دراية بالظاهرة التي تقول إنه إذا قمت بنشر غبار الحديد (أو الحشوات) بجوار المغناطيس، فيمكنك رؤية الغبار يرتب نفسه على خطوط المجال المغناطيسي. وأيضًا، إذا قمنا بتحريك مغناطيس بجانب سلك نحاس، فإنه يولد تيارًا كهربائيًا في السلك، وإذا مررت تيارًا في هذا السلك، فيمكنه تحريك إبرة البوصلة.

    قام ماكسويل، الخبير في موضوع الديناميكا المائية، ببناء نموذج هيدروديناميكي يصف ويشرح كيفية حدوث تلك الظواهر في ما يسمى بالأثير. ووفقا لهذا النموذج، فإن الأثير يشبه إلى حد ما البحر، وفي هذا البحر هناك تيارات، وتموجات، وضغوط، ودوامات، وكما هو الحال في أي بحر، هناك أيضا أمواج.

    وفي نموذجه، أظهر ماكسويل أن خط القوى هذا الذي نراه عندما ننشر غبار الحديد بجوار المغناطيس، هو في الواقع تيارات تتدفق من القطب الشمالي للمغناطيس إلى القطب الجنوبي، وتولد تيارًا آخر في " البحر"، وهو متعامد مع التيار المغناطيسي ويمكن اعتباره تيارًا كهربائيًا. وهذا التيار الكهربائي بدوره يخلق تيارًا مغناطيسيًا بزاوية 90 درجة، والذي بدوره يولد تيارًا كهربائيًا مرة أخرى، وهكذا، حتى نهاية الزمان.

    كل هذه اللعبة العكسية والقوة عبارة عن مجال كهربائي يتحول إلى مجال مغناطيسي وبالعكس ينتشر في البحر كموجة عرضية، وهي موجة أطلق عليها ماكسويل اسم الموجة الكهرومغناطيسية.

    ثم ذهب ماكسويل لحساب سرعة هذه الموجة.

    استخدم صيغة نيوتن للموجات الصوتية في الهواء لسرعة الموجة: الكثافة ρ تحدد سرعة الصوت (موجات الضغط)، وفقًا لصيغة نيوتن-لابلاس:
    ج=\sqrt {\frac {K} {\ rho}}.
    وبينت أنه إذا استبدلنا K في صيغة (معامل المرونة) وصف الحروف اليونانية (الكثافة) بثوابت المعاملات المعروفة للكهرباء والمغناطيسية، فسنحصل على سرعة انتشار الموجة في هذا الوسط "الأثيري".

    وعندما تحقق لمعرفة قيمة هذه السرعة، وجد لدهشته أن هذه هي سرعة الضوء كما قاسها أرماند فيزو عام 1849 (انظر المعادلة 136 في نموذج ماكسويل للأثير):

    ثم استنتج ماكسويل أن الضوء هو أيضًا موجة كهرومغناطيسية.
    أخبرني الناس في الماضي أن ماكسويل كان يعلم بالفعل أنه سيحصل على سرعة الضوء وأنه فقط "رسم الهدف حول نقطة إصابة السهم"، لكن ماكسويل نفسه نفى ذلك.
    وبعد 4 سنوات، استخدم ماكسويل نفس نموذج الأثير للوصول إلى معادلاته التفاضلية الشهيرة والتي يمكن رؤيتها على القمصان التي يرتديها الطلاب.

    توفي ماكسويل عام 1879 وهو شاب عن عمر يناهز 48 عامًا.

    وقد أشاد العلماء في عصره بنموذج ماكسويل للكهرومغناطيسية، ولكن لم تكن هناك تجارب يمكن أن تدعم نظريته. في عام 1879، وهو العام الذي توفي فيه، كانت الشوارع مظلمة في الغالب أثناء الليل (اخترع إديسون المصباح الكهربائي في هذا العام) وكانت معظم وسائل النقل تتم بواسطة الخيول. فهل تتوقع أن يصدق الناس بعض الموجات الغامضة، غير المرئية والتي لم تمسها، والتي تتحرك بسرعة الضوء؟ ما هي الفكرة الأخرى التي سيطرحها هؤلاء الفيزيائيون المجانين، ربما أن الناس سيكونون قادرين على قيادة سياراتهم في أمريكا واستخدام الموجات الكهرومغناطيسية للتحدث مباشرة مع أصدقائهم في الهند ورؤيتهم أيضًا؟ المجانين الحقيقيين..

    قدم هيرمان فون هيلمهولتز، رئيس قسم الفيزياء في برلين، جائزة برلين للشخص الذي سيثبت تنبؤات ماكسويل بشكل تجريبي. مرت السنوات ولم يبدو أن أحدًا نجح في المهمة. كان هلمهولتز على وشك الاستسلام، لكن بعد ذلك، نجح تلميذه اللامع، هاينريش رودولف هيرتز، عام 1886 في إرسال واستقبال ما سيعرف بموجات الراديو، وأيضا التأكد من أنها تتحرك بسرعة الضوء، وهي مستقطبة ومستقطبة. بقية تنبؤات ماكسويل.

    وعن عرض هيرتز للموجات الكهربائية، قال هيلمهولتز للجمعية الفيزيائية في برلين: "أيها السادة! يجب أن أنقل لكم اليوم أهم اكتشاف فيزيائي في القرن التاسع عشر.

    توفي هيرتز عام 1894 وهو صغير جدًا، وكان عمره 36 عامًا.

    إذن، نحن هنا، تقريبًا في نهاية القرن التاسع عشر، ويعتقد الكثيرون أن الفيزياء قد اكتملت تقريبًا وأن الشيء الوحيد المتبقي هو تحسين القياسات للحصول على دقة أفضل. تم اختزال الديناميكا الحرارية بقانونيها القويين، الأول والثاني، إلى فرع من ميكانيكا الحركة. أعطانا الإنجليزي نيوتن قوانين الميكانيكا والجاذبية، وأعطانا الاسكتلندي ماكسويل قوانين الكهرومغناطيسية، وكل ما يتعين علينا فعله الآن هو الاستمتاع بالعالم المريح الذي رتبه لنا هذان السادة البريطانيان ونشرب كوبًا من الشاي، مثل نحن نفعل في وندسور.
    يجب أن أعترف أنني أفتقد عصر البراءة هذا، عندما حكمت الفيزياء الكلاسيكية، هنا كان هنا، وكان هناك، والحاضر لم يكن ماضيا، وكان لدينا كون واحد. هل يمكنك مقارنتها بالفوضى التي نعيشها اليوم، مع الأكوان المتعددة، حيث تقفز الإلكترونات من جانب واحد من الكون إلى الجانب الآخر في وقت قصير ودون حتى أن تكون في المنتصف، مع وجود قياسات في الحاضر تؤثر على قياسات في الماضي؟

    في زمن جدي، كانت الموجة موجة، وكان الجسيم جسيمًا.

    لا يزال هناك سؤال لم يتم حله: إذا كانت الموجات الكهرومغناطيسية تنتشر في "بحر" الأثير، كما افترض ماكسويل وأظهر هيرتز تجريبيًا، فما هو الإطار الباقي لهذا البحر؟ كل وسط هيدروديناميكي لديه إطار راحة. المحيطات - مثل الأرض. الهواء - الأرض، أو إذا كانت هناك ريح، الريح. الهواء في الطائرة، الطائرة.
    أريد فقط أن أضيف أن الإيمان بالأثير في نهاية القرن التاسع عشر كان مثل الإيمان بالذرات اليوم: لا جدال فيه تقريبًا. هذا صحيح، لم يرى أحد أو يشعر بالأثير، لكن لم يرى أو يشعر بذرة أو إلكترون في عصرنا هذا أيضًا.

    إذن ما هو الإطار الباقي للأثير؟

    وفي عام 1887 أجرى ميشيلسون ومورلي تجربة للكشف عن "رياح الأثير"، ومنها لاستنتاج الإطار الباقي للأثير بالنسبة للأرض. استخدموا أداة تسمى مقياس التداخل، استخدموها عدة مرات في فصول مختلفة من العام، ولكن مما أثار استياءهم أنهم لم يجدوا إطارًا للراحة للأثير. وكان هذا الأمر محيرًا لهم ولأغلب المجتمع العلمي، إذ كما قلنا، كل الأجسام تقريبًا تؤمن بالأثير.
    لكن "فشل" تجربة ميكلسون-مورلي - لقد رأوا فيها فشلًا، وربما مشكلة فنية - وقاموا بذلك مرارًا وتكرارًا، جنبًا إلى جنب مع مشكلة إشعاع الجسم الأسود، أدى إلى التطورات الأكثر إبداعًا وإبهارًا في مجال العلوم. الفيزياء – النظرية النسبية وميكانيكا الكم.

    أدت النتيجة الفارغة لتجربة ميشيلسون-مورلي في عام 1887، إلى جانب اعتبارات أخرى، إلى قيام ألبرت أينشتاين بنشر بحثه في عام 1905 بعنوان: "حول الديناميكا الكهربائية للأجسام المتحركة".

    حيث قدم النسبية الخاصة للعالم.

    بهذا نختتم الجزء الأول من العرض التقديمي، فيزياء القرن التاسع عشر. إذا كان لدى أي شخص أي أسئلة أو تعليقات، فهذا هو الوقت المناسب قبل أن ننتقل إلى الجزء الثاني.

    الجزء الثاني: النسبية في قشرة الجوز.

    لقد وعدت جوليان بعدم المبالغة في تعقيد العرض التقديمي أكثر من اللازم، لكنني أشعر أنه يجب علي أن أتناول القليل من التفاصيل حول هذا الموضوع.

    أبحر حاييم فايتسمان، رئيس إسرائيل الأول، ذات مرة مع أينشتاين وشرح له أينشتاين موضوع النسبية العامة. وفي نهاية الرحلة، سأل المراسلون في الميناء وايزمان: "هل تفهم الآن النسبية؟" أجاب وايزمان: "لا، ولكن لدي انطباع بأن أينشتاين يفعل ذلك".
    النسبية الخاصة في الواقع ليست بهذا التعقيد، ولهذا السبب أود أن أبقى في الموضوع لبعض الوقت، وإذا شعر شخص ما أنه يفهمها بشكل أفضل، فقد قمت بدوري. لكن جوليان، إذا كنت تعتقد أننا نتناول المزيد من التفاصيل هنا، فلا تتردد في قطع كلامي حتى نتمكن من المضي قدمًا.

    استخدم أينشتاين في نظريته مسلمتين:
    قوانين الفيزياء هي نفسها بالنسبة لجميع المراقبين الذين يتحركون بشكل موحد بالنسبة لبعضهم البعض (مبدأ النسبية).
    سرعة الضوء في الفراغ هي نفسها لجميع المراقبين، بغض النظر عن حركتهم النسبية أو حركة مصدر الضوء.
    أظهر أينشتاين أن مشكلة الإطار الساكن للأثير التي فشل ميشيلسون ومورلي في العثور عليها، يتم حلها إذا تجاهلنا افتراض الأثير واستبدلنا فكرة المكان والزمان الثابتين في الميكانيكا النيوتونية بمعلمات نسبية بينما الشيء الثابت الوحيد هو c ، سرعة الضوء.
    لذلك دعونا نرى ماذا يعني هذا:

    الفرضية الأولى:
    قوانين الفيزياء هي نفسها بالنسبة لجميع المراقبين الذين يتحركون بشكل موحد بالنسبة لبعضهم البعض (مبدأ النسبية).

    وهذا المبدأ معروف منذ زمن جاليليو، أي قبل 300 عام. ما يقوله هو أنه إذا كان لديك جسمان يتحركان بالنسبة لبعضهما البعض ولم يتسارعا، فلا توجد طريقة لمعرفة أي منهما في حالة سكون "حقًا" وأيهما يتحرك "حقًا". ونتيجة لذلك، لا توجد تجربة يمكن أن تخبر الجسم بالقصور الذاتي إذا كان في حالة سكون أو حركة.

    الفرضية الثانية:
    سرعة الضوء في الفراغ هي نفسها لجميع المراقبين، بغض النظر عن حركتهم النسبية أو حركة مصدر الضوء.
    لفهم هذه العبارة، دعونا نفكر في قطار طويل يقف على خط السكة الحديد. إذا أطلقنا رصاصة من مسدس على طول القطار وكانت الرصاصة تتحرك بسرعة 1000 م/ث، فإن حركة الرصاصة تكون 1000 م/ث بالنسبة إلى البندقية التي أطلقتها، ويكون صوت الطلقة 330 م /s نسبة إلى الهواء، الوسط الذي يحمل الصوت. تلك هي أيضًا سرعة الرصاصة والصوت بالنسبة للقطار.
    إذا كان القطار سيبدأ التحرك بعد الطلقة في اتجاه الطلقة، فستتحرك الرصاصة الآن بشكل أبطأ بالنسبة للقطار لأن القطار يتحرك بالنسبة إلى الموقع الأصلي للبندقية، وكذلك يتحرك الصوت بشكل أبطأ بالنسبة للقطار لأن القطار يتحرك بالنسبة للهواء، وهو الوسط الذي يحمل الصوت.
    فإذا تحركنا بسرعة الرصاصة يمكننا الإمساك بها بيدنا. إذا تحركنا بسرعة الصوت، فسنحدث دويًا أسرع من الصوت. يمكننا أن نتحرك بشكل أسرع من كليهما، الرصاصة والصوت، ونتركهما خلفنا.

    ونحن نسمي سرعة الرصاصة سرعة الانبعاث، لأنها نسبة إلى المصدر الذي أطلقها.
    ولكن ماذا عن سرعة الضوء، الوميض الناتج عن اللقطة؟ هل هو نسبة إلى البندقية التي أطلقتها؟

    لقد فحص أينشتاين ما يسمى "نظرية الانبعاث"، لكنه أدرك أنها لا يمكن أن تكون حقيقية في حالة الضوء. وكما رأينا فإن النتيجة الصفرية لتجربة ميكلسون-مورلي عام 1887 أظهرت أن سرعة الضوء لا تتناسب مع الوسط الأثيري.
    وتخيل أينشتاين أيضًا ما سيحدث إذا تحرك بسرعة الضوء. فهل يكون النور ساكنا بالنسبة له؟ هل سيكون قادرًا على الوصول إلى يده والتقاط بعض الضوء كما لو كان بإمكانك التقاط رصاصة إذا حلقت فوقها بنفس سرعة الرصاصة؟
    أدرك أينشتاين أن الإجابة يجب أن تكون سلبية. لا يهم مدى السرعة التي تتحرك بها، فالضوء يتحرك بعيدًا عنك بنفس السرعة ج، سرعة الضوء. هذه هي المسلمة الثانية للنسبية.

    موضوع الوقت والتزامن معقد بعض الشيء ولن أتطرق إليه إلا إذا كان هناك طلب من المجموعة. في الأساس، كيف نعرف أن الساعتين تتحركان بنفس المعدل إذا كانتا في مكانين مختلفين؟ لماذا هم؟ يجب أن أخبرك أنه منذ أن بدأت في تعلم المادة، أتحقق من وقت لآخر من الساعة في الغرفة الأخرى للتأكد من أنها تظهر نفس الوقت الموجود في غرفتي.
    وعلى أية حال، فإن الفصل الأول من أينشتاين في ورقته النسبية مخصص لموضوع التزامن - فهو يقترح طريقة للتأكد من مزامنة أي عدد من الساعات في إطار القصور الذاتي، أي إظهار نفس الوقت دائمًا بغض النظر عن الموقع في النظام. .

    لذلك دعونا نعود إلى قطارنا، ولنفترض أن هناك ساعات متزامنة في القطار.

    القطار لا يتحرك، لذلك فهو نظام بالقصور الذاتي. من أجل التبسيط، لدينا السكك الحديدية في الفضاء، لنفترض أن السكك الحديدية هي السكك الحديدية المجرية التي بناها النباتيون وكلفت حياة العديد من العبيد الأندروميدين سيئي الحظ. وله مساران متوازيان ويبلغ طوله 100 سنة ضوئية. السكك الحديدية، كما تعلمون، مثل تلك الموجودة في لوس أنجلوس

    يبلغ طول قطارنا 300,000 ألف كيلومتر، والسيارات لا تحجب الضوء، وطول كل سيارة كيلومتر واحد. وفي النهاية هناك مرآة. لذا، إذا وجهنا ضوءًا أخضر وامضًا على أحد جوانب القطار، فسيستغرق الضوء ثانية واحدة بالضبط للانتقال إلى الجانب الآخر والاصطدام بالمرآة ثم العودة للخلف، وثانية أخرى للوصول إلى النقطة أصل.

    لاحظ أنه إذا كانت لدينا مرآة على الجانب الأول أيضًا، فسنحصل على ساعة، نظرًا لأننا نحصل على "علامة" عندما يضرب الفوتون جانبًا واحدًا و"علامة" على الجانب الآخر، ففي كل ثانية هناك علامة تك، فقط مثل الساعة.
    فإذا كانت الساعات في القطار متزامنة كما اقترح أينشتاين، ففي الوقت 0 سيغادر الضوء أحد جوانب القطار، وسيصل إلى الجانب الآخر في زمن 1 ثانية، وسيصل إلى الجانب الأول في زمن 2 ثانية. الجميع موافق؟
    إذا تحرك القطار الآن على السكة الحديدية، فسنحصل على نفس النتائج لأي سرعة للقطار. تذكر أن القطار في إطاره الخاص يكون دائمًا ثابتًا والمسارات تتحرك.
    لنفترض الآن أنه على المسار الموازي يقف قطار مماثل، القطار رقم 2 ذو الضوء الأحمر. وبطبيعة الحال سيحدث نفس الشيء للتدريب 2.

    الآن ماذا يحدث إذا كان أحد القطارين يتحرك بالنسبة إلى الآخر، وفي الوقت 0 يتزامن كلا القطارين.
    في الوقت 0 في كلتا الساعتين يترك الضوء نهاية القطارات، فلنقل باللون الأخضر. لدينا كاميرا عالية الدقة وتظهر: الساعة في القطار 1: 0، والساعة في القطار 2: 0. وبما أن كلا القطارين عبارة عن نظامين بالقصور الذاتي، فسوف يصلان إلى الجانب الآخر في زمن 1 ثانية كما في السابق، وفي الوقت 2 ثواني من ناحية أخرى. تذكر أن كل قطار هو نظام بحد ذاته، وحركة الآخر ليس لها أي تأثير عليه. كلاهما واقف، والقطار الآخر، أو السكة الحديد نفسها، يتحرك بالنسبة لهما.
    ولكن عندما يصل الضوء إلى الجانب الأول من القطار 2، يكون القطار 2 قد تقدم بالفعل على طول القطار 1، لنفترض 1000 كيلومتر، أو 1000 عربة قطار. الزمن في ساعة القطار 2 هو ثانيتان، لكن ما الزمن في الساعة المجاورة للقطار 2 في السيارة 1؟
    لا يمكن أن تكون ثانيتين لأن الضوء يصل إلى نهاية القطار واحدًا تلو الآخر بعد ثانيتين، ولا يزال أمامه 2 كيلومتر ليقطعها.. ولهذا السبب فإن الوقت في الساعة المجاورة أقل من ثانيتين. إذا التقطنا صورة - لقطة سريعة - بكاميرا عالية الدقة للساعتين معًا في هذه النقطة، من أي قطار، ستظهر الصورة: الساعة من القطار 2، ثانيتان، الساعة من القطار 1000 أقل من 2 ثانية.
    وكما نتذكر، فقد بدأ كلاهما في الوقت 0 في كل قطار، ولدينا أيضًا صور تثبت ذلك.

    ولهذا السبب لدينا تمدد زمني لجسم متحرك في نظام متزامن. يمكننا الاستمرار وإظهار سبب انكماش الطول، ولكن دعونا نتخطى ذلك الآن.

    لاحظ أن تمدد الزمن يحدث أيضًا إذا لم نستخدم الضوء على الإطلاق، فقد استخدمناه فقط لتوضيح هذه النقطة.
    ثم ذهب أينشتاين ليثبت أنه لا يوجد شيء يمكنه التحرك بشكل أسرع من الضوء. سأوفر لك الدليل، وأقول فقط هذا: أي سرعة تحقق المسلمة الأولى، أي هي نفسها لجميع المراقبين، بغض النظر عن حركتهم النسبية أو حركة مصدرها، فهي الحد الأعلى لجميع السرعات، ولا لا يهم إذا كانت سرعة الضوء أو سرعة الصوت أو سرعة كلب الصيد.

    وفي وقت لاحق من هذا العام، أظهر أينشتاين في ورقة بحثية أخرى أن الكتلة والطاقة متكافئتان ومرتبطتان بالصيغة الشهيرة الآن: E=mc^2. وبعد 10 سنوات، في عام 1915، وسع أينشتاين نظريته لتشمل الجاذبية، فيما يعرف بالنسبية العامة.

    هنا ننتهي من أول جزأين من العرض، الفيزياء قبل أينشتاين والنظرية النسبية الخاصة في قشرة الجوز. إذا كان لدى أي شخص تعليقات أو أسئلة، فهذا هو الوقت المناسب قبل أن ننتقل إلى الجزء الثالث، تحديات النسبية.

    الجزء 3: تحديات النسبية.

    1. على الرغم من وجود تناقضات رياضية داخل النظرية، إلا أن النسبية الفيزيائية غريبة جدًا لدرجة أن العديد من الفيزيائيين وجدوا صعوبة بالغة في قبولها. لا يزال علماء الفيزياء النظرية مثل أبراهام ولورنتز وبوانكاريه ولانجفين يؤمنون بوجود الأثير.
    على سبيل المثال، يصعب فهم انكماش طول لورينز بشكل خاص وليس له دعم تجريبي مباشر؛ http://math.ucr.edu/home/baez/physics/Relativity/SR/experiments.html#Length_Contraction
    اختبارات انكماش الطول
    في هذا الوقت لا توجد اختبارات مباشرة لانكماش الطول. تخيل أنك تتحرك نحو النجم ألفا سنتوري (على بعد 4 سنوات ضوئية) بسرعة كافية بحيث يكون عامل جاما:

    يساوي 1000,000، وتلتقط صورة للنجم من الأرض. بما أن انكماش الطول يكون فقط في اتجاه حركتك وليس في الإحداثيات المتعامدة، فماذا ستظهر صورتك؟ — نجم ضخم أقرب من القمر؟

    2. التناقضات مع نظرية الكم. انظر إلى العرض الذي قدمه Sky Jason Shields، على الرغم من أنني فاتني محاضرة يونيو.
    من العرض الذي قدمه: في الأساس، يمكنك التفكير في التقسيم بين النسبية والأنظمة الكمومية على أنه "سلس" مقابل "مكتنز" أو مترابط بشكل مستمر مقابل مجزأ بشكل منفصل.

    3. إذا كان نموذج ماكسويل الأثير خاطئا - فنموذج ماكسويل معقد وحساس للغاية - فكيف تمكن من استخلاص سرعة الضوء من هذا النموذج الهيدروديناميكي الخاطئ (المعادلة 136 أعلاه) ومعادلاته الشهيرة؟ صدفة؟ تخمين محظوظ؟ سحر؟

    لدينا في إسرائيل بطل إذاعي مشهور اسمه مار نحسون، والذي يعني السيد الضيف. تمت مقابلته عبر الراديو وسئل كيف نجح في أن يكون الشخص الوحيد الذي يخمن جميع نتائج مباريات كرة القدم لمدة 4 أسابيع متتالية.

    وأوضح: "حسنًا، هذا بسيط جدًا. ما أفعله هو عد الكرات الموجودة على الأرض، وضربها في طول الجدران، وتقسيمها على أعداد الجيران. بشكل افتراضي، يجب أن يأتي مع الإجابة الصحيحة.
    ولديه نظام آخر للصيف: فهو يأخذ جميع أفراد الأسرة إلى الشاطئ عندما تكون هناك أمواج كثيرة في البحر ويسمح لهم بالدخول، ثم يشاهد: إذا خرج أحدهم، يكتب 1. إذا 2، 2. و إذا لم يخرج أحد – x.
    لذا ربما يكون نموذج ماكسويل مثل السيد جيستر، وكان محظوظًا بكل بساطة؟

    4. المسلمة الأولى للنسبية:
    قوانين الفيزياء هي نفسها بالنسبة لجميع المراقبين الذين يتحركون بشكل موحد بالنسبة لبعضهم البعض (مبدأ النسبية). وهذا لا يتوافق مع نظرية الانفجار الكبير وإشعاع الخلفية الكونية الميكروي.
    انظر إلى ذلك في مفارقة التوأم عندما يلتقي التوأم الصغير بأخيه الأكبر، يكون الكون أكثر برودة وقت لقائهما حسب معادلة فريدمان:
    http://hyperphysics.phy-astr.gsu.edu/hbase/Astro/expand.html#c3
    وهكذا، خلال رحلته التي تستغرق وقتًا أقصر من أخيه، يلاحظ انخفاضًا أسرع في درجة الحرارة من أخيه، على الرغم من أنهما في إطار القصور الذاتي.
    لاحظ أن التسارع ليس حلاً في هذه الحالة: فإذا كان التوأم الأكبر يتحرك بالنسبة للكمبر، فهو الذي لاحظ الانخفاض السريع في درجة الحرارة، وسيبقى الصغير دافئًا ومرتاحًا.

    5. في مفارقة EPR (1935):
    https://en.wikipedia.org/wiki/EPR_paradox
    يصف أينشتاين وبودولسكي وروزن تجربة فكرية لإظهار سبب عدم اكتمال نظرية ميكانيكا الكم. بحسب المقال في ويكيبيديا:
    وإليكم المفارقة ملخصة:
    إن القول بأن القياس الفيزيائي لزخم الجسيم الأول يؤثر على عدم اليقين في موضعه هو شيء، ولكن القول بأن قياس زخم الجسيم الأول يؤثر على عدم اليقين في موضع الجسيم الآخر هو شيء آخر تمامًا. تساءل أينشتاين وبودولسكي وروزن كيف يمكن للجسيم الثاني أن "يعرف" أن له زخمًا محددًا بدقة ولكن موقعه غير مؤكد؟ وبما أن هذا يعني أن أحد الجسيمين يتواصل مع الآخر بشكل فوري عبر الفضاء، أي أسرع من الضوء، فهذه هي "المفارقة".

    تم إجراء نسخة مختلفة من التجربة بواسطة آلان أسبكت في الثمانينيات، ووجد أنها متوافقة مع ميكانيكا الكم ونظرية جون ستيوارت بيل، وضد ادعاء أينشتاين.
    استندت المحاولات اللاحقة لحفظ النسبية من تجربة آسبكت إلى الادعاء بأن ما تحظره النسبية حقًا هو نقل المعلومات، وأنه من المستحيل إرسال المعلومات باستخدام تجربة بيل (ASPECT هي تجربة الجرس) وفقًا لنظرية عدم الاتصال:
    https://en.wikipedia.org/wiki/No-communication_theorem
    حسنًا، حقيقة أنه من المستحيل إرسال المعلومات عن طريق التشابك كانت معروفة بالفعل في عام 1935. فلماذا لدينا هذه الفكرة في ورقة ويكيبيديا الخاصة بـ EPR:
    "بما أن هذا يعني أن أحد الجسيمين يتواصل مع الآخر بشكل فوري عبر الفضاء"؟
    هل سيحرج أينشتاين نفسه أمام العالم أجمع في أهم أبحاثه منذ النسبية العامة؟ أو ربما رأى المايسترو أن نقل المعلومات بشكل أسرع من الضوء وبالتالي إلى الماضي، يحدث بشكل يتعارض مع النسبية في حالة التشابك؟

    وإذا قال أينشتاين أن هناك تناقضًا بين التشابك الكمي والنسبية، فهل يمكننا أن نقول لآينشتاين أنه مخطئ؟ هل نفهم النسبية أفضل من أينشتاين؟

    6. ولكن ماذا عن كل النجاحات التي لا جدال فيها في النسبية؟ نظام تحديد المواقع؟ الميونات؟ وقبل كل شيء - E=mc^2؟

    أستطيع الإجابة على ذلك بسؤال: ماذا عن كل النجاحات التي لا جدال فيها لنظرية نيوتن؟ تقريبًا كل التقنيات التي نعرفها تعتمد عليها، ومع ذلك فإن نظرية نيوتن هي مجرد حالة خاصة من النسبية في السرعات المنخفضة.
    وE=mc^2؟ هذا أينشتاين خالص أليس كذلك؟

    من ويكيبيديا:

    أولينتو دي بريتو (26 أبريل 1857 - 16 مارس 1921) كان صناعيًا وجيولوجيًا إيطاليًا من سكيو، فيتشنزا. هناك ادعاءات[1] بأن دي بريتو ربما كان أول شخص اشتق معادلة كتلة الطاقة E=mc^{2}، والتي تُنسب عمومًا إلى ألبرت أينشتاين. واقترح أن التحلل الإشعاعي لليورانيوم والثوريوم كان مثالاً على تحول الكتلة إلى طاقة.

    من عام 1899 إلى عام 1903، بدأ دي بريتو بدراسة المجال الناشئ للفيزياء النووية وعلاقته بعلم الفلك. وركز على نظرية الأثير، وهي مادة افتراضية كان يعتقد في ذلك الوقت أنها تملأ كل الفضاء.

    استخدم دي بريتو التعبير mv^2 للدلالة على "vis viva" ومخزن الطاقة داخل المادة، حيث حدد v بسرعة الضوء. صيغته تسبق بسنتين، وهي تتفق مع صيغة ألبرت أينشتاين اللاحقة E=mc^{2} لتكافؤ الكتلة والطاقة، والتي اشتقها أينشتاين نتيجة للنظرية النسبية الخاصة.

    لقد أثرت هذه الاعتراضات على النسبية لسنوات عديدة، ولم يتفق معي الكثيرون. ولكن منذ أقل من عام، وجدت مقالًا في بحث مجلة SCIENTIFIC AMERICAN لعام 2009: تهديد كمي للنسبية الخاصة، والذي يقول بالضبط ما كنت أقوله لسنوات عديدة:

    "إن وضع النسبية الخاصة، بعد أكثر من قرن من تقديمها للعالم، أصبح فجأة مسألة مفتوحة بشكل جذري وسريعة التطور. لقد حدث هذا الوضع لأن الفيزيائيين والفلاسفة تابعوا أخيرًا الأطراف الفضفاضة لحجة أينشتاين المهملة منذ فترة طويلة مع ميكانيكا الكم.

  63. إسرائيل
    إسقاط نظرية الكم للحظة. تجاهلها تماما.
    فهل هناك الآن سبب لعدم قبول الافتراضات الأساسية للنظرية النسبية، واستنتاجاتها؟

    كتمرين، حاول شرح قصة الميونات دون اختصار الطول.

    النسبية بسيطة، خاصة وعامة. نظرية الكم معقدة للغاية. لماذا تفضل المعقد؟

  64. دعونا نرى ما أحاول القيام به هنا.

    لا أحاول أن أشرح أن المشهد أو نقل المعلومات -رغم أن لدي فكرة ما- فقد اتفقنا بالفعل على أنه طالما أنك تحصل على القدرة على التأثير من مسافة بعيدة في زمن الصفر فلا داعي لاستخدام الكلمة معلومة.

    ما أحاول القيام به كجزء من التعليقات في هذه المقالة هو إظهار أساس التناقض المفترض بين الكم والنسبية.

    لقد أحضرت مقالة مجلة ساينتفيك أمريكان. يمكن أن نتفق مع ما ورد فيه أو لا، لكن الأطروحة هناك هي أن عدم المحلية في التشابك يهدد النظرية النسبية الخاصة، بسبب تأثير من مسافة أسرع من الضوء.

    لا يتضمن ادعائي التشابك أو التأثير الأسرع من الضوء، ولكنه يتعلق بمشكلة أساسية في الفرضية الأساسية للنسبية: أن الفوتون موجود فقط عند نقطة معينة في لحظة معينة. وهذا الاختباء عما نعرفه من ميكانيكا الكم يسبب، في رأيي، كل المشاكل الأخرى إن وجدت، كما هو مكتوب في المقال.

    أحضرت أيضًا وصفًا للفوتون يمكنه حل غرابة النسبية وأيضًا تسليط الضوء على أشياء غريبة جدًا في الكم مثل التأثير على الماضي.

    وهذا الوصف ليس اعتباطيا فحسب - بل هو استدلال منطقي ناشئ عن نموذج ماكسويل للأثير في نظام مفتوح ولا نهائي، ومما يبدو لي أنه خطأ جوهري في نقطة البداية لتجربة ميكلسون-مورلي.

  65. إسرائيل
    كيف يفسر هذا النسيج؟ كيف يقوم زوج من الفوتونات بنقل المعلومات بينهما؟ هل يرسلون المزيد من الفوتونات بينهم؟ إذا كان الأمر كذلك فإن الكاشف الموجود بينهما سوف يكتشف هذه الفوتونات.

    جميع السرعات شاملة جميع الاتجاهات؟ إذا لم يكن الأمر كذلك، كيف تفسر التجاوز؟

  66. بلا شك. يمكنك إصدار فوتون من النقطة A وبعد وقت مناسب تستقبله عند النقطة B.

    أما إذا كانت منتشرة في الفضاء من قبل، وباحتمال متساو، فهناك احتمالان:

    1. إن حقيقة الفوتون فيما بعد B ليست شيئًا ماديًا - فهي مجرد وظيفة موجية أو فلسفة أو علم فيروسات الحمار الوحشي، ولكن ليس الفوتون نفسه.

    (غريب بعض الشيء إذا اعتبرنا أن مربع الدالة الموجية هو احتمال العثور على الجسيم عند نقطة معينة).

    2. الجسيم نفسه أيضًا يتجاوز B.

    في الحالة الأولى، لا توجد حقائق بالنسبة للفوتون الذي يتحرك بشكل أسرع من c، وأينشتاين على حق والكم يندم. لكن حسب فهمي، ليس هذا هو الحال.

    في الحالة الثانية، إذا كان الفوتون نفسه يتجاوز B في زمن أقل من ct، فلا مفر من الاستنتاج بأنه يتحرك بشكل أسرع من c وفي الواقع بأي سرعة.

    هل أعرف ما هو الحل؟ سلبي. ولكن يبدو لي أن لا أحد هنا يعرف. اتفق معي طبيب في الفيزياء على أن الجسيم كان أيضًا يتجاوز B قبل القياس.

    اقترحت ما يبدو لي معقولًا ويناسب ظروف المشكلة: الفوتون يتحرك في جميع السرعات مثل شريط مطاطي يتمدد، لكن أجهزة القياس لدينا لا يمكنها التقاط سوى الجزء الذي يتحرك بالنسبة إلى جهاز القياس عند c. وشرحت أيضًا السبب - على سبيل المثال، يمكن تسجيل القذائف التي تمر عبر نفق في إسرائيل وتلك التي تتحرك بسرعة معينة. في هذا المثال، الجزء الذي يتحرك بشكل أسرع من c لا يمكن ببساطة التقاطه بواسطة جهاز القياس، تمامًا كما لن يتم التقاط القذيفة التي تتحرك بما يتجاوز سرعة الإفلات في النفق.

    ليلة سعيدة وسنة جديدة سعيدة

  67. إسرائيل
    أنا لم أقل ذلك أبدا. أقول ذلك طالما أننا لم نجده على بعد مليار سنة ضوئية، فلا يهمنا.

    يغادر A ويصل إلى B. إذا بحثت عنه في خط مستقيم بين A و B ستجده. إذا بحثت عنه في أندروميدا فلن تجده.
    هذا هو الرأي كجسيم. إذا كنت تريد أن تنظر إليها كموجة مطبقة في الفضاء، فهذا شيء آخر.

    هل تقبل أن أتمكن من إصدار فوتون من النقطة أ وبعد وقت مناسب أستقبله عند النقطة ب؟

  68. فلسفة اليك..

    إذًا أنت تدعي أن الفوتون فقط من الناحية الفلسفية يمكن أن يكون على بعد مليار سنة ضوئية بعد 9 ثوانٍ وليس ماديًا؟ أنه ليس له وجود مادي على بعد مليار سنة ضوئية؟

    فلماذا يظهر هذا الوصف في كتاب الفيزياء وليس كتاب الفلسفة؟

  69. إسرائيل
    هل الشجرة التي تسقط في الغابة دون وجود أحد حولها تحدث ضجة؟
    عندما لا ينظر أحد، هل يمكن أن يكون الفوتون في كل مكان؟ نعم - طالما لم ينظر إليه أحد. وهذا ليس هو الحال بالنسبة لفوتون أينشتاين. سنكتشفه على مسافة مقطعية دائمًا.
    فكر في تجربة الفتحتين. نحن نعرف متى يغادر الجسيم ومتى يصل. في الطريق، يكون للجسيم خصائص موجة - تمر الموجة أولاً عبر شقوق ضيقة وبالتالي يتم إنشاء الحيود. عند الحواف - إنه جسيم لكل شيء. في تجربة أينشتاين، سوف تتجلى هذه الظاهرة في تشتت معين للفوتونات في المجمع. حتى أفضل الليزر لديه تشتت الشعاع.

  70. فهل تعتقد مثل أينشتاين أنه إذا غادر الفوتون النقطة A في الزمن 0 فإنه بعد الزمن t سيبتعد عنها ط م؟

    إذن ما هي القصة بأكملها أنه قبل ثانية واحدة كان على بعد مليار سنة ضوئية منها؟

    وما علاقته بالقياس؟ هذه تجربة فكرية لم يتم فيها ذكر القياس، بل حساب موضع الفوتون من خلال الفكر.

    أو ربما يتسبب الفكر في تركيز الفوتون عند نقطة معينة؟

  71. إسرائيل
    وأنا أتفق مع ذلك. لا أفهم ما الذي يزعجك في ذلك.

    وهذا لا يتعارض مع نظرية الكم. والأكثر من ذلك - هناك تجارب معملية للفوتونات المفردة، والتي نعرف فيها مكان وجود الفوتونات الفردية، ومتى.

    نحن نعرف كيف نرى العديد من الجسيمات الأولية، بمساعدة الخلايا الفقاعية على سبيل المثال. يكتشف عداد جيجر أيضًا الجزيئات الفردية.

    صحيح - ينص مبدأ عدم اليقين على أنه لا يمكن معرفة أزواج معينة من البيانات، مثل الزخم والموضع، بدقة غير محدودة. لكن الدقة عالية بالتأكيد بما يكفي لتأكيد النظرية النسبية الخاصة.

    هم... ربما بسبب مبدأ عدم اليقين، عندما نرى فوتوناً واحداً لا نعرف ما لونه؟ (هذا هراء بالطبع... هناك سبب آخر لذلك).

  72. لا أعتقد أن فكرتي جيدة مثل فكرتهم أو حتى جيدة. فقط التجربة ستحدد.

    ونظرية نيوتن لا توفر الدقة. تجربة مذهلة؟ ونظرية بطليموس؟

    لم أفهم لماذا لا توافق على أن النسبية مبنية على فكرة أن الفوتون موجود في لحظة معينة في مكان معين، وهو ما يتعارض مع الكمات.

  73. إسرائيل
    هناك العديد من التناقضات بينهما (الثقوب السوداء، الانفجار الكبير، الحتمية مقابل العشوائية، المحلية وما إلى ذلك). ولكن - كل في مجاله يوفر دقة تجريبية لا تصدق.

    هناك نضارة في الجمع بينهما، أفكار لأشخاص يفهمون كليهما. ما الذي يجعلك تعتقد أن فكرتك أفضل من فكرتهم؟

  74. حسنًا ، بالفعل.

    ولكن هذا هو ما قيل لي بشكل أو بآخر عن ما يسمى بالتناقض بين الكم والنسبية، ثم فجأة هناك مقال في مجلة ساينتفيك أمريكان يقول بالضبط ما أنا عليه.

    هذا لا يعني أنني وأنا على حق، لكنه يعني بالتأكيد أن هناك من يفهمني.

    لذلك ربما في غضون سنوات قليلة سيكون هناك مقال حول ما أقوله الآن.

  75. لطيف

    أيضا في العين. وماذا في ذلك؟ ألا ترى أن هذه تجربة فكرية؟

    حسنًا، إذا لم أتمكن من إيصال هذه النقطة إليك الآن، فسننتظر إجابة من المدونات التي طرحت فيها السؤال.

  76. إسرائيل
    من الناحية الفنية – من الممكن إرسال فوتون واحد واستقباله في الكاشف. ويتم ذلك في التجارب المعملية.

    أنا حقا لا أفهم ما الذي يزعجك.

  77. المعجزات

    بإمكانك أن تقرأ مباشرة في ورقة أينشتاين النسبية من عام 1905، بدون ترجمة.

    دع شعاع الضوء يغادر من A في الوقت ta، ودعه ينعكس عند B في الوقت t، ويصل إلى A مرة أخرى في الوقت t'a

    http://www.fourmilab.ch/etexts/einstein/specrel/www/

    التجربة هي تجربة فكرية، وما تقوله هو كما يلي: دع الفوتون يترك النقطة أ، ويصل إلى النقطة ب، ويعود إلى النقطة أ.

    ثم يواصل تطوير النسبية بالطريقة المألوفة، ولكن لا يتعلق الأمر بالقياس، بل يتعلق فقط بتجربة فكرية واستخلاص استنتاجات منطقية منها.

    لكن هذا المنطق يرتكز على ما كان واضحا من إيليو عام 1905، وهو أن الفوتون له موقع محدد في وقت معين وفي لحظة معينة، وهو بعيد عن المصدر.

    وإذا كانت هذه الفرضية غير صحيحة، فماذا يقول ذلك عن النسبية؟

    حيث أن فرضية عدم نقل المعلومات في التشابك هي أن "جميع البيانات موجودة بالفعل في النظام".

    ولكن إذا تم انتهاك هذه الفرضية الأساسية، كما هو الحال في تجربة جانبية حيث يتم إضافة عامل آخر إلى النظام، وهو حالة المستقطبات التي لم يتم تضمينها في البيانات الأولية، فماذا يقول هذا عن نقل المعلومات؟

    قليل من التوافه: تم تسمية التجربة باسم Aspect على الرغم من إجراء تجارب متطابقة تقريبًا قبل عقد من الزمن. إن الشيء العظيم في Aspect هو أنها تمكنت من تغيير حالة المستقطبات بعد أن غادرت الفوتونات المصدر بالفعل، وبالتالي تجنب الخوف من أن المستقطبات سوف "تتآمر على مؤامرة" (هكذا في الأصل!) وبالتالي ستختفي البيانات. تم تضمينها بالفعل في النظام.

  78. "جذر الخلاف"، إسرائيل شابيرا، هو في التفسير. إن السياسة في العلم هي التي تخلق الجدل. وليس التجارب.

  79. إسرائيل
    لا، أنا لا أوافق. الفكرة كلها مبنية بالتأكيد على القياس. إن إطالة الزمن وتقصير الطول وزيادة الكتلة ما هي إلا نتائج قياس.

    إذا جلست في مركبة فضائية تتسارع بمقدار 1 جرام لسنوات، فلن يتغير شيء بالنسبة لك. أنت لا توافق على ذلك؟ إذا قام شخص ما في نظام آخر بالقياس، فهو الذي سيحصل على نتائج نسبية.

    لا أفهم المشكلة هنا. يتم إجراء تجربة أينشتاين كل يوم عن طريق قياس الميونات، على سبيل المثال.

    من الناحية الفنية، الفوتون هو جسيم ضوئي... نرى فوتونًا واحدًا في العين 🙂

  80. المعجزات

    ו?

    إن فكرة تمدد الزمن برمتها (على الأقل بحسب مقالة أينشتاين)، مبنية على حقيقة أن الفوتون موجود في مكان معين في لحظة معينة، ولا علاقة له بالقياس. يوافق؟

    إذن هل تدعي أن هذا صحيح؟ هل يمكن للفوتون أن يتواجد في لحظة معينة في مكان معين؟ فماذا كان يفعل على بعد مليار سنة ضوئية من قبل؟

    وما علاقة ما عرفه أينشتاين عام 1905؟ لماذا إذا كان لا يعرف ثم لم يحدث؟

    ولا ينبغي أن نذكر أن أينشتاين نفسه حارب بكل قوته من أجل هذا الوصف للفوتون، وخسر.

  81. إسرائيل
    لقد قمت بإنشاء فوتون في مصباحك اليدوي. قلق النقل في اتجاه معين. لا يمكنك تحديد شروط تكوين الفوتون بدقة، وبالتالي سيكون له تشتت معين.
    فكر في شعاع الليزر - كلما كان الشعاع أضيق، زاد التشتت (وبالتالي، عند قياس المسافة إلى القمر، يتم إرسال النبض عبر التلسكوب، مما يؤدي إلى توسيع الفص).
    مثال آخر -- الرادار. لتحسين التمييز الزاوي، يتم زيادة الهوائي. يؤدي التكبير إلى توسيع الفص، لكنه يقلل من التشتت.

    ألبانزو
    في هذه الحالة (إرسال فوتون من A إلى B)، يصعب علي أن أفهم أنه لا توجد معلومات حول الموضع على طول الطريق. إذا وضعت قرصًا به ثقب (يمكن سده) في المنتصف، فلن نتمكن من رؤية الفوتون عند B إلا إذا كانت الفتحة مفتوحة في الوقت المناسب. إذا كان جسيمًا مشحونًا، فيمكنني أيضًا التحقق من أن الجسيم مر عبر الثقب.

  82. حاول مرة أخرى هذه المرة.

    هل للفوتون موقع محدد قبل القياس؟

    أليست العلاقة مبنية على. يعني لها موقع مميز؟

    لقد قرأت المقال، أليس كذلك؟ وخلافا لما يقال هنا، فإن عنوان المقال يشكل تهديدا كميا للنظرية النسبية، والأقوال فيه واضحة لا لبس فيها. أنا فقط أحاول أن أبين أين يكمن جذر الخلاف، وذلك دون تشابك وسرعة الضوء.

  83. ألبانزو

    الفوتونات والإلكترونات والسرعات والقوى. فيزياء.

    وليس أنت، لم تدرس ولم تفهم وكذبت. علم النفس.

    علاوة على ذلك، كشخص يكتب لا يوجد فرق إلا، وبعد ذلك اتضح أن هذا الاستثناء هو الشيء الرئيسي، كنت أفكر مرتين قبل إلقاء محاضرة على شخص ما حول فهم القراءة.

    نسيم، المشرح الوطني.

    فكيف يعمل؟

  84. معجزات,

    بالنجاح. أعتقد أنك كنت قادرًا على متابعة كلامي وفهمه. إذا كنت تريد تجربة حظك، سأقدم لك المساعدة.

    إسرائيل،

    الجواب على سؤالك موجود في ما كتبته. إذا كنت تريد أن تفهم، كل ما عليك فعله هو قضاء بضع دقائق في الرجوع إلى الوراء - أعدك أن الإجابة موجودة. إذا كنت لن تكلف نفسك عناء قراءة الأشياء التي قضيت الكثير من الوقت في كتابتها وبذلت الكثير من الطاقة في محاولة مساعدتك - فأنا لا أرى كيف يمكنك أن تدعي بصدق أنك تريد أن تفهم.

  85. إسرائيل
    لقد حددت موقعها واتجاهها.

    لم تقم بتحديد استقطابه - لذلك ستجد أنه عند B هناك احتمال بنسبة 50% أن يكون الفوتون مستقطبًا رأسيًا أو أفقيًا، اعتمادًا على كيفية وضع المستقطب.
    إذا وضعت مستقطبًا رأسيًا عند A، فستجد الآن أنه إذا وصل الفوتون إلى B، فإن استقطابه يكون عموديًا دائمًا.
    وهنا يكمن السحر - إذا وضعت مستقطبًا بزاوية 45 درجة على الطريق، فستجد أنه مرة أخرى هناك احتمال أن يكون مستقطبًا رأسيًا أو أفقيًا (مرة أخرى، اعتمادًا على كيفية وضع المستقطب عند النقطة B).

  86. لطيف - جيد،

    ما القياس الذي أجريته على الفوتون في التجربة (الفكرية) التي هي أساس النسبية؟

    دع شعاع الضوء يغادر من A في الوقت ta، ودعه ينعكس عند B في الوقت t، ويصل إلى A مرة أخرى في الوقت t'a

    http://www.fourmilab.ch/etexts/einstein/specrel/www/

    أين ترى القياس هنا؟

  87. ألبانزيون

    شكرًا على السلسلة التعليمية، ولكن إذا كنت لا تريد القتال أثناء كتابتك، فحاول التحدث فقط عن الفيزياء ولا تصبح شخصيًا.

    وإذا لم يكن أينشتاين يعرف ميكانيكا الكم فهل يعني ذلك أنها غير موجودة؟ ففي النهاية، الافتراض الأساسي للنسبية هو أن الفوتون موجود في مكان محدد في وقت محدد، ويتحرك بسرعة واحدة فقط، ج.

    فإذا كان هذا الافتراض غير صحيح، فماذا يقول هذا عن النسبية؟

  88. إسرائيل
    حصلت على إجابات:
    1) نعم، وهذا ينطبق أيضًا على الفوتونات.
    2) بمجرد وصف الفوتون، تكون قد قمت بقياسه بالفعل، لذلك فهو لا ينتشر في الفضاء كما في 1).

    ألبانزو
    ما لا أفهمه هو كيف سارت الأمور مع تجربة الشقين - ففي نهاية المطاف، نحن نطلق الجسيمات هناك كما تصفها إسرائيل، ومع ذلك نشهد أن الجسيمات منتشرة في الفضاء. أعتقد أن التفسير هو المعرفة الجزئية - أعرف متى خرج الجسيم، لكني لا أعرف في أي اتجاه. على وجه الدقة - ليس معرفتي هو المهم، ولكن القياس نفسه.

    شموليك
    فكرة بنروز مثيرة للاهتمام. وهذا ما يفسر لماذا يبدو لنا العالم العادي ... عاديا.

  89. إسرائيل،

    أنت فقط لا تقرأ ما أكتب. لماذا يجب أن أزعجك بعد الآن؟

    في عام 1905، لم تكن هناك ميكانيكا الكم، وكتب أينشتاين النسبية الخاصة كامتداد للميكانيكا الكلاسيكية. ومنذ ذلك الحين تعلمنا وأعيدت صياغة التوراة بطريقة يمكن تطبيقها أيضًا على ميكانيكا الكم. إن الإصرار على النظر في ورقة بحثية صدرت عام 1905 لفهم النسبية الخاصة يشبه محاولة دراسة التطور من خلال التركيز فقط على "أصل الأنواع"، أو علوم الكمبيوتر فقط من تورينج وعمله في الأربعينيات.

    ربما لا تريد أن تفهم. على اللوحة في وقت لاحق.

  90. لم أقل أنني لا أعرف، أردت التأكد فقط..

    لذلك أعود إلى السؤال الأصلي الذي فتح كل الرهانات:

    إسرائيل شابيرا
    26 ديسمبر 2017 الساعة 12:43 مساءً
    هل هناك من يتطوع للمساعدة؟

    ينص مبدأ عدم اليقين على أن الجسيم الكمي ليس له خصائص مميزة قبل القياس. فهل هذا صحيح أيضًا بالنسبة للفوتونات؟ وإذا كان الأمر كذلك، فكيف يتناسب مع النظرية النسبية التي يقول فيها أينشتاين في مقالته الأصلية:

    دع شعاع الضوء يغادر من A في الوقت ta، ودعه ينعكس عند B في الوقت t، ويصل إلى A مرة أخرى في الوقت t'a

    http://www.fourmilab.ch/etexts/einstein/specrel/www/

    بعد كل شيء، نحن هنا نتحدث عن حقيقة أن شعاع الضوء (يمكن أيضًا أن يكون فوتونًا واحدًا) موجود في مكان محدد - النقطة A - في وقت محدد - t'a - وزخمه محدد - ثابت بلانك مقسومًا على الطول الموجي للفوتون.

    فكيف يعمل؟

  91. لدى روجر بنروز فكرة مثيرة للاهتمام تربط بين ميكانيكا الكم والنسبية العامة. ويقول إن الجسيم يمكن أن يبقى في حالة تراكب طالما أن انحناء الزمكان لا يتجاوز عتبة معينة. إذا تم تجاوز العتبة (يفترض أن العتبة تبلغ حوالي كتلة بلانك واحدة) فسيحدث الانهيار. وفي فهمي جاءت فكرته لتفسير الانتقال من النظام الكمي إلى النظام الكلاسيكي وليس الانهيار الذي يحدث في تجربة الشقين. حتى أنه يقترح تجربة لاختبار فكرته.
    https://en.m.wikipedia.org/wiki/Penrose_interpretation

    ألبانتيزو، هل تعرف أي تجارب أخرى تهدف إلى محاولة اختبار حدود ميكانيكا الكم؟

  92. ألبانزو
    القياسات لا تعني أن المرء يعرف الموقع، ولكن كل قياس "كلاسيكي"، أليس كذلك؟

    مثال: إذا قمت بإجراء تجربة الشق المزدوج مع الإلكترونات في معمل مغلق، وقمت بتسجيل موضع الإلكترونات باستخدام كاشف، فلن تتشكل صورة تداخل. هذا الشرط موجود، حتى لو لم تقم أجهزة الكشف بالتسجيل.

    ما هو غير واضح بالنسبة لي: الفيلم الفوتوغرافي هو أيضًا نوع من أجهزة الكشف... هل يشبه المستقطبات؟ هناك تجربة بسيطة مع 3 مستقطبات، والتي تبين أن القياس على محور واحد "يحذف" المعلومات الموجودة على المحور الرأسي. إنها تجربة مذهلة للغاية 🙂

  93. بالطبع، عندما أكتب "يمكن للفوتون أن يتحرك بشكل أسرع من الضوء" أعني "يمكن للفوتون أن يتحرك بشكل أسرع من الثابت c". الفوتون هو الضوء وبالتالي حسب تعريفه فإنه يتحرك دائما بسرعة الضوء :). وبطبيعة الحال، الإشارة هنا هي إلى حركة أسرع من سرعة الضوء في الفيزياء الكلاسيكية، وهو الثابت الذي يرمز إليه بـ c. وكما قلت، حتى في ميكانيكا الكم يتحرك الضوء بهذه السرعة ولكن بين نقطتي قياس فقط. وعندما لا يتم قياسه، فإنه لا يتحرك بالضرورة بهذه السرعة.

  94. كما كتبت في التعليقات السابقة (يمكنك الاطلاع عليها مرة أخرى والتأكد بنفسك)، يلزم وجود فوتون كينو للتحرك بسرعة الضوء. الالتزام الوحيد هو فيما يتعلق بالقياسات، ولكن طالما لم يتم قياسها، فيمكن أن تتحرك بشكل أسرع أو أبطأ من الضوء. وينطبق هذا أيضًا على الجسيمات الأخرى، حيث يمكن للإلكترون أن يتحرك بشكل أسرع من الضوء عندما لا يتم قياسه. يمكن للجسيمات أيضًا أن تتحرك في مسارات منحنية على الرغم من عدم وجود قوة تؤثر عليها. وميكانيكا الكم تميز بين خصائص الجسيم والصورة التي نراها. وهذا الاختلاف هائل في أهميته وهو المسؤول عن معظم الظواهر الكمومية. معظم الأشياء التي اعتدنا عليها من الميكانيكا الكلاسيكية موجودة فقط في الصورة التي تم الحصول عليها من القياس، وليس من حيث المبدأ.

    إذا كنت قد درست منذ سنوات عديدة ولا تتذكر حتى المبادئ أو المصطلحات الأساسية، فكيف تتوقع أن تفهم؟ أنصحك بإجراء تحديث شامل للغاية. الأشياء التي أشرحها لك هنا هي مجرد أساسيات وبدونها فلا عجب أنه ليس لديك أدنى فكرة عندما تطرح أسئلة أكثر تقدمًا حول النسيج، على سبيل المثال.

  95. لقد تعلمت، ولكن كان ذلك منذ سنوات عديدة.

    "ولكن في الثانية التاسعة (عندما لم تقيسها) كان من الممكن أن تكون على بعد مليار سنة ضوئية."

    لذا، إذا تمكن الفوتون من السفر مليار سنة ضوئية في تسع ثوان، ألا يعني ذلك أنه تجاوز سرعة الضوء قليلاً؟

  96. إسرائيل،

    إذا كنت تريد أن تحاول فهم ميكانيكا الكم والتحدث عنها، عليك أن تبدأ بتطبيق أفكارها الأساسية. في الفيزياء الكلاسيكية لا يوجد فرق بين "الجسيم هنا" و"عندما قمت بالقياس وجدت الجسيم هنا". في الكم هناك فرق سحيق.

    كما كتبت بالفعل عدة مرات - إذا قمت بقياس موضع الفوتون عند نقطة معينة (أو أنتجت فوتونًا يكون موضعه محددًا بشكل أو بآخر، حتى مستوى قليل من عدم اليقين) وقياس موضعه بعد 10 ثوانٍ، فإن الفرق بين المواضع ستكون 10 ثواني ضوئية. لقد كتبت هذا بوضوح عدة مرات بالفعل. ولكن طالما أنك لا تقيس الفوتون، فلا يمكنك ضمان مكان وجوده. أي أنك بعد 10 ثوانٍ قمت بقياسها ورأيت أنها كانت على بعد 10 ثوانٍ ضوئية، لكن في الثانية التاسعة (عندما لم تقم بقياسها) كان من الممكن أن تكون على بعد مليار سنة ضوئية. هذا غير ممكن في الفيزياء الكلاسيكية، وهو كذلك في الفيزياء الكمومية. وهذا الفرق كبير، لأن الفيزياء لا تتحدد بما نقيسه فحسب، بل أيضًا بما يحدث خلف الكواليس عندما لا نقيسه (من الأمثلة على ذلك تجربة الشقين، ومقياس تداخل ماخ-زيندر، وما إلى ذلك).

    الفوتون ليس له مسار في ميكانيكا الكم. أنت تعرف مكانه لحظة القياس الأول، وإذا قمت بقياسه مرة أخرى ستعرف مكانه لحظة القياس الثاني. سيكون الفرق بينهما ct، لكن هذا ليس سوى جزء صغير من فيزياء المشكلة. إذا كنت تريد فهم هذه المواضيع، فيجب عليك دراسة مقدمة عن الكم.

  97. في الفيزياء الكلاسيكية، إذا أطلقت بندقية رصاصة بسرعة 300 متر في الثانية، فبعد 10 ثوانٍ ستكون على بعد 3000 متر من البندقية. ليس من الممكن العثور على الكرة على مسافة 2900 أو 3100 متر.

    لذلك أريد التأكد: إذا بعث مصدر ليزر فوتونًا واحدًا في اللحظة 0، وقمنا بتقسيم الخط المستقيم في اتجاه حركة الفوتون إلى فترات 300,000 كيلومتر، فبعد 10 ثوانٍ سيجد الضوء على بعد 10 ثوانٍ من الليزر؟ أليس لديه احتمال أن يكون بين الفاصل 7 و8 أو بين 13 و14؟ وإذا كان الاحتمال موجودا فهل هو ضئيل؟

  98. معجزات,

    الآن أعتقد أنك ربما سألت شيئًا آخر. ربما قصدت أنك تقيس زخم الجسيم (الذي لا تعرف ما هو طوله الموجي - فأنت مجرد مجرب يجد إلكترونًا في المختبر ويقيس زخمه). بالطبع، سيكون لنتيجة القياس انحراف معياري، وهو ما يمثل عدم اليقين في موضع الإلكترون (افترض في الوقت الحالي أن أدوات القياس الخاصة بك دقيقة بشكل لا نهائي ولم تعد تدخل خطأ في النظام). هل تتساءل عما إذا كان الانحراف المعياري الكبير في الزخم في مثل هذه الحالة (أو بعبارة أخرى، نقص المعلومات في الزخم) يشير إلى توازن المعلومات في المركز؟

    نعم و لا. نعم، إذا كان لديك نسخ عديدة من نفس الجسيم وقمت بإجراء قياسات الزخم في النصف، وفي النصف الآخر قياسات الموقع، فإن الكثير من المعلومات على جانب واحد ستترجم إلى معلومات قليلة على الجانب الآخر. أي أنه في معظم الحالات، إذا كان بإمكانك وصف الزخم باستخدام أجزاء قليلة جدًا من نصف الجسيمات، فسوف تحتاج إلى الكثير من البتات لوصف موضع النصف الآخر (هناك بالطبع مواقف تنطوي على قدر كبير من عدم اليقين في كل من الزخم و الموقف، ولكن دعونا ننسى لهم الآن).

    لا، لأن هذا بالطبع مجرد بيان إحصائي حول مجموعة من الجسيمات. بمجرد قياس زخم جسيم واحد، فإنك تغير حالته وتحركه على وجه الخصوص (تغير موضعه). من المستحيل ببساطة قياس الزخم والموقع في نفس الوقت، وبالتالي فإن السؤال ليس محددًا بشكل جيد (يعتمد الأمر كليًا على ترتيب قياساتك - الزخم الأول ثم الموضع، أو الموضع الأول ثم الزخم).

  99. إسرائيل،

    الفوتونات لها موقع. وبما أن الفوتونات نسبية دائمًا، فيجب أن نتحدث عنها في إطار ميكانيكا الكم النسبية - أو بعبارة أخرى، نظرية المجال. هناك طرق للتعامل معها أيضًا في إطار ميكانيكا الكم "العادية"، لكن هذا مجرد تعقيد للأمور. في نظرية المجال، الموضع ليس عاملًا ولكنه تسمية للمجال - تصبح الشكلية الرياضية بأكملها أكثر تعقيدًا. لذا فإن السؤال "هل للفوتون عامل موضع" مضلل للغاية، لأن التوراة التي نصف فيها الفوتونات لا تتضمن مشغلي الموضع على الإطلاق. لكن نعم، إذا كنت تصر على التمسك بالشكليات الكلاسيكية لميكانيكا الكم، فمن الممكن تحديد عامل موضع للفوتون (لكن عليه مزج الموضع والزمن بسبب التباين النسبي، أي لأن الزمان والمكان مختلطان نسبيًا). . ليس من الواضح بالنسبة لي سبب متابعتك لهذا السؤال، ربما تريد التوضيح لأنك قرأت شيئًا عنه؟ على أية حال، هذا سؤال تقني معقد إلى حد ما ولا أرى أهميته. للفوتونات موقع يمكن قياسه (لكن لا يمكن تحديد موقعه بشكل كامل كما شرحنا سابقًا). الليزر الخاص بك لا ينتج فوتونات ذات زخم محدد، بل فوتونات في حزم موجية ضيقة تتمركز بقوة حول تردد معين، لكنها مع ذلك حزم موجية - أي تراكب من الزخم، وهذا يكفي للسماح لنا بـ " ضعهم ووصفهم بالشعاع. لأن الشعاع الموجي ضيق، لذلك عندما تقيس تردده الضوء تحصل باحتمال كبير جداً على نفس التردد لجميع الفوتونات وبالتالي القياس يظهر لك أن الضوء يكون على تردد واحد فقط، أي متماسك جداً. لكننا تحدثنا بالفعل عن الفرق السحيق بين ما يظهره القياس وما يحدث في الواقع - في ميكانيكا الكم، تظهر لنا نتيجة القياس نهاية واحدة فقط للفيزياء، والكثير من الأشياء تحدث خلف الكواليس. وكما ذكر نسيم، فإن هذا هو السر الكامل وراء تشابك الجسيمات أو تجربة الشقين والتي يتم تدميرها بالكامل بمجرد قياسها.

    معجزات,

    عليك أن تكون حذرا للغاية هنا. على أية حال، إذا كانت لدينا الدالة الموجية لجسيم ما (أو حالته الكمومية، فهي نفس الشيء تمامًا) فإننا نعرف كل ما يمكن معرفته عنه. تصف الحالة (أو الهاتف) الجسيم بالكامل. وعندما نقول إن هناك قدرا كبيرا من عدم اليقين في موقعه، فهذا لا يعني أننا نفتقر إلى المعلومات. وهذا يعني أن الجسيم نفسه موجود في مثل هذا التوزيع للمواقع بحيث يصعب جدًا تخمين المكان الذي سيتم قياسه فيه. أنت تعرف بالضبط ما هو التوزيع على أي حال.

    هناك بالفعل صلة هنا بنظرية المعلومات - التوزيعات التي تتسم بقدر كبير من عدم اليقين هي توزيعات ذات إنتروبيا عالية بالمعنى الكلاسيكي. لكنني لن أخوض في هذا الأمر، أعتقد أنه سيربكك أكثر من تعليمك أي شيء جديد. إجابتي المختصرة والمختصرة هي أن فون هيجل يخبرنا بكل ما يجب معرفته عن الجسيم في أي حال، سواء أخبرتنا أنه من المؤكد أنه موجود في نقطة معينة أو ما إذا كانت تخبرنا أنه يمكن العثور عليه في أي نقطة في الكون باحتمالات متساوية (من أجل ذلك، لنفترض أن الكون محدود). التطبيقات المثيرة للاهتمام لنظرية المعلومات في ميكانيكا الكم هي عند الحديث عن الجسيمات التي لا يمكن وصفها باستخدام حالة في فضاء هيلبرت لأنها ليست جزءًا من نظام مغلق. أو بمعنى آخر، حالات الديناميكا الحرارية.

  100. ليس فوتونًا مثاليًا، بل فوتونًا مثاليًا.

    واحدة منتشرة على مرتبة فوتون في حالة شاعرية، في حالة غير محددة، منتشرة على كامل المساحة، من قاعدة ضعيفة...

    المعجزات

    في فهمي للكم، ليس الأمر أنك إذا كنت تعرف الزخم الدقيق فإنك لا تعرف الموضع - ببساطة لا يوجد زخم أو موضع حتى القياس.

    ألبانزو

    قد يظن الكثير من الناس أنك إذا كتبت "لا يوجد فرق إلا..." فإنك تقصد أنه لا يوجد فرق إلا في التفاصيل الهامشية، وليس في الجوهر.

    ولكن كما كتبت، إذا اتفقنا على أن أحد الجانبين في التشابك يؤثر على الفور على الجانب الآخر، فسوف نقاضي.

    أنا لا أخاطبكم فقط - فالإشارة كانت "لكل من يتطوع للمساعدة". لقد تطوعت. يتجاهل؟

    إذن، أليست الفوتونات الناتجة عن الليزر منتشرة في الفضاء باحتمالية متساوية؟ هل يركزون فقط على المسافة المقطعية؟ هل يمتلك الفوتون عامل موضع على الإطلاق؟

  101. ألبانزو
    لدي سؤال غريب قليلا من وجهة نظري، كلما عرفت الزخم بشكل أفضل، حصلت بالفعل على مزيد من المعلومات (عدد البتات المهمة التي تمثل قيمة الزخم). هل يتعلق الأمر بـ "نقص المعلومات" لدي حول الموقع؟ أي هل هناك أي صلة بالمعلومات هنا على الإطلاق، أم أن التفسير أكثر مالية.

  102. معجزات,

    صحيح. الحالات التي لها زخم محدد جيدًا (أو موضع محدد جيدًا) موجودة رياضيًا ولكنها ليست مادية، إلا عندما يكون الفضاء محدودًا.

    إسرائيل،

    1. أنا أفهم. إذن أصل المشكلة هو أنني قلت صراحة ووضوح أن هناك فرقا وهو أن أحدهما كمي والآخر كلاسيكي، وقررت بنفسك أن هذا الاختلاف لا يكاد يذكر ومن ذلك استنتجت أنني كنت كذلك في الواقع يقول أنه لا يوجد فرق. من الصعب بالنسبة لي أن أفكر في فرق أقل أهمية، ولكن فليكن.

    2. أعترف وأعترف - أنني لم أقرأ حتى ما كتبته عن تجربة أسبا. بمجرد أن أدركت أنك تأخذ شيئًا محددًا قلته عن حالة معينة (اللغز حيث تم تحديد المعلومات المشتركة بين اثنين من المستخدمين النهائيين مسبقًا عند مطابقة أزواج الدوران الخاصة بهم) وتطبيقها على شيء لم أتحدث عنه حتى، أدركت ذلك في أحسن الأحوال، سأضطر إلى البحث في المناقشات السابقة مرة أخرى وأعلمك درسًا آخر خاصًا (أخبرني، هل لاحظت أثناء المناقشة عدد الأشياء التي أشرحها لك؟ ما مقدار المعلومات التي أشاركها معك؟ انتقل الآن إلى المناقشة و انظر كم من الأشياء قلتها لي. آسف، ولكن ما يحدث هنا لا يمكن أن يسمى مناقشة على الإطلاق لأنه من جانب واحد، ببساطة لا توجد كلمة أخرى باستثناء الدرس الخصوصي). وفي أسوأ الأحوال، إنها مجرد معركة أخرى. سأتركك للتفكير في تجربة التجميع بنفسك والتوصل إلى أي نتيجة تريدها.

    3. اسمع، لا أعرف كيف أشرح الأمر بشكل أوضح مما فعلت. يوصف كل جسيم في ميكانيكا الكم بشيء يسمى "الحالة". يمكن أن تكون هذه الحالة ترددًا معينًا (ما نسميه الفوتون المثالي)، ولكن لا يجب أن يكون كذلك، فهناك أيضًا فوتونات ليس لها تردد معين ولكن لها تراكب من الترددات. الفوتونات "المثالية" كما تقول (لا يوجد شيء أكثر مثالية فيها من غيرها، لكنني سأعتمد لغتك إذا كانت ستساعدك على الفهم) منتشرة في جميع أنحاء الفضاء، بغض النظر عن ترددها. ينتشر الفوتون المثالي بتردد 1 هرتز في الفضاء بأكمله، وينتشر أيضًا الفوتون المثالي بتردد مليار تيراهيرتز في الفضاء بأكمله. لا تنتشر الفوتونات غير المثالية في كامل المساحة، وبالطبع لا تختلف عن بعضها البعض في التردد (بعد كل شيء، ليس لها تردد محدد جيدًا، لذا فمن الواضح تمامًا أن هذه ليست السمة المميزة) بينهم). على الأكثر يمكن القول أنها تختلف في الطول الموجي لها في الفضاء الترددي (والذي، بالطبع، لا يحتوي على معلومات حول ترددها فقط ولكن عن جميع خصائصها الفيزيائية مثل الموقع).

    4. لا يمكن إنشاء فوتون مثالي كما تقول (أي له تردد معين) في مصباح يدوي. وهي منتشرة بالتساوي على كامل الفضاء، سواء في لحظة الخلق أو بعد 13 مليار سنة. الشيء الوحيد الذي يمكنه تغيير ذلك هو إذا تغير حالته الكمومية، نتيجة للتفاعل مع نظام آخر أو نتيجة للقياس. لذلك لن يكون منتشرا على كامل المساحة، لكنه لن يكون مثاليا أيضا.

  103. إسرائيل
    بقدر ما أفهم، يقول ألبينزو أنه بمجرد حصولك على معلومات حول الفوتون، تكون قد قمت بقياسه، وبالتالي لم يعد "مبعثرًا في جميع أنحاء الفضاء". إنها مشابهة لتجارب الشق - بمجرد النظر من خلال الشق الذي مر به الجسيم، فإنه لم يعد يمر عبر كليهما (بدون تداخل).
    ولذلك، فإن نفس الفوتون من مصباحك سوف يتحرك بطريقة كلاسيكية.

  104. ألبانزو
    أعتقد أنني أفهم ما تقوله:
    1) إذا كان لدي معرفة كاملة بزخم جسيم ما، فليس لدي أي معرفة بموقعه، لذا فإن هناك احتمالًا متساويًا للعثور عليه في أي مكان في الفضاء.
    2) عمليا - من المستحيل بالنسبة لنا أن نعرف الزخم (أو الموقع) بشكل مطلق.

  105. لنبدأ من النهاية.

    "ربما تظن أنني أريد إذلالك فقط وأنني أستمتع بإخبارك أنك لا تفهم ولا تعرف، لكنني أؤكد لك أن الأمر ليس كذلك".

    مستحيل؟ يسعدني أنك تخبرني بأشياء لم أكن أعرفها وتحاول التعلم منها.

    لكن الأمر مختلف قليلاً عندما تبدأ بالكذاب، والأحمق، والمحتال، وما إلى ذلك. هل ترى على الأقل في هذه الحالة الفرق الأساسي؟

    ولكن هل هذا يعني أنني أتفق مع كل شيء؟ سلبي. إذا قدمت حجة صحيحة أفهمها وأقبلها، فأنا أوافق وأشكرك، كما فعلت لشرح حالات بيل الأربع.

    إذا لم يكن الأمر كذلك، سأستمر في طرح الأسئلة وأضع فهمي ضد فهمك. هذه هي الطريقة للمناقشة والتعلم، حتى يقتنع أحد الطرفين (أو لا).

    ولكن ماذا أفعل عندما لا أتفق معك، تغضب على الفور وتبدأ بشتمي؟ حسنًا، دعونا نأمل أن يتحسن الأمر.

    هنا مثال:

    لقد كتبت: "لا يوجد فرق بين هذا الحل وأي حل آخر للكود الكلاسيكي المنسق مسبقًا (كما حاول بعض المعلقين هنا اقتراحه)، باستثناء أن هذا الكود يعمل وفقًا لقوانين الكم، وعلى وجه الخصوص وجد في التراكب."

    بالنسبة لي، الأمر يشبه القول: عليك أن تختار بين التوأم تشايا وموارتا. لا يوجد فرق بينهما باستثناء بعض التفاصيل الهامشية - بالنسبة لهيا هناك بقعة على ظهرها، وبالنسبة لمويرتا هناك ندبة صغيرة على يدها، وما إلى ذلك.

    لكن عندما تحاول أن تفهم ما هو الفرق، يتبين أنهما متطابقان بالفعل من الناحية الجينية، ويبدوان متماثلين وهما يرتديان نفس الملابس، كل شيء متماثل باستثناء الحقيقة التافهة والهامشية وهي أنها على قيد الحياة، ومويرتي ميتة. (ميرتي بالإسبانية). ضئيلة

    إذن ماذا يعني "لا يوجد فرق بين هذا الحل وأي حل آخر للكود الكلاسيكي المطابق مسبقًا"؟ الفرق هو في الغالب، أليس كذلك؟

    وهل يهم إذا التقيت من قبل وقمت بتنسيق الدورات؟ منذ اللحظة التي قدمت فيها بيانات جديدة - حالة المستقطبات في تجربة Aspect، والتي تتغير بعد مغادرة الفوتونات للمصدر وبالتالي لم يتم تضمينها في التنسيق السابق - لم يعد التنسيق السابق ذا صلة.

    ويمكن ملاحظة ذلك في مثال الأرقام المكتوبة على الملاحظات والمفصولة. صحيح في التنسيق الكلاسيكي أنظر إلى رقمي وأعرف ما هو رقمك دون نقل المعلومات.

    ولكن منذ اللحظة التي يتم فيها إدخال عامل إضافي - على سبيل المثال، يجب على كل طرف أن يضرب رقمه برقم آخر لا يعرفه الطرف الآخر - فما أهمية كل التنسيق المبكر بعد الآن؟

    لا يهم. إذا اتفقنا على أن نسج القياس على جانب واحد *يؤثر* على القياس على الجانب الآخر (وهو أمر غير موجود في حالة الأرقام أو القفازات التي تم فصلها) فلن يكون بيننا خلافات أخرى، وقد كتبت بالفعل مقالة طويلة منذ زمن مضى، لا يهمني إذا أطلقوا عليها اسم "معلومات"، بل يمكن أن يطلق عليها أيضًا اسم "موشيه".

    مجرد نصيحة صغيرة لك كمدرس: حاول عند كتابة "الارتباطات غير المحلية" و"قوانين الكم" أن تذكر أيضًا أن ذلك يتضمن القدرة على التأثير من مسافة بعيدة.

    الفوتونات. لا يزال الأمر غير واضح بالنسبة لي: هل هناك فرق بين الفوتونات باستثناء الطول الموجي؟ هل يمكن أن يكون هناك فوتونان لهما نفس الطول الموجي ولا يزال بينهما نوع من الاختلاف؟ حزم موجة مختلفة ربما؟

    وإذا كان الجواب سلبيا. عندما تقول "لنفترض أنني أنظر إلى فوتون محدد له زخم محدد جيدًا، أي أن عدم اليقين في الزخم هو 0. نعم، في هذه الحالة سيكون له عدم يقين لا نهائي في موضعه، أي أنه سوف ينتشر في كل مكان" عبر الفضاء"، ألا يتطلب الأمر أن تكون كل الفوتونات هكذا في الواقع؟ بعد كل شيء، الفرق بين ذلك الفوتون المثالي والآخرين هو فقط ترددهم، أليس كذلك؟ (الآن لا ندخل في نظرية توريا وما هو التردد، يكفي أن يكون هناك فوتون واحد ينتشر بالتساوي في الفضاء).

    وسؤال ثاني. هل يمكن إنشاء نفس الفوتون المثالي في مصباحي اليدوي؟

  106. إسرائيل،

    لنبدأ من النهاية. أنا آسف، فهمك للقراءة سيء. عندما يقولون "لا يوجد فرق بين موشيه وداني إلا أن موشيه أشقر وداني أحمر"، فهل يعني ذلك أن موشيه وداني هما نفس الشيء؟ عندما تقابلهم، هل ستتمكن من التمييز بينهم أم لا؟ كتبت أنه لا يوجد فرق *إلا* أن أحدهما يشير إلى القوانين الكلاسيكية والآخر إلى الكم. إنه فرق كبير. كيف توصلت إلى استنتاج أنني أقول أنه لا يوجد فرق بين الأنظمة المحلية والأنظمة غير المحلية؟ لقد كتبت بوضوح، بالأبيض والأسود، أن هناك فرقًا، وهذا الاختلاف هو الفرق بين القوانين الكمومية والقوانين الكلاسيكية. الكم يسمح باللا محلية، والكلاسيكي لا يسمح بذلك. يا إلهي، يبدو الأمر وكأنك لا تحاول حتى قراءة ما يُكتب لك. كالعادة، المشكلة تنبع من أنك تركز على جملة واحدة (أو فقرة واحدة) تعتقد أنها ستساعدك على التقدم برأيك وإثبات أنك على حق، وتتجاهل كل شيء آخر. العودة إلى المناقشة. حاولت في المناقشة الأصلية أن أشرح لك شيئًا بسيطًا:

    1. من الناحية الكلاسيكية، من المستحيل حل اللغز دون نقل المعلومات. نعم، يمكن حلها عن طريق التنسيق المسبق للشفرة بين المجربين.
    2. يمكن حلها عن طريق الدورات المتشابكة.
    3. لقد استنتجت أن الدورات المتشابكة تتضمن نقل المعلومات.
    4. هذا خطأ. إن السبينات المتشابكة التي تم النظر فيها لا تشبه حل نقل المعلومات، ولكنها تشبه حل التنسيق المسبق. والدليل على ذلك أن هذه الطريقة لا يمكن تطبيقها دون أن يجتمع المجربان أولاً ويعدلان أنظمتهما. ومن الممكن أيضًا تحديد كمية المعلومات التي يحتاجون إليها للتنسيق فيما بينهم مسبقًا والتأكد من أنها لا تقل عن كمية المعلومات التي يتعين عليهم نقلها لبعضهم البعض لحل اللغز.

    هذا كل ما سأقوله عنه. أخبرتك أن هذا ليس وقت القصة حيث يمكنك استحضار أي ذكرى من الماضي تتبادر إلى ذهنك وتتوقع مني أن أستمتع بها. السبب الوحيد الذي جعلني أجيب على سؤالك هو أنني صدمت من مستوى فهمك للقراءة ("أنت تقول أن الفرق الوحيد بين كيلو من الحديد وأطنان من الحديد هو الوزن؟ إذن أنت تقول في الأساس أنه لا يوجد فرق بين" هم؟!")….

    فيما يتعلق بالفوتون، يؤسفني أن أخبرك - فأنت مازلت لا تفهم. أولًا، لمجرد أن لديك جهاز قياس يُظهر طولًا موجيًا معينًا، لا يعني أن الفوتون له طول موجي معين. كالعادة، سأشرح أنني لا أقصد إهانتك، لكن يجب أن أكرر الجملة الثابتة - أنت ببساطة لا تفهم المبادئ الأساسية لميكانيكا الكم. حتى الفوتون الموجود في حالة تراكب لعدد لا نهائي من الترددات المختلفة سيُظهر لك ترددًا واحدًا محددًا عند قياسه. أنت ببساطة لا تدرك أن الحالة الكمومية للجسيم ونتائج القياسات التي تم الحصول عليها هي أشياء مختلفة. هذا في الواقع هو كل ما يتعلق بميكانيكا الكم على قدم واحدة - في الميكانيكا الكلاسيكية يتم وصف الجسيم بشكل فريد وكامل من خلال مجموعة نتائج القياس عليه، في حين يتم إساءة تفسير الكم. قد يكون هناك جسيمان مختلفان لهما فيزياء مختلفة تمامًا مما سيعطي نتائج القياس نفسها تمامًا.

    إنه حقًا أبسط شيء في العالم، وهو تمامًا كما قلت من قبل: لقد تم إنشاؤه في الفضاء بأكمله. سؤالك "أين يصنع" هو مثل السؤال عن لون شعر الأصلع أو كم متر في الكيلوغرام. هو سؤال هراء ينبع من التفكير الكلاسيكي الذي هو خاطئ بشكل واضح في ميكانيكا الكم. منذ لحظة انبعاث الفوتون، فإنه يوجد في جميع أنحاء الفضاء. لم يتم إنشاؤه بالقرب من المصباح اليدوي (بالطبع أنا أتحدث عن الفوتون بتردد محدد جيدًا، وليس المصابيح الكهربائية الموجودة في المختبر والتي تولد الفوتونات التي هي عبارة عن حزم موجية). بمجرد قياس موضعه، فإنك تنهار طوله الموجي. يضمن هذا الخطاف أنك إذا قمت بقياس موقعه مرة أخرى، أي مرة ثانية، بعد ثانية واحدة، فسوف تحصل على نتيجة تبعد بالضبط ثانية ضوئية واحدة عن المكان الأول الذي وجدته فيه. إذا كنت تريد حقًا فهم هذه الأشياء، فيجب عليك أن تتعلم مقدمة عن الكم، وتحديدًا أن تتعلم ما هو القياس وما هي الحالة الكمومية وما هو الفرق بينهما.

    فيما يتعلق بالاحتمال، كلاكما على حق إلى حد ما. إن الجسيم الذي له زخم محدد جيدًا لديه بالفعل احتمالية متساوية للعثور عليه في أي نقطة في الفضاء. نسيم محق في قوله إن مثل هذا التوزيع الموحد غير ممكن في الفضاء اللانهائي، أو بالأحرى - لا يمكن تطبيعه. هذا صحيح تمامًا، وبالفعل في ميكانيكا الكم في الفضاء اللانهائي لن تكون هناك جسيمات ذات زخم محدد جيدًا (أي أن العناصر الذاتية لمشغل الزخم المستمر ليست في فضاء هيلبرت، ولكن في الفضاء المساعد الذي يتضمن مساحة هيلبرت وهي ما يسمى بالأساس الكامل للحالات). ولهذا السبب حرصت على القول إنه لا يمكن خلقها إلا بشكل تقريبي - فهي كائنات مادية حقيقية فقط في مساحة محدودة. وما زلنا نتحدث عنها في ميكانيكا الكم لعدة أسباب: أولاً، إنها ليست فيزيائية حقًا ولكنها أقرب ما تكون إلى الحالات الفيزيائية الحقيقية. ثانيًا، يمكن بناء أي حالة فيزيائية منها، وبالتالي فإن فهم فيزياءها يجعل من الممكن فهم فيزياء أي حالة فيزيائية حقيقية. ثالثًا، يمنحوننا حدسًا جيدًا لأن حساباتهم الرياضية أبسط من حسابات الرزم الموجية.

    يا إسرائيل، أنتم تكررون نفس الأخطاء مراراً وتكراراً. قلت سابقًا أن كل فوتون له زخم معين. لقد شرحت لك أن هذا ليس صحيحًا كميًا. والآن أنت تقول أن الشيء الوحيد الذي يميز الفوتونات هو الزخم. حسنًا، لقد غيرت بعض الكلمات ولكنك مازلت مصرًا على تجاهل ميكانيكا الكم تمامًا والتعامل فقط مع الفيزياء الكلاسيكية، حيث يكون لكل فوتون مسار يحدد الموقع والزخم بشكل فريد. في حالة أن الأمر لا يزال غير واضح - لا، فالفرق بين الفوتونات ليس فقط في ترددها. معظم الفوتونات ليس لها تردد، كما سبق أن شرحنا لك عدة مرات. الفرق بين الفوتونات يكمن في حالتها الكمومية (والتي تتضمن توزيعاتها الترددية المحتملة). نأسف لأن يبدو الأمر وكأننا أسطوانة مكسورة، ولكن إذا كنت تريد أن تفهم حقًا أيًا من هذه الأشياء، فعليك أن تدرس مقدمة عن الكم. قد تظن أنني أريد إذلالك فقط وأنني أستمتع بإخبارك أنك لا تفهم ولا تعرف، لكني أؤكد لك أن الأمر ليس كذلك. تذكر أنني أقوم بالتدريس كجزء من مهنتي وأن لدي غرائز المعلم. أنا حقًا أخبرك بهذه الأشياء لتساعدك على الفهم - لديك فجوة كبيرة في معظم الأفكار الأساسية لميكانيكا الكم (ما الذي يحدد الجسيم في ميكانيكا الكم؟ ما الفرق بين وصف الجسيم المادي والوصف الذي نراه) عندما نقيسه؟ ماذا يفعل القياس بالجسيم؟ ما العلاقة بين الزخم والموضع؟ إلخ. إلخ.). بدون استكمال هذه الفجوات، لن تتمكن من فهم الحلول لأسئلتك، ولن يؤدي إلا إلى الغرق بشكل أعمق وأعمق في سوء فهمك.

  107. إلبينتزو

    أرجو الإجابة على السؤال الذي طرحته عليك سابقا.

    لقد كتبت "لا يوجد فرق بين هذا الحل وأي حل آخر للكود الكلاسيكي المنسق مسبقًا (كما حاول بعض المعلقين هنا اقتراحه)، باستثناء أن هذا الكود يعمل وفقًا لقوانين الكم، وعلى وجه الخصوص وجد في التراكب"

    هل تتضمن قوانين الكم والتراكب إمكانية التأثير في الزمن الصفري من أحد طرفي التجربة إلى الطرف الآخر، وهو أمر غير موجود في الكود الكلاسيكي؟ هذا هو أصل المشكلة، أليس كذلك؟

    الموضوع مهم لأن هناك خيارين:

    1. الجواب إيجابي، أي أن قوانين الكم والتراكب تتضمن إمكانية التأثير في الزمن الصفري من أحد طرفي التجربة إلى الطرف الآخر، وهو أمر غير موجود في الكود الكلاسيكي، ثم في الحقيقة هناك هناك فرق كبير وجوهري بين هذا الحل وأي حل آخر للكود الكلاسيكي المنسق مسبقًا (كما يقترح بعض المشاركين هنا).

    2. الجواب بالنفي، فلا فرق في الواقع بين الخيارين.

    إذا كان الخيار 1 - فأنت تقول في الأساس أنه لا يوجد فرق بين النظام الذي يسمح بعدم المحلية والنظام الذي لا يسمح بذلك؟

    وإذا كان الجواب 2 فالوضع أسوأ. ألا ترى الفرق الواضح بين الحالتين؟

    وعلى أية حال، إذا قلت إنه لا يوجد فرق - فلماذا تغضب وتصفني بالكذاب والكذاب عندما أقول لك أن تحاول إنشاء مثل هذا الكود الكلاسيكي؟ نحن نعلم أن الكم يعمل، لذا إذا لم يكن هناك فرق كما تقول، فلماذا لا تحاول تقديم مثل هذا النظام؟

    يبدو لي، مثل العديد من الحالات في الماضي، أنك ببساطة لم تفهم ما كنت أقوله وهاجمتني على الفور.

  108. ما العيب في الاحتمال المتساوي؟

    الميونات ليس لها علاقة بالفوتونات. الفوتونات هي حيوانات مختلفة في غابة الجسيمات، لأنها الوحيدة (على حد علمي) التي تخضع للمسلمة 2.

    يتم إطالة عمر الميونات أو تقصير المسافة التي تقطعها في طريقها إلى الأرض بسبب إطالة الزمن أو تقصير الطول حسب النسبية. يحدث هذا لأي عظم، وخاصة عظام التوائم.

    لكن هذا لا علاقة له بالمسألة المطروحة، إلا إذا رأيت أنها دليل قاطع على النسبية.

    المسألة المطروحة أمامنا هي: إذا كان لفوتون معين زخم محدد جيدًا، أي أن عدم اليقين في زخمه هو 0، فهناك عدم يقين لا نهائي في موضعه، أي أنه سوف ينتشر في جميع أنحاء الفضاء، ألا يتطلب هذا أن تكون جميع الفوتونات هم من هذا القبيل؟ ففي نهاية المطاف، الشيء الوحيد الذي يفرق بين الفوتونات هو طولها الموجي، فلماذا يختلف جزء الفوتون لدينا عن بقية إخوته؟ ما هو المميز فيه أو ما هو المميز فيها؟

    وإذا كان كل فوتون منتشرًا باحتمالية متساوية في جميع أنحاء الفضاء، فلماذا نقيسه فقط على مسافة ط م من المصدر؟

    أعتقد أن السبب قد يكون بسبب أن أدوات القياس لدينا لا يمكنها قياس الأجزاء السريعة أو البطيئة من الفوتون من c، ولكن فقط الجزء الذي يتحرك بالنسبة لها تمامًا عند c، ولكن قد يكون هناك سبب آخر.

  109. إسرائيل
    عمر الميون أقصر من أن يمر عبر الغلاف الجوي. ومن وجهة نظر الراصد على الأرض، نرى أن زمن الميون يطول، مما يعني أن عمره الافتراضي يزداد.

    لكن بالنسبة له، يجب تقصير الطريق عبر الغلاف الجوي.

  110. إسرائيل
    لا يبدو لي أنك تفهم ما هو التوزيع الموحد (أفترض أن هذا ما تعنيه).
    مثل هذا التوزيع لا يمكن أن يوجد إلا في نطاق محدود.
    تعلمنا هذا في المدرسة الثانوية ...

  111. كما هو مكتوب:

    "إن عدم اليقين في الزخم هو 0. نعم، في هذه الحالة سيكون هناك عدم يقين لا نهائي في الموقع، مما يعني أنه سوف ينتشر في كل الفضاء."

    وما لم يُكتب إلا أنني أعتقد أن ألباتيزو سيؤكد قريبًا:

    مع احتمال متساو.

  112. إسرائيل
    كتبت سابقًا - هناك احتمال كبير جدًا للعثور عليه حيث تتوقع، واحتمال أقل في البيئة القريبة جدًا من هذه النقطة، واحتمال منخفض جدًا جدًا للعثور عليه على مسافة أكبر (أعتقد أن الاحتمال أسي ).

    كيف وصلت إلى احتمال متساو؟

  113. دعونا نرى ما كتبه ألبانتيزو:

    لنفترض أنني أنظر إلى فوتون معين لديه زخم محدد جيدًا، مما يعني أن عدم اليقين في الزخم هو 0. نعم، في هذه الحالة سيكون لديه عدم يقين لا نهائي في موضعه، مما يعني أنه سينتشر في جميع أنحاء الفضاء.

    هو، هو الذي أحبه. ليس من الواضح بالنسبة لي: أين تم إنشاؤه؟ هل يمكنني إنشائه في سانتا مونيكا؟ لأنه إذا كان الأمر كذلك، ولا أرى سببًا لذلك، فبالرغم من عدم اليقين اللامتناهي في الفضاء والاحتمال المتساوي لتواجده في كل مكان، كن مطمئنًا أنك على بعد ثانية ضوئية فقط سوف تحصل عليه بعد ثانية واحدة من خلقه.

    فأين اختفت رفاته متناثرة بنفس الاحتمالية في الفضاء؟

    ما هي مشكلة الميونات؟ تمديد الوقت؟

  114. إسرائيل
    ليس صحيحاً أنك سوف تحصل عليه في كل مكان - هناك احتمال مؤكد أنك سوف تحصل عليه في كل مكان، وهذا شيء مختلف تماماً. احتمال حصولك عليه على مسافة c*t بعد الوقت t مرتفع جدًا، واحتمال تلقيه في المنطقة المجاورة لهذه النقطة صغير جدًا.

    بالمناسبة - كيف يمكنك حل مشكلة الميون؟ كيف يصلون إلى الأرض؟ فهي ليست فوتونات ولا تتحرك بسرعة ج.

  115. يبعث الليزر الموجود في سانتا مونيكا فوتونًا أخضر أحادي اللون بطول موجة يبلغ 507.65789654346 نانومتر. وهذا ما يظهره جهاز القياس.

    فإذا خلق في الكون كله ففي أي نقطة في الكون بعد ثانية سنمتصه في أي نقطة أخرى. نظرًا لأن المكان الوحيد الذي تم التقاطه فيه هو على بعد ثانية ضوئية من سانتا مونيكا، فمن المحتمل أن يكون هذا هو المكان الذي تم إنشاؤه فيه.

    وأنت لم تفهم المثال المطاطي. فكر فقط في 100 حجر تتحرك بسرعة تتراوح من 1 إلى 100 متر في الثانية. إذا كنت جنديًا في MGB في المناطق التي تتحرك فيها الحجارة في اتجاهي مثل وابل الصواريخ، هل يمكنك إخباري بسرعة الوابل؟

    المعجزات ليست مشكلة، بل هي الحل.

  116. إسرائيل
    وحتى ذلك الحين، لم أكن لأستثمر المال في تلك الفوتونات.
    لكن لنفترض ذلك - ما المشكلة هنا؟ في الإلكترونيات اليومية، يتم استخدام المكونات القائمة على مثل هذه الظواهر (يمكنك شراؤها من أمازون).

  117. إذا كنت في لوس أنجلوس وقمت بتشغيل ضوء ينتج عنه فوتونًا جديدًا، وكان هذا الفوتون *فوتونًا له زخم محدد جيدًا* (أي له طول موجي محدد وليس حزمة موجية) فهذا يعني أنه لم يتم إنشاؤه في لوس أنجلوس انجليس. فهو مخلوق بالتساوي في كل نقطة من الكون، أي في التراكب. مرحبًا بك في ميكانيكا الكم، أنصحك بإعادة الدورة التمهيدية من السنة الثانية. وكما سبق أن أخبرتك عدة مرات، فإن حدسك بأن الفوتون يخرج من المصباح اليدوي وله مسار (شعاع) يمكن اتباعه مباشرة ينبع من حقيقة أن المصابيح الكهربائية التي تعرفها لا تنتج فوتونات بتردد محدد، ولكن إنتاج حزم موجية تحتوي على سلسلة من الترددات.

    ولا، حتى في العالم الكلاسيكي، لا يمكن تعريف الجسم المطاطي على أنه سرعة إلا إذا كان جسمًا صلبًا، أي أن الأجزاء التي يتكون منها تتحرك معًا. يمكنك تكرارها بقدر ما تريد، فهذا لا يجعلها صحيحة. يتحرك كل جزء من المطاط بسرعة واحدة، ولأن السرعات غير متطابقة مع بعضها البعض، يحدث توتر في المطاط.

  118. ولأنني لم أرسل فوتونًا واحدًا، فقد أرسلت الفيلم بأكمله "ذهب مع الريح" وحصلت عليه في أندروميدا.

    من الممكن أن هذه ليست هي نفس الفوتونات، ولكن يمكن قول الشيء نفسه عن فوتون المصباح الموجود.

    هل أوافق من فضلك إيراف كيف أعرف؟ أقول إنه على الأقل محتمل مثل البديل: عوالم متعددة وتأثير على الماضي. عليك أن تقوم بالتجربة والتحقق.

  119. إسرائيل
    نعم - ولكن هل توافق على ذلك؟

    بالمناسبة - الفوتون الخاص بك لن يكون في أندروميدا بعد 10 ثواني. لنفترض أننا قمنا بالتجربة. وضعت كاشفين واحد في أندروميدا والآخر في مكراه على بعد 3 مليون كيلومتر. وبعد 10 ثوانٍ، لم تقم بتسجيل فوتون في أقرب كاشف، بل قمت بتسجيل فوتون في مجرة ​​المرأة المسلسلة.
    كيف تعرف أنه نفس الفوتون؟

  120. وفيما يتعلق بعدم وجود اتصالات.

    ما كتبته هو الصحيح. في الجهاز الذي وصفته بأجهزة الراديو والعملات المعدنية، يشير nc فقط إلى الجزء الثاني، وهو إرسال المعلومات عبر الجهاز وهو أمر مستحيل. ولا يتناول الجزء الأول، أي كيفية إجراء الاتصال بين أجهزة الراديو نفسها.

  121. من

    https://www.hayadan.org.il/is-the-universe-ramdom-0405168/comment-page-6/#comment-707429

    "أخيرًا، فيما يتعلق بتعليقي بأن جميع المعلومات موجودة بالفعل في النظام. إذًا لقد بدأنا بـ 300 زوجًا مغزليًا متشابكًا، أليس كذلك؟ لكن مهلا، كيف حصلنا عليهم؟ وهل يكفي أن نعرف أنهما متشابكان؟ يجب أن يتم إنتاج كل زوج عن طريق القياس المحلي. أي أنه قبل بدء التجربة، كان على المجربين (واحد من KDA والآخر من المريخ) أن يجتمعوا في مكان ما وينتجوا الأزواج المتشابكة. ولكن هذا ليس كافيا. يجب أن يعرفوا بالضبط الحالة المتشابكة لديهم (لأنه حتى أصغر نظام يحتوي على 4 حالات متشابكة على الأقل)، وإلا فلن يمكن تعيين الكود. لذا، لا يتعين عليهم الاجتماع من قبل وإجراء القياس فحسب، بل يتعين عليهم أيضًا أن يعرفوا بالضبط القياس الذي أجروه وما هي عملية انهيار النظام. عندها فقط يمكن لكل منهم الذهاب إلى نقطة النهاية وإجراء التجربة. أي أنهم يأتون بكمية *ضخمة* من المعلومات المتطابقة مسبقًا، وكل ما يفعلونه أثناء التجربة هو طيها بقياس محلي، كل منها على حدة. لا يوجد فرق بين هذا الحل وأي حل آخر للكود الكلاسيكي المنسق مسبقًا (كما حاول بعض المعلقين هنا اقتراحه)، باستثناء أن هذا الكود يعمل وفقًا لقوانين الكم، وعلى وجه الخصوص فهو موجود في التراكب ( وكما تعلمون فإن سر سحر التشابك هو أنه حالة تراكب، وليس مجرد تراكب خاص وهو غير كروي إلى أنظمة فرعية خالصة).

    عندما تقول:

    "لا يوجد فرق بين هذا الحل وأي حل آخر للكود الكلاسيكي المنسق مسبقًا (كما حاول بعض المعلقين هنا اقتراحه)، باستثناء أن هذا الكود يعمل وفقًا لقوانين الكم، وعلى وجه الخصوص فهو موجود في التراكب "هل تتضمن قوانين الكم والتراكب إمكانية التأثير في زمن صفر محدود على أحد التجربة على الطرف الآخر، وهو أمر غير موجود في الكود الكلاسيكي؟ هذا هو أصل المشكلة، أليس كذلك؟

    "لا تشير المقالة إلى مشكلة في النموذج الرياضي. هذا بالضبط ما أقوله، لماذا لا تفهمين؟ يتحدثون في المقال عن توتر معين، لكن هذا التوتر موجود في فلسفة الفيزياء وليس في رياضياتها.

    لم أرى ذكر للنموذج الرياضي في المقال. إليك ما هو مكتوب في المقالة حول النموذج المادي:

    "لقد حطمت ميكانيكا الكم العديد من البديهيات، ولكن هذا هو الأعمق منها جميعًا. وهذا التحطم بالتحديد يحمل في أجنحته تهديدًا لم تتم إزالة ظلاله بعد، وهو تهديد للنسبية الخاصة، وهي إحدى الركائز الأساسية لفيزياء القرن الحادي والعشرين.

    إن الأمر الأكثر إثارة للقلق بشأن اللامكانية، بصرف النظر عن الغرابة الصارخة المتأصلة فيها، هو أن هذه الميزة تحمل في طياتها تهديدًا للنظرية النسبية الخاصة كما نعرفها اليوم. في السنوات الأخيرة، أصبح هذا الاهتمام، الذي دخل أخيرًا قاعة الأفكار الجادة في مجال الفيزياء، محورًا للمناقشات التي قد تؤدي في النهاية إلى وشم أو تشويه أو إعادة تصور أو ترسيخ أو تفكيك أسس العلم ذاتها. الفيزياء.

    وبالتالي فإن الأخبار السيئة لم تكن من نصيب ميكانيكا الكم، بل من مبدأ المحلية، وعلى أي حال، على ما يبدو، النسبية الخاصة، لأنها، على الأقل ظاهريًا، مبنية على افتراض المحلية.

    يبدو أن نوع اللامكانية الذي نواجهه في ميكانيكا الكم يتطلب التزامن المطلق، مما يشكل تهديدًا حقيقيًا وقاتلًا للنسبية الخاصة.

    إن حالة النسبية الخاصة، بعد ما يزيد قليلاً عن قرن من ظهورها على المسرح العالمي، أصبحت فجأة مسألة مفتوحة على مصراعيها وسريعة التطور.

    لذا فأنت تزعم أن عبارات مثل "تفكك أسس الفيزياء ذاتها" و"تهديد حقيقي ومميت للنظرية النسبية الخاصة" هي مجرد "توتر معين، ولكن هذا التوتر موجود في فلسفة الفيزياء وليس في رياضياتها" "؟

    "هذا مثال ممتاز على التحريف المجنون لكلماتي وحتى الاختراع: "هل تدعي أنهم في النص الفرعي كانوا يقصدون العكس بالفعل وأنه لا يوجد أي تهديد وأنهم على وفاق كبير وأن مؤلفي الكتاب مقال مفبرك شيئا خياليا لزيادة تداوله؟"

    ألم تكتب قبل أيام:

    "بادئ ذي بدء، عليك أن تتذكر أن كل مقال في مجلة - حتى لو كانت علمية، وبالتأكيد في مجلة ساينتفيك أمريكان، التي تخاطب عامة الناس ولديها اهتمام كبير ببيع النسخ - يمكن أن يقع في ميل طبيعي للإثارة أو عناوين منمقة."

    و أيضا:

    "في الختام، من الصعب جدًا بالنسبة لي أن أرى كيف يوجد في محتوى المقال أي خطر للجمع بين النسبية الخاصة وميكانيكا الكم."

    في الواقع لا يوجد خطر. "لتقويض أسس الفيزياء ذاتها" "تهديد حقيقي وقاتل للنظرية النسبية الخاصة". لقد شوهت واختلقت واخترعت، وكما هو الحال دائمًا كذبت وكذبت وكذبت مرة أخرى.

    في الحقيقة:

    "5. وكما كتبت سابقًا، فإن الفوتون ذو التردد المعين سيكون له بالفعل قدر لا نهائي من عدم اليقين في موضعه. تسأل عما إذا كان من الممكن العثور عليه على بعد عشر سنوات ضوئية من المكان الذي كان فيه قبل 10 ثوانٍ. يحتوي هذا السؤال على إجابة مكونة من جزأين: أولاً، قبل عشر ثوانٍ لم تكن تعرف حتى مكان وجوده. وهو فوتون له زخم معين، وموقعه غير معروف. تفترض أنك عرفت مكانه قبل عشر ثوانٍ، لكن هذا غير ممكن. تمامًا كما أنك الآن لا تعرف مكانه، لم تكن تعرفه أيضًا قبل عشر ثوانٍ».

    أنا في لوس أنجلوس وقمت بتشغيل مصباح يدوي وإنشاء فوتون جديد لم يكن موجودًا من قبل. وبعد 10 ثوانٍ، تم العثور على الفوتون أيضًا في مجرة ​​المرأة المسلسلة، التي تتوافق ساعاتها مع الأرض.

    إذن ماذا تعني عبارة "منذ عشر ثوانٍ لم تكن تعرف حتى أين كان"؟ وكان في إسرائيل حيث ولد. وإذا كان بعد 10 ثوانٍ في أندروميدا، فهو فوتون سريع جدًا، أليس كذلك؟

    "المطاط لا يتحرك بأي سرعة، هذا ما أشرحه لك. لا يوجد شيء مادي مثل "المطاط". أعني، من الواضح أنه يمكنك تعريفه (اتصال جميع الجزيئات التي يتكون منها) ولكن لا يمكنك تحديد سرعة له.

    نحن نتحدث عن عالم كلاسيكي، حيث يوجد مثل هذا الشيء المادي، المطاط. إذا قمت بإطلاق 10 أحجار وتحرك كل واحد منها بسرعة مختلفة عن صديقه، فإنها تتحرك بسرعة 10. وإذا كانت متصلة ببعضها البعض بالمطاط فإن أحجار الجسم + المطاط تتحرك بـ 10 سرعات + جميع السرعات بينهما. نفس الشيء مع 1000 تريليون حجر، حتى لا يتبقى لك في النهاية سوى المطاط الذي يتحرك بجميع السرعات من 0 إلى الأعلى.

    والمهم لأغراضنا أن مثل هذا الوصف الغريب للفوتون يحل أوصافًا لا تقل غرابة مثل التأثير على الماضي ويشرح أيضًا المسلمة 2.

    ليس عليك الرد إذا لم تكن مهتمًا، ليلة سعيدة.

    المعجزات

    هذا ما يدعيه كوانتوم، أليس كذلك؟

  122. 1. أفضّل عدم العودة إلى الشجارات القديمة وبالتأكيد لا أريد أن أبدأ بالحديث عن المحادثات السابقة. كل ما حدث هنا هو أنك وضعت رابط لمقال معين وأعطيت رأيي فيه. ثم كتبت شيئا عن الجمل المحظورة ورأيت أن ما كتبته غير صحيح ويمكن أن يضلل الآخرين أيضا، لذلك كتبت توضيحا. لم أكن أنوي في أي وقت من الأوقات وما زلت لا أنوي فتح أي نقاش أو قتال معك على الإطلاق. ولكن لا توجد أمور مطلقة في الحياة وقد ترغب في فتح محادثة قديمة لأنها في الواقع موضوع محادثة مثير للاهتمام وله صلة بشيء كتبته هنا. في هذه الحالة، من المحتمل جدًا أن أكون مقصّرًا وسأكون سعيدًا بتوضيح ما أقصده، كما فعلت عندما سألتني عن عدم اليقين في الفوتون (على الرغم من أنني سبق أن شرحت لك ذلك من قبل) . وفي الختام - وهذا واضح قدر الإمكان - أنا لست مهتماً بفتح مناقشات قديمة، لكني لا أضمن 100% أنك إذا طلبت مني توضيح مطالبة سابقة سأرفض. إذا كانت المطالبة مثيرة للاهتمام وتبدو ذات صلة بالنسبة لي أو أنك تستحق التوضيح، فسوف أقوم بالتوضيح. في هذه الحالة يجب أن تعطيني رابطًا كاملاً لما قلته والسياق الذي قيل فيه لأن هذه هي الطريقة الوحيدة التي يمكنني من خلالها شرح ما قصدته.

    2. لم يذكر المقال مشكلة في النموذج الرياضي. هذا بالضبط ما أقوله، لماذا لا تفهمين؟ يتحدث المقال عن توتر معين، لكن هذا التوتر موجود في فلسفة الفيزياء وليس في رياضياتها. هذا كل ما ادعيته في التعليق الأصلي الذي كتبته على المقال، وهذا ما أدعيه الآن. بالنسبة لي هناك فرق *كبير* بين مشكلة في النموذج النظري ومشكلة في التفسير الفلسفي. إذا كنت تفضل عدم إحداث فرق، فارحل. يمينك اعتقدت أنه من المهم جدًا التعليق على ذلك.

    إليكم مثال ممتاز على تحريف مجنون لكلامي وحتى اختراع: "هل تدعي أنهم في النص الفرعي كانوا يقصدون العكس بالفعل وأنه لا يوجد أي تهديد وأنهم على وفاق كبير وأن مؤلفي المقال ملفقة شيئًا خياليًا لزيادة التداول؟" أنت تعلم جيدًا أنني لم أقل شيئًا كهذا أبدًا. ولم أكتب أنهم اختلقوا أي شيء. ولم أكتب أنهم يقصدون العكس. أنا فقط أكرر - للأسف أنت لا تستطيع أن تفهم، ولكن هذا بينك وبين نفسك - أن المشكلة التي يطرحونها هي مشكلة في فلسفة الفيزياء، وأن المشكلة نفسها وحلها موجودان في تفسير كيف نحن فهم النموذج والواقع، وليست مشكلة في النموذج الرياضي نفسه.

    3. ماذا يعني "لماذا لا يكون لها طول موجي مطلق في نظام مرجعي معين"؟ لماذا 2+2 لا يساوي 9؟ سوف تتعلم ميكانيكا الكم. للفوتون حالة (أي موصوفة بواسطة متجه في فضاء هيلبرت). يمكن أن تكون هذه الحالة حالة ذاتية لمؤثر الزخم (في هذه الحالة، فإن أي قياس لزخم الفوتون في إطار مرجعي معين سيعطي دائمًا بعض النتائج، ونحن نسمي هذه النتيجة زخم الفوتون) ولكن يمكن أن تكون حالة الفوتون أيضًا حالة هذه ليست حالة ذاتية لمشغل الزخم. نظرًا لأن مشغل الزخم الهرمي وحالاته الذاتية عبارة عن مجموعة كاملة تمتد عبر فضاء هيلبرت، فسيكون من الممكن دائمًا تقديم هذه الحالة كتراكب لحالات الزخم الذاتية - أي أن الفوتون لن يكون له زخم محدد، ولكن كل قياس سيعطي زخمًا واحدًا بمثل هذا الاحتمال، وزخمًا ثانيًا باحتمال آخر، وهكذا. أنت ببساطة تصر على تجاهل ميكانيكا الكم والنظر إلى الصورة الكلاسيكية حيث يكون لكل جسيم زخم محدد جيدًا.

    فيما يتعلق بالفوتون الأخضر، فلنبدأ بحقيقة أنك ربما لم ترَ فوتونًا أخضر من قبل. عندما نرى فوتونًا "أخضرًا" في الواقع، فإننا نرى في الواقع فوتونًا في مجموعة متنوعة من الأطوال الموجية، لكن الفوتون الأخضر له السعة الأعلى بينها. وهذا يعني أن ما تسميه فوتونًا أخضر وتعتقد بشكل بديهي أن له طولًا موجيًا دقيقًا يتوافق مع اللون الأخضر هو عادةً مجرد حزمة موجية ضيقة نسبيًا تتمركز حول التردد الأخضر. الفوتون ذو التردد الأخضر هو بالضبط، كما قلت سابقًا، شيء يمكن إنشاؤه تقريبًا في المختبر ولا يوجد فعليًا في الطبيعة. لكن مرة أخرى - كما قلت سابقًا - أنا لا أدعي أنه لا يوجد شيء اسمه الفوتون الأخضر، وأنه لا يمكن أن يوجد. أنا فقط أقول أن هذا ليس صحيحا بشكل عام. إن القول بأن "الفوتون له طول موجي معين" يشبه القول "وفقًا للميكانيكا النيوتونية، يؤثر الجسم الضخم حوله بقوة جاذبية قدرها 3 نيوتن". بشكل عام هذا غير صحيح (القوة تعتمد على كتلة الجسم والمسافة منه) ولكن من الواضح أنه قد تكون هناك حالات تكون فيها قوة الجاذبية فعلا 3 نيوتن... هناك فوتونات خضراء. لكن القول بشكل عام بأن الفوتون له طول موجي معين هو ببساطة عدم فهم لأساسيات بداية مقدمة ميكانيكا الكم. آسف.

    5. كما كتبت سابقًا، فإن الفوتون ذو التردد المعين سيكون له بالفعل قدر لا نهائي من عدم اليقين في موضعه. تسأل عما إذا كان من الممكن العثور عليه على بعد عشر سنوات ضوئية من المكان الذي كان فيه قبل 10 ثوانٍ. يحتوي هذا السؤال على إجابة مكونة من جزأين: أولاً، قبل عشر ثوانٍ لم تكن تعرف حتى مكان وجوده. وهو فوتون له زخم معين، وموقعه غير معروف. تفترض أنك عرفت مكانه قبل عشر ثوانٍ، لكن هذا غير ممكن. تمامًا مثلما لا تعرف أين هو الآن، لم تكن تعرفه أيضًا قبل عشر ثوانٍ. تفترض أنه يمكنك معرفة أنه تم إخراجه من الفانوس وبالتالي كان هناك في الفانوس. لكن هذا ليس صحيحا كميا. إذا كان لديك مصباح يدوي يصدر فوتونات بزخم معين، فيمكنه إصدارها في أي نقطة في الفضاء - حتى بعيدًا جدًا عن نفسه. مرة أخرى، يرتبط هذا بالطبع بحقيقة أن المصابيح الكهربائية التي تعرفها من الحياة اليومية ببساطة لا تعمل بهذه الطريقة ولا تنبعث منها فوتونات ذات طول موجي محدد (وبالتالي من الممكن متابعة موضعها بشكل أو بآخر شعاع ضوء).

    الجزء الثاني من الإجابة يتعلق بكون ميكانيكا الكم هي نظرية إحصائية. الحالة الكمومية (أو موجة الفونون) لا تخضع لكل القوانين التي اعتدت عليها، وعلى عكس الفوتون الذي يتحرك بسرعة الضوء. ما يخضع لهذه القوانين هو *القياسات*. هناك بالطبع برهان رياضي مفاده أننا إذا قمنا بقياس موضع فوتون في لحظة معينة، وقمنا بقياس موضعه مرة أخرى بعد ثانية، فإن الفرق بين الموضعين سيكون ثانية ضوئية واحدة. ولكنها مرتبطة بعملية القياس التي تنهار فيها موجة هيجل. وهذا ليس صحيحًا بالنسبة للحالة الكمومية للفوتون، والتي يمكن أن تكون في حالة تراكب لجميع أنواع الحالات. عندما لا يتم قياسه، يكون الجسيم افتراضيًا وغير ملزم بالامتثال لمعادلات الحركة (وقد شرحت لك هذا أيضًا من قبل). في ميكانيكا الكم، تنطبق الظروف الفيزيائية على الأشياء التي نقيسها.

    6. المطاط لا يتحرك بأي سرعة، هذا ما أوضحه لكم. لا يوجد شيء مادي مثل "المطاط". أعني أنه من الواضح أنه يمكنك تعريفه (اتصال جميع الجزيئات التي يتكون منها) ولكن لا يمكنك تحديد سرعة له. تمامًا مثلما يمكنك تعريف كائن "Moshe and Danny"، لكن لا يمكنك ربط السرعة بهذا الكائن. فإذا كان داني واقفاً وموسى في مركبة فضائية تتحرك بسرعة قريبة من سرعة الضوء، فهذا لا يعني أن الجسم "موسى وداني" يتحرك بسرعة الضوء ويقف ساكناً في نفس الوقت. من الممكن تحديد السرعة لجسم ما فقط عندما تتحرك جميع الجسيمات التي يتكون منها معًا، وهذا ما يسمى بالجسم الصلب. نحن نعلم أنه في الواقع لا يوجد شيء اسمه جسم صلب، لكنه موجود في الفيزياء الكلاسيكية وهو تقريب جيد (على سبيل المثال، يمكنك أن تسأل عن مدى سرعة حركتي على الرغم من أنني مكون أيضًا من العديد من الجسيمات المختلفة. إن معاملة الإنسان كجسم جامد ليس دقيقًا ولكنه تقريب جيد جدًا على المقاييس اليومية التي ليست نسبية أو كمية).

    7. إذا قطعت المطاط فلن يقترب منك شيء. ببساطة عند نقطة معينة لن يكون هناك المزيد من القوى الداخلية في المطاط، أي التوتر، ونتيجة لذلك سيتوقف نصف المطاط عن التسارع (مرة أخرى، يوجد تفاعل متسلسل هنا لأنه لا يزال هناك توتر في الجزء المقطوع أيضًا) ). ليس لدي أي فكرة عما كنت بعد.

    يا إسرائيل، مازلت تتجاهل ما سألتك عنه، وتعاملني مرارًا وتكرارًا كمدرس خاص يمكنك فقط قصفه بالأسئلة وتوقع منه أن يعلمك ميكانيكا الكم أو النسبية. مع كل النية الطيبة، ليس لدي الوقت لذلك. إذا كتبت شيئًا (ردًا عليك أو على شخص آخر) وتريد التوضيح أو تريد أن تجادل بأنني مخطئ أو شيء من هذا القبيل، فهذا حقك. لكنني أعتقد أن وقت القصة هذا، حيث تجلس وتطرح علي أسئلة حول الفوتونات وعدم اليقين والترددات والتراكبات والأجسام الصلبة والمطاط وما الفرق بين الفيزياء والفلسفة وراء الفيزياء، قد انتهى.

    مساء الخير.

  123. لا تفهم.

    هل تريد مني إحضار روابط لما قلته في الماضي كما هو موضح ضمنيًا في عبارة "من فضلك أعطني رابطًا دقيقًا لما قلته والسياق" أم أنك لا تريد مني إحضار الروابط السابقة كما هو ضمني في عبارة "لا أريد" لا أعرف لماذا تعتقد أن لدي الوقت للذهاب معك للبحث عن معارك الماضي والانغماس فيها مرة أخرى".

    عرفتني بوضوح، أن قراءة الأفكار وإرسال الرسائل النصية لم تكن تعمل بشكل جيد مؤخرًا.

    حيث ورد في المقال "مشكلة في النموذج الرياضي الذي يحاول الجمع بين ميكانيكا الكم والنسبية الخاصة، أي من الاحتمالات الثلاثة موجود - هل نظرية المجال ليست نسبية كما أدعي، أليست كمية كما أدعي، أم هل هي مجرد نظرية غير متناسقة؟" وأزعم أنه لا يوجد أي تناقض على مستوى النموذج، بل وأعطيت مثالاً لنموذج يحتوي على النظريتين. الآن عليك أن تساعدني على فهم أنني مخطئ وأن النموذج لا يجمع بين الاثنين حقًا.

    أحضرت 4-5 اقتباسات من المقال بما في ذلك العنوان: التهديد الكمي للنسبية. هل تزعمون أنهم في الواقع يقصدون العكس وأنه لا يوجد تهديد وأنهم على ما يرام وأن مؤلفي المقال اختلقوا شيئًا خياليًا لزيادة التداول؟ وداعا، بصحة جيدة.

    لكن ليس هناك شك في أنهم، على الأقل في ظاهر الأمر، وبعبارات بسيطة، يقولون بالضبط ما أقوله، أليس كذلك؟ لذا اشرح جميع الاقتباسات التي قدمتها.

    "ليس للفوتون بالضرورة زخم مطلق" لماذا ليس له طول موجي مطلق في إطار مرجعي معين؟ إذا كان الأمر كذلك، فما الطول الموجي لفوتون أخضر معين؟ أليس زخم الفوتون يساوي قسمة ثابت بلانك المطلق على الطول الموجي للفوتون؟

    "ولكن دعونا نمضي قدما. لنفترض أنني أنظر إلى فوتون معين لديه زخم محدد جيدًا، مما يعني أن عدم اليقين في الزخم هو 0. نعم، في هذه الحالة سيكون لديه عدم يقين لا نهائي في موضعه، مما يعني أنه سينتشر في جميع أنحاء الفضاء. وماذا في ذلك؟ ليس من الواضح بالنسبة لي ما الذي تهدف إليه، أو لماذا اخترت الفوتون، لأن هذا ينطبق على كل شيء في إطار ميكانيكا الكم.

    جمال. لذلك دعونا نعمل مع هذا الفوتون بالتحديد. هل لديه عدم اليقين الكامل في مكانه؟ ويمكن أيضًا أن تكون على بعد ساعة ضوئية من المصدر الذي أطلقها قبل 10 ثوانٍ؟ فإذا كانت السرعة محددة بأنها المسافة مقسومة على الزمن، ألم نحصل على سرعة أعلى بكثير من سرعة الضوء؟

    "بشكل عام، إذا نظرت إلى المنشورات الأخيرة سترى أن هناك اتجاهًا قويًا هنا حيث تحاول فقط البحث عن المعارك" كيف حصلت على هذا؟ أنا لم ألجأ إليك منذ أكثر من عام، لكنك أنت الذي التفتت وتلجأ إلي.

    لكن دعونا نستمر بالطريقة الحالية. في الواقع بدأت تكون منتجة.

    "حقيقة أن كل جزء من المطاط يتحرك بسرعة مختلفة لا يعني أن" المطاط يتحرك بجميع السرعات ".

    إذن ما هي السرعة التي يتحرك بها المطاط؟ وماذا لو وضعنا علامة على كل أيون منه وقمنا بقياس سرعة ذلك الأيون، ألن نحصل على ما يقرب من 100 سرعة مختلفة تزداد تدريجياً من 0 إلى سرعة السهم؟

    وبما أن هذا مجرد مثال على الطريقة التي أرى بها الفوتون، فإذا قطعنا المطاط في مرحلة ما أثناء رحلة السهم ("القياس")، ألن "ينهار" المطاط مرة واحدة عندما تكون سرعة الانهيار من ترتيب سرعة المطاط؟ تذكر شيء؟

    ليلة سعيدة، 3 صباحا.

  124. وحقيقة أن كل جزء من المطاط يتحرك بسرعة مختلفة لا تعني أن "المطاط يتحرك بجميع السرعات". يعني يمكنك أن تقول ذلك إذا أردت، لكن ليس له أي معنى في الفيزياء لأن المطاط ليس جسمًا صلبًا وبالتالي لا يمكن أن تنسب إليه أي سرعة. يمكنك بالطبع تعيين سرعة متوسطة له، أو القول إن كل جزء آخر منه يتحرك بسرعة مختلفة قليلاً (أي التمدد). إذا ركضت بسرعة معينة، ركض نسيم أسرع قليلاً، وموشيه ركض أسرع وهكذا، فهل هذا يعني أن الشخص يتحرك بجميع السرعات؟ جسديًا، هذا بيان لا معنى له، على الرغم من أنه يمكن فهمه حرفيًا.

    توضيح: إذا كان المطاط مصنوعًا من سلسلة من الجزيئات، فلكل منها سرعة محددة واحدة (بالطبع، في الوقت الحالي نتحدث بشكل كلاسيكي). لا يوجد جسيم إلا ويتحرك بسرعة معينة. إن فكرة "التحرك بكل السرعات" هي مجرد نتيجة لأخذ العديد من الجزيئات المختلفة واتخاذ قرار بمعاملتها كجسم واحد. لكنهم ليسوا كذلك، وهذا هو بالضبط ما يسمى الجسم الصلب ونحن نعلم أنه غير ممكن جسديا - لا في المطاط ولا في أي جسم معقد آخر.

  125. 1. كالعادة، تتحدث في العناوين الرئيسية. في المقالة، لا يقول الكتّاب عبارة "هناك مشكلة" فحسب أثناء استمرارك في الاقتباس، بل يحددون أيضًا ما يعتقدون أن المشكلة هي. توقف، توقف للحظة لتقرأ عن الجانبين الإشكاليين اللذين يطرحانهما، وتحقق من ما يثير الإشكال فيهما. توقف عن الاقتباس الأعمى لكلمات "هناك مشكلة" فقط لأنها تناسب جدول أعمالك، وتعرف على ما هي المشكلة بالضبط. قضيت بعض الوقت في الموضوع (ليس فقط قراءة المقال نفسه ولكن أيضًا البحث في المواضيع المعروضة فيه في المجلات المتخصصة وعلى الإنترنت) وأدعي أن المشكلة فلسفية تمامًا ولا تنعكس في النموذج الرياضي. ويتحدث المؤلفون أيضًا عن حل محتمل من خلال اعتماد تفسير (لفضاء هيلبرت باعتباره الفضاء الذي نعيش فيه بدلاً من الزمكان). بدلًا من المطالبة المستمرة بأن أجيبك وأشرح لك، ربما تكون أنت من يشرح لمرة واحدة: اشرح لي إذا كانت هناك مشكلة في النموذج الرياضي الذي يحاول الجمع بين ميكانيكا الكم والنسبية الخاصة، أي من الثلاثة الاحتمالات موجودة - هل نظرية المجال ليست نسبية كما أدعي، أم أنها ليست كمومية كما أدعي، أم أنها مجرد نظرية غير متسقة؟ وأزعم أنه لا يوجد أي تناقض على مستوى النموذج، بل وأعطيت مثالاً لنموذج يحتوي على النظريتين. الآن يجب أن تساعدني على فهم أنني مخطئ وأن النموذج لا يجمع بين الاثنين حقًا.

    2. ليس لدي أي فكرة عما تتحدث عنه. لا أعتقد أنني تفلسفت هنا من قبل، فأنا أتحدث فقط عن الفيزياء. مرة أخرى، قسم كامل بدون محتوى، أنت فقط تبحث عن القتال.

    3. لا أستطيع أن أفهم ما لا تفهمه. الآن قمت بإضافة عبارة "لقد قلت دائما". حسنا. سأقولها مرة أخرى، ربما بشكل أبطأ قليلاً - إذا كنت تريد مني أن أشرح ادعاءً معينًا من المفترض أنني قدمته، من فضلك أعطني رابطًا دقيقًا لما قلته والسياق. الأمر لا يتعلق بالفيزياء على الإطلاق، بل يتعلق باللياقة الأساسية فقط. عندما يُطلب من شخص ما تفسير ادعاء ما، يجب أن يُمنح الفرصة لفهم ماهية الادعاء بالضبط، ومتى تم تقديمه وفي أي سياق. إذا أعطيتني رابطًا محددًا لمناقشة قديمة، سأحاول تخصيص وقت لقراءتها والشرح مرة أخرى. على الرغم من أنني آمل أن تفهم، فإن القتال معك يحتل مرتبة منخفضة جدًا في قائمة أولوياتي. لا أعرف لماذا تعتقد أن لدي الوقت للذهاب معك للبحث عن المعارك الماضية والانغماس فيها مرة أخرى.

    4. ليس لدى الفوتون بالضرورة كمية حركة مطلقة. كل جسيم (بما في ذلك الفوتون، ولكن أيضًا الإلكترون، وما إلى ذلك) في ميكانيكا الكم يكون في حالة كمية معينة. يمكن أن تمثل هذه الحالة جسيمًا له زخم محدد جيدًا (على المستوى الرياضي - الحالة عبارة عن حالة ذاتية لمشغل الزخم)، ولكن ليس من الضروري أن تكون كذلك. يمكن أن يكون الجسيم في حالة تراكب من زخمين، أو في أي حزمة موجية لا نهائية تحتوي على طيف من زخم مختلف. وهذا ينطبق أيضًا على الفوتونات. في الواقع، الفوتونات ذات الزخم الدقيق هي شيء لا يوجد إلا تقريبًا في المختبر، وجميع الفوتونات التي تواجهها في الحياة اليومية هي في الواقع حزم موجية ليس لها زخم محدد.

    ولكن دعونا نمضي قدما. لنفترض أنني أنظر إلى فوتون معين لديه زخم محدد جيدًا، مما يعني أن عدم اليقين في الزخم هو 0. نعم، في هذه الحالة سيكون لديه عدم يقين لا نهائي في موضعه، مما يعني أنه سينتشر في جميع أنحاء الفضاء. وماذا في ذلك؟ ليس من الواضح بالنسبة لي ما الذي تهدف إليه، أو لماذا اخترت الفوتون، لأن هذا ينطبق على كل شيء ضمن إطار ميكانيكا الكم.

    بشكل عام، إذا نظرت إلى المنشورات الأخيرة، سترى أن هناك اتجاهًا قويًا هنا حيث تحاول فقط البحث عن المعارك، وتسألني أسئلة غامضة دون توضيح ما تريد (تكتيك معروف لمحاولة ضرب شخص ما) أسفل) وليس من الواضح أين ظهروا فجأة، وليس في المعارك الماضية. وفي الوقت نفسه، أنت تتجاهل ما أكتبه، ولا سيما فيما يتعلق بوجود نموذج رياضي كامل ومتسق، كمي ونسبوي في نفس الوقت. لقد أصبح الأمر مملًا حقًا.

  126. المعجزات

    الفوتونات فريدة من نوعها من حيث أنها تتحرك بسرعة c بالنسبة إلى كل شيء. الآن دعونا نتحدث عن الفوتونات لأن النسبية تدور حولهم.

    ما مدى سرعة تحرك السهم؟ والجزء المتعلق بالشجرة؟ والوسط بينهما عازل؟

    يتحرك كل جزء من المطاط بسرعة مختلفة (أليس كذلك؟) وبالتالي يتحرك المطاط بجميع السرعات.

  127. إسرائيل
    التشابك لا يقتصر فقط بين الفوتونات. يمكن أن تكون الجسيمات إلكترونات وجزيئات مزدحمة وحتى ماسًا صغيرًا. هل فكرتك تغطي هذه أيضًا؟

    وأما المطاط: فلا يوجد فيه جسيم واحد له أكثر من سرعة واحدة.
    إذا كان لجسيم ما سرعة متجهة، فإن له زخمًا لا نهائيًا، فكيف يحدث ذلك؟ وأيضا الطاقة اللانهائية...

  128. المعجزات

    فكر في مطاط طويل مربوط بشجرة من جهة ومتصل بالضغط من جهة أخرى.

    أنت تطلق السهم. ما مدى سرعة تحرك المطاط؟ ليس من 0 إلى سرعة السهم الخطي؟

    أنا لا أقول أن هذا ما يحدث، لكن مثل هذا التفسير للفوتون يمكن أن يفسر العديد من الظواهر الغريبة، بما في ذلك اللامكانية، والمسلمة 2 ومفارقة التأثير على الماضي.

  129. 1. "تعرض المقالة توترًا معينًا على المستوى الفلسفي بين النسبية الخاصة وميكانيكا الكم. ويوضح أن هذا التوتر لا يؤدي بالضرورة إلى التناقض، ويوضح أنه يمكن حله على مستوى التأويل - أي في الفلسفة. وليس هناك ذكر للتناقض الجسدي (أي التناقض في النماذج)'.

    من المقال:

    "الشيء الأكثر إثارة للقلق بشأن اللامكانية، بصرف النظر عن الغرابة الصارخة المتأصلة فيها، هو أن هذه الميزة تحمل تهديدًا كبيرًا للنسبية الخاصة كما نعرفها اليوم. في السنوات الأخيرة، أصبح هذا الاهتمام، الذي دخل أخيرًا قاعة الأفكار الجادة في مجال الفيزياء، محورًا للمناقشات التي قد تؤدي في النهاية إلى وشم أو تشويه أو إعادة تصور أو ترسيخ أو تفكيك أسس العلم ذاتها. الفيزياء.'

    لا فلسفة، فقط فيزياء.

    "إن حالة النسبية الخاصة، بعد ما يزيد قليلاً عن قرن من ظهورها على المسرح العالمي، أصبحت فجأة مسألة مفتوحة على مصراعيها وسريعة التطور. لقد نشأ هذا الوضع لأن الفيزيائيين والفلاسفة اتبعوا أخيرًا النهايات الصريحة لذلك النقاش المنسي بين أينشتاين وميكانيكا الكم.

    صحيح أنهم فلاسفة (وماذا في ذلك؟)، ولكن أيضًا فيزيائيون.

    إذن ما الفرق بين ما يقوله المقال وما أقوله؟

    2. "من الواضح أن القياس على جانب واحد يؤثر على الجانب الآخر أيضًا."

    إذن، ما كل هذه الضجة حول القفازات والأرقام التي تم كتابتها وإرسالها إلى أطراف مختلفة؟ هل هناك أي طريقة يمكن أن يؤثر بها القياس أو أي عملية أخرى يتم إجراؤها على قفاز أو على رقم مكتوب على مذكرة بطريقة أو بأخرى على الجانب الآخر؟

    وما الضجة في كذبي بتحريف كلامك؟ ألم تقل مرة أخرى وتقول إنه لا يوجد فرق بين الكود الكلاسيكي والكود الكمي إلا أنه في الكود الكلاسيكي يمكنك القيام بأشياء مختلفة؟ هل تشمل هذه الأشياء المختلفة التأثير عن بعد في زمن الصفر؟ أليس هذا هو أصل الخلاف؟

    لقد قلت مرات عديدة أنه إذا كنت توافق على أن أحد الجانبين في التشابك يؤثر على الجانب الآخر في وقت صفر، فلا داعي لتسميته معلومات، فيمكنك تسميته موشيه.

    3. لقد زعمت دائمًا أنه لا يتم تمرير أي معلومات لأن جميع البيانات موجودة بالفعل في النظام.

    حتى لا تدعي أنني أحاول خذلانك، في تجربة Aspect، لا توجد جميع البيانات في النظام، ويتم تحديد حالة المستقطبات بعد التشابك.

    4. يجب تشغيل (البوكر!) والموضوع مهم وسوف نتوسع فيه لاحقا، ولكن هنا هو الجوهر:

    ففي النهاية، يتمتع الفوتون بزخم واضح ومطلق. ألا يتطلب هذا غيابًا تامًا للموقع؟

  130. 1. يعرض المقال توترًا معينًا على المستوى الفلسفي بين النسبية الخاصة وميكانيكا الكم. ويوضح أن هذا التوتر لا يؤدي بالضرورة إلى التناقض، ويوضح أنه يمكن حله على مستوى التأويل - أي في الفلسفة. ولا يوجد ذكر للتناقض الجسدي (أي التناقض في النماذج). أفهم أنك تسيء تمثيل نفسك من خلال الاقتباس الانتقائي لأشياء مثل العنوان وتجاهل نص المقالة. لكن الحقيقة هي أنني لا أفهم تمامًا ما الذي تريده مني لأنني لا أعتقد أنني اتهمتك بأي شيء (باستثناء الاستنتاج الذي يشير إلى أن المقال يشير إلى تناقض بين النسبية الخاصة وميكانيكا الكم). الادعاءات التي قدمتها كانت حول المقال وليس حول كلماتك. إذا كان هناك شيء محدد قلته يتعلق بكلامك وتريد مني أن أشرحه أو تزعم أنه خطأ، فحدد ما هو وسأتناوله. في هذه اللحظة يبدو لي أنك فقط تبحث عن قتال آخر بالقوة أو طلب عام أجده بقوة ما نختلف عليه.

    2. لماذا؟ يؤدي قياس الحالة المتشابكة إلى انهيار الحالة على جميع أجزاء النظام. ومن الواضح أن القياس على جانب واحد يؤثر أيضًا على الجانب الآخر. أنا فقط أحاول أن أشرح لك أن هذا التأثير ليس تعبيراً عن نقل المعلومات. إنه يعبر عن الارتباط، لكن هذين شيئين مختلفين. ليس لدي أي فكرة من أين حصلت على فكرة أنني أنكر واحدة من أكثر ظواهر ميكانيكا الكم شهرة ودراسة في القرن العشرين. يمكنك أيضًا أن تسألني إذا كنت أنكر أنه يمكن العثور على الجسيم في مكانين مختلفين في نفس الوقت.

    3. كما كتبت لك بالفعل، ليس لدي أي فكرة عما تقصده. إذا كنت تريد مني أن أتناول مطالبة قدمتها من قبل، فمن العدل أن تقدم لي المطالبة بما في ذلك السياق الكامل الذي تم تقديمها فيه، وسيكون من دواعي سروري أن أحاول توضيحها. إن إعطائي اقتباسًا من أربع كلمات وإخباري بأن الأمر يتعلق "بتجربة تجميع"، ومن ثم توقع أن أكون قادرًا على فهم موضوع الأمر وشرحه لك، هو إما مزحة سيئة أو محاولة متعمدة لإحباطي.

    4. بالطبع ينطبق مبدأ عدم اليقين على الفوتونات. في الواقع، لقد سألتني هذا بالفعل من قبل وقد شرحته لك بالفعل. ولكن أود أن أشرح مرة أخرى.

    إن ميكانيكا الكم كما كتبت في العشرينيات والثلاثينيات من القرن الماضي (جذورها أقدم بالطبع ولكن هذه هي سنوات النمو الأساسي) ليست نظرية نسبية. في هذه النظرية، ترتبط السرعات بالتحويل الجليلي، حيث يؤدي التغير في الجهد إلى تغير فوري في تسارع جميع الأجسام في الفضاء، وما إلى ذلك. أي أنها تناقضت مع علاقتها الخاصة.

    وبالمثل، كانت النسبية الخاصة كما كتبت عام 1905 امتدادًا للميكانيكا الكلاسيكية. على وجه الخصوص، مواقع الجسيمات وزخمها محدودة ويمكن معرفتها (لست بحاجة إلى النظر إلى الفوتونات، يمكنك النظر فقط إلى جسيم ضخم من المادة). لا يمكن العثور على الجسيمات في مكانين في نفس الوقت، ولا يمكن أن تؤدي إلى التشابك، وما إلى ذلك. هذه النظرية ليست كمية وتتناقض مع ميكانيكا الكم.

    ولكن منذ وقت طويل بالفعل - ما يقرب من قرن من الزمان - أدرك الناس أن هذه المشاكل هي مشاكل في صياغة النظريات. من الممكن تطبيق مبادئ النسبية الخاصة على مراسيها في نظرية حيث الموقع والزخم ليسا وضعيين ولكنهما عوامل غير مؤكدة. يمكن تطبيق جميع مبادئ ميكانيكا الكم دون استثناء في نظرية لا يتم فيها ربط السرعات بطريقة مباشرة. بعد سنوات قليلة من نشر التوراة النسبية، أدرك الناس أنها لم تكن توراة واحدة، بل مجموعة من المبادئ وأن أي توراة تتمسك بها هي "نسبية". كما أن ميكانيكا الكم ليست نظرية واحدة بل مجموعة من المبادئ وأي نظرية تدعمها تسمى "الكم". والآن يمكن طرح السؤال، هل هناك تعاليم الكم والنسبية في نفس الوقت؟ تم تقديم الإجابة بالفعل منذ حوالي 80 عامًا (أعتقد أن ديراك كان أول من طرحها وأحرز تقدمًا كبيرًا جدًا في العثور على الإجابة). بالطبع الجواب هو نعم - ليست هناك مشكلة في كتابة توراة نسبية وكمية في آن واحد. في عام 1965، حصل فاينمان وشفينجر وتومونجا على جائزة نوبل لكتابة نظرية كاملة، متسقة رياضيًا وذات دقة رصد عالية، والتي كانت كمومية ونسبية (الديناميكا الكهربائية الكمية).

    على وجه الخصوص، إذا أخذت هذه النظرية ونظرت إلى الحد الرياضي حيث تصل سرعة الضوء إلى ما لا نهاية (أي أن جميع السرعات لا تذكر مقارنة بسرعة الضوء وفي هذا الحد يتم تحديد تحويل لورنتز مع التحويل الجليلي) ، سوف تحصل بالضبط على الوصف الكمي للكهرومغناطيسية كما كانت معروفة في النسخة الأولى من ميكانيكا الكم، قبل أن يعرفوا كيفية دمجها مع النسبية. إذا أخذت نفس النظرية ولكن بدلاً من حد السرعة العالية للضوء نظرت إلى الحد الذي يذهب فيه ثابت بلانك إلى الصفر (أي أن حجم الخصائص الكمومية صغير بشكل لا يذكر مقارنة بحجم أي نظام)، فسوف حصلوا على الوصف الدقيق للكهرومغناطيسية في إطار النسبية الخاصة، كما نُشرت عام 1905 وقبل أن يدركوا أن صيغة أخرى لها تتوافق مع الكميات. هذه الحدود ليست مجرد اختراع اعتباطي: حد سرعة الضوء الذي يذهب إلى اللانهاية هو بالضبط ما يربط النسبية الخاصة بالميكانيكا النيوتونية، وحدود ثابت بلانك الذي يذهب إلى الصفر هو بالضبط ما يربط ميكانيكا الكم بميكانيكا نيوتن.

    وفي الختام - إذا نظرت إلى الطريقة التي كتب بها أينشتاين النسبية الخاصة عام 1905، فسوف ترى صورة لا تتناسب مع ميكانيكا الكم. لكن ما يجب فعله، في عام 1905 لم يكن أحد يعرف أي شيء عن الكميات حتى الآن، لذا فلا عجب أن أينشتاين لم يكتب النظرية بشكل متوافق. بمجرد إنشاء ميكانيكا الكم، وجد الناس طرقًا معقولة تمامًا لصياغة النسبية الخاصة دون الحاجة إلى معرفة موقع الجسيم بالإضافة إلى زخمه، وما إلى ذلك، ولم تكن هناك مشكلة في كتابة النظرية النسبية والكمية.

  131. هيا، لا يمكنك القول أننا لم نحاول.

    "لقد كذبت مرات عديدة، لأن معظم حججك فارغة من المضمون، بسبب ميلك القوي للحديث عن أشياء ليس لديك فكرة عنها"

    إذن أنت تدعي أنني كذبت بوعي.. جميل. كنت أظن دائمًا أن الفيزياء تتعامل مع الحقائق والمنطق، لكن هنا عالم فيزياء ذو ​​قوى يتعامل مع السحر والتنجيم يعرف عني أشياء حتى أنا لا أعرفها..

    بعض الأسئلة لك:

    1. هل يمكنك أن توضح لي وللمنتدى ما الفرق بين ما يقال في المقال وما أقوله؟

    2. هل تدعي أن قياس أحد الجسيمين المتشابكين لا يؤثر على حالة الجسيم الآخر؟

    3. هل هناك جانب في التجربة مفاده أن "جميع البيانات موجودة بالفعل في النظام" كما زعمت عدة مرات لشرح سبب عدم مرور المعلومات بين الجزيئات؟

    4. والسؤال الذي طرحته (أريد أن أعرف حقاً): هل ينطبق مبدأ عدم اليقين على الفوتونات؟ ويبدو لي أن هذا مطلوب، وإلا فيمكن استخدام تأثير كومبتون لمعرفة الموقع الدقيق للإلكترون وزخمه (لست متأكدًا ولكنه يبدو منطقيًا).

    وإذا كان الأمر كذلك، فكيف يتم تطبيق ذلك مع النسبية على أساس تحديد الموقع والزخم للفوتون؟

  132. إسرائيل،

    :). أنت تعرف بالضبط ما أفكر فيه، فلماذا تضيع وقتك؟ مرة أخرى، التقنية ولا الأحذية، لقد كذبت مرات عديدة، لأن حججك في معظم الأحيان فارغة من المحتوى، بسبب ميلك القوي للحديث عن أشياء ليس لديك أي فكرة عنها. تمامًا كما يحدث الآن، عندما تكتب المزيد والمزيد من الأشياء عن المعلومات عندما يكون من الواضح للجميع - بما فيهم أنت - أنك ببساطة لا تعرف شيئًا عن نظرية المعلومات، بما في ذلك التعريفات الأساسية لماهية المعلومات أو ما هو أبعد من المعلومات.

    مرة أخرى، سأستمر في تناول التعليقات حول الفيزياء. هذا حقي. يمكنك الإجابة لي. هذا حقك. يمكنك تجاهل وهذا أيضاً حقك. يمكنك محاولة جرني إلى المعارك. افعل ما تشاء.

  133. ط ط ط ...

    إذن أنت تدعي أنني كذبت، لذا يجب أن أفشل في اختبار كشف الكذب، أليس كذلك؟ ففي نهاية المطاف، هذا ما يفعله، ويكتشف الأكاذيب..

    أليس من الأفضل الاعتذار وإغلاق الموضوع؟ أنت تعرف الآن أنك تحاول فقط أن تظهر حقوقك بناءً على تفاصيل فنية وليس لديك طريقة لمعرفة ما إذا كنت كذبت أو إذا كنت أصدق ما أقوله (حتى لو كنت مخطئًا).

  134. 1. ليس النص الفرعي الذي تفتقده، بل هو النص. تنظر إلى العنوان (بالمناسبة، لا أعرف إذا كنت تعرف أم لا، لكن العنوان والعنوان الفرعي مكتوبان بواسطة المحرر وليس مؤلف المقال) وسطر الملخص بأكمله. انظر إلى كل شيء بينهما. نعم، عندما يكتب مؤلفو المقال أن فون نيومان أعطى ضمانة للفيزيائيين بعدم وجود تناقض، ومن هذه اللحظة فصاعداً كل قسم في هذا الادعاء يأتي من الحجج الفلسفية (أولاً من رقم من 94 ثم من أحد المؤلفون)، والإشارة الوحيدة لمرجع علمي أحدث هو من عالم رياضيات اختار اتخاذ وجهة نظر مختلفة عن فون نيومان والتوصل إلى نفس النتيجة - أرى أن هذا تأكيد من قبل مؤلفي المقال على أنه لا يوجد تقسيم على التعايش بين الفيزيائيين. لا يعني ذلك أنني بحاجة إلى مثل هذا الإذن. كل ما هو مطلوب هو فتح أي مقال حديث عن نظرية المجال.

    2. نعم، لقد كذبت بالتأكيد في المناقشات العديدة التي أجريناها. من المضحك أن تتوقع مني الآن أن أعرف كيف أقتبس لك بالضبط أين وكم ولماذا وكيف مع الأخذ في الاعتبار أن هذه مناقشات كانت بعضها قبل 3 أو 4 سنوات، ولكن لدي بعض الأمثلة في الاعتبار. أحد الأمثلة على ذلك هو أنك ستعود لفترة طويلة من الزمن وتكتب اقتباسًا لي مرارًا وتكرارًا، والذي سيتم قطعه دائمًا بأعجوبة في منتصف الجملة. لقد أوضحت لك مرات عديدة أنك تستشهد بنصف اقتباس وبالتالي تغير ما قلته بالكامل، لكن ذلك لم يزعجك. يتعلق الجزء الأول من الجملة بحقيقة أن هناك أشخاصًا لا يفهمون ميكانيكا الكم وبالتالي يزعمون أنها خاطئة - وأنت لديك نفس المشكلة. الجزء الذي كنت قد حذفته يوضح أنني لا أتهمك بإنكار ميكانيكا الكم ولكن بنفس النوع من الخطأ - أنت لا تفهم شيئًا ما، وبالتالي تستنتج أنه ربما يكون خطأ. مثال آخر هو أنه خلال مناقشة طويلة حول حل لغز واحد أو آخر من العملات المعدنية بمساعدة التشابك، زعمت أنني قلت أن لديها حلًا كلاسيكيًا على الرغم من أنني كتبت صراحةً عدة مرات أنه ليس لديها حل كلاسيكي. كنت أحاول فقط أن أشرح لك أن الحل الكمي لا يتضمن نقل المعلومات لأنه يتطلب المزامنة المسبقة لعدد كبير من الأزواج المتشابكة - وهو المعادل الكمي للتنسيق المسبق للكود، والغش وفقًا لقواعد اللعبة. ورغم ذلك اتهمتني مرارا وتكرارا بالادعاء بأن هناك حلا كلاسيكيا.

    بالطبع سترفض هذه الأمثلة، رغم أن هناك الكثير غيرها. لكنني حقا لا أهتم. إذا كنت لم تكتشف الأمر بعد، فليس لدي أي نية لبدء قتال معك. لن أعتذر أيضًا، ولن أوقّع أي عقد معك. كلانا يعرف بالضبط سبب محاولتك جاهدة لدفع المناقشة إلى هذا الحد - لأنه ليس لديك ما تقوله. هل تريد التحدث في الفيزياء؟ إنه ممكن لا اريد؟ حقك، وعد بعدم تحمل رفضك ضدك كما يفعل الناس في بعض الأحيان. هل تريد أن تكتب مائة مرة أخرى أنك ضحية وأنني أضطهدك؟ ربما لذلك.

    3. حسنًا، يمكن لأي شخص أن يرتكب الأخطاء أحيانًا. ليس سيئًا. لكن أرجو أن تدرك الآن أنه حتى في ساكوراي لا يوجد أي دعم لادعاءاتك، ناهيك عن الأدبيات المهنية ذات الصلة. وهذا على عكس ما كتبته في التعليق السابق، والذي من المفترض أن يفهم من ساكوراي أن هناك نقطة في كلامك.

    4. "ولكن كيف يمكننا معرفة ما إذا كانت المعلومات قد مرت، سواء كانت مفيدة أم لا، إذا لم يحدد المراجع ما هي المعلومات ولا تعرف ذلك إلا بعد المراجع"؟ سهل جدا. سنذهب إلى كتاب ذي صلة بنظرية المعلومات بدلًا من كتاب تعريفي بميكانيكا الكم، سندرسها وتعريفاتها وأدلة النظريات التي اكتشفها الناس. لذا يبدو أنه وفقًا لنظرية المعلومات، لم يتم تمرير أي معلومات. وكالعادة أذكرك أنه إذا أردت أن تخترع توراة جديدة بتعريف جديد للمعلومات المفيدة وغير المفيدة وكل ذلك فهذا حقك. في الوقت الحالي، أفضل ما لدينا هو نظرية المعلومات، ووفقًا للفهم الذي توفره لنا (والذي يبدو أنه فهم ممتاز وفقًا لنجاحاتها العديدة في الفيزياء وعلوم الكمبيوتر والهندسة)، فإن المعلومات لا تمر.

    "وبشكل عام، كيف يتحدث علماء الأحياء بهذه الثقة عن "معلومات وراثية" موروثة وبدون تعريف؟" هذا سهل أيضًا. لا يتعامل علماء الأحياء مع نظرية المعلومات ويستخدمون كلمة معلومات أو معلومات بنفس الطريقة التي تستخدمها بها، بالمعنى اليومي. فكما يستطيع المؤرخ أن يقول عن الإمبراطورية الرومانية إنها كانت أقوى قوة في العالم في عصرها، لكن ذلك لا يعني أنها ارتقت إلى معادلة نيوتن. من الممكن بالتأكيد إجراء التوازي بين المعلومات كما هي محددة في نظرية المعلومات والمعلومات الموجودة في كود معين (مثل كود الكمبيوتر، أو الكود الجيني المماثل)، ولكنه ليس بالأمر التافه. ومرة أخرى الحل هو دراسة التوراة وتعريفاتها *قبل* الحكم بعدم صحتها أو محاولة تفسير الجمل الواردة فيها.

    هل هذا مرتبط بما زعمت عدة مرات أن "جميع البيانات موجودة بالفعل في النظام"؟ ليس لدي أي فكرة عما كنت تتحدث عن. ربما كنت تتحدث عما ذكرته من قبل، حول كيف أن حل لغز العملات المعدنية باستخدام العديد من أزواج اللفات المتشابكة يتطلب أنه قبل بدء اللغز، يجلس المشاركون ويجهزون جميع أزواج اللفات معًا، بل ويتأكدون بالضبط في أي منها حالة متشابكة لكل زوج (لأن الحالة المتشابكة ليست فريدة من نوعها، هناك دائمًا الكثير مما يجعل التمييز بينهما ليس تافهًا على الإطلاق). لا أعلم، ربما في هذا السياق أخبرتك ذات مرة أنهم لا يمررون المعلومات فيما بينهم ولكنهم قاموا بالفعل بتنسيق المعلومات مسبقًا عند تشفير الأزواج المتشابكة. ولكن إذا كنت تريد مرجعًا أكثر تحديدًا، فسأحتاج إلى إشارة إلى ما قلته بالضبط والسياق الذي قلته فيه.

  135. المعجزات

    بسيط جدًا: لا يمكن إرسال المعلومات من خلال التشابك.

    إلبينتزو

    "لقد تلقيت توضيحات مفصلة تمامًا عن سبب عدم ذكر المقال في مجلة Scientific ما تدعي أنه يقوله"

    عنوان المقال: تهديد كمي للنظرية النسبية. الأطروحة المركزية: "يبدو أن نوع اللامكانية الذي نواجهه في ميكانيكا الكم يتطلب التزامن المطلق، مما يشكل تهديدًا حقيقيًا ومميتًا للنسبية الخاصة."

    وهذه هي المشكلة.

    الجملة التي تفتتح ملخصه: "حالة النسبية الخاصة، بعد ما يزيد قليلاً عن قرن من ظهورها على المسرح العالمي، أصبحت فجأة مسألة مفتوحة على مصراعيها وسريعة التطور."

    هل هناك نص فرعي هنا لم أستطع فهمه؟

    و هل تقصد الجملة التالية:

    "لعقود من الزمن، رأى مجتمع الفيزيائيين بأكمله دليل فون نيومان كنوع من شهادة التأمين على أن اللامكانية لميكانيكا الكم والنسبية الخاصة يمكن أن توجد بسلام جنبًا إلى جنب."

    كمرجع للإجماع في المجتمع الأكاديمي على عدم التناقض بين النسبية والكم؟ هل قرأت تتمة؟

    أنا لا أسميهم كاذبين وقحين... حسنًا، حسنًا. بالمناسبة، لم يكذبوا على حد علمي، لذلك لا أعرف لماذا أطلق عليهم هذا الاسم.

    لذا أعتقد أنني كذبت بلا خجل..

    هل يمكنك الإشارة إلى مكان واحد فقط كذبت فيه، وهو أنني فشلت في اختبار كشف الكذب؟

    "لقد قلت لي أيضًا الكثير من الأشياء غير اللطيفة."

    هل يمكنك الإشارة إلى المرة الوحيدة التي هاجمتك فيها أولاً؟ لأنني أستطيع أن أشير إلى عشرات المرات التي قمت فيها بذلك.

    وإذا كنت أنت أو أي شخص آخر ستأتي بحجة طفل الروضة، أي أنه لا يهم من هاجم أولاً ومن رد، فهل هذا المثقف مستعد للهجوم على نفسه ويصمت؟

    "هل تتحدث عن مكان آخر في الكتاب؟ أم أنك احترت بين "عدم تمرير المعلومات المفيدة" و"عدم تمرير المعلومات المفيدة"؟

    لم أطلع على الكتاب منذ فترة طويلة وأنا في العمل، ولكن أعتقد أنك على حق والمعنى هو "عدم تمرير معلومات مفيدة".

    ولكن كيف يمكننا معرفة ما إذا كانت المعلومات قد تم تمريرها، سواء كانت مفيدة أم لا، إذا لم يحدد سكوراي ما هي المعلومات ولم نتعلم ذلك إلا بعد سكوراي؟

    وبشكل عام، كيف يتحدث علماء الأحياء بهذه الثقة عن "معلومات وراثية" موروثة وبدون تعريف؟

    هل هذا مرتبط بما زعمت عدة مرات أن "جميع البيانات موجودة بالفعل في النظام"؟

    نحن - قريبًا سنعود إلى المجرة البعيدة حيث يمكننا أن نتناقش بحرية.

    هل تواجه أيضًا مشكلة في كتابة النصوص؟ اعتقدت أنه كان موجودًا على نظام Mac فقط، ولكنني أرى أنه موجود أيضًا على جهاز الكمبيوتر.

  136. ألبانزو
    شكرا!
    حسب فهمي، فإن ما "لا تسمح به" النظرية النسبية هو تناقض. يصف ماريو ليفيو تجربة فكرية تصطدم فيها طائرة هبوط بمركبة تعبر المدرج (تراقب في نهاية المدرج). فلو كانت سرعة الضوء هي نفس سرعة الطائرة، فمن وجهة نظر الراصد، لن يتصادما، وهذا تناقض بالطبع.
    لكن - لا يوجد أي قيد، طالما لا يوجد تناقض. على وجه الخصوص - التشابك لا يخلق تناقضا.

    وفي سياق ما قلته، لأنها عملية عشوائية، ليس من المنطقي أن نقول إن "المعلومات تنتقل من جهة إلى أخرى". ومن ناحية أخرى - نحن (على الأقل أنا) لا نفهم حقًا ما يحدث هناك.

  137. شيء آخر: أردت أن أقرأ في ساكوراي ما قلته ("نقل معلومات عديمة الفائدة") لمحاولة فهم ما حدث هناك - هل يستخدم كلمة معلومات بطريقة يومية بسيطة (لا يعرّف المعلومات على أنها عملية رياضية) المفهوم وحتى تناول الموضوع في الكتاب بأكمله)، أو ربما لم تحصل على شيء صحيح. أعدت قراءة جميع الأجزاء ذات الصلة من الكتاب وقمت حتى بإجراء بحث عن الكلمات. لم أجد شيئا من هذا القبيل. كل ما وجدته هو المكان الذي يقول أنه لم يتم تمرير أي معلومات مفيدة (وهو *مختلف جدًا* عن "تم تمرير معلومات غير مفيدة")، وبعد بضعة أسطر يقول أنه لم يتم تمرير أي معلومات على الإطلاق. أعتقد أن هذه الصياغة المتهورة جاءت ببساطة لأن الكتاب لا يميز بين الارتباط والمعلومات. وهذه الطبعة الثانية، صفحة 245.

    هل تتحدث عن مكان آخر في الكتاب؟ أم أنك احترت بين "لا يمرر معلومات مفيدة" و"تمرير معلومات غير مفيدة"؟

  138. معجزات,

    1. لا أعتقد ذلك. من حيث المبدأ، تحدد نظرية المعلومات المعلومات وتتحدث عن المعلومات في سياق المتغيرات العشوائية (النظرية الكلاسيكية بالطبع. في نظرية الكم، كل شيء مختلف قليلاً). كل متغير عشوائي هو في الأساس يانصيب يتم الحصول بعد تنفيذه على نتيجة واحدة فقط، وبالتالي فإن الظاهرة التي ذكرتها مشتركة بين جميع المعلومات وليس فقط في التشابك. في الواقع، ترتبط المعلومات على وجه التحديد بهذا "الانهيار" - بمعنى ما، فهي تصف مدى توقع أنه من بين جميع النتائج المحتملة، كانت النتيجة التي تم الحصول عليها هي النتيجة الصحيحة.

    2. لا أعتقد أنني فهمت بالضبط ما تقصده. هل يمكنك محاولة التوضيح؟ ما يميز التشابك هو أن كل جزء من النظام ليس محددًا بشكل جيد من تلقاء نفسه، لذلك يتم إجراء كل قياس في وقت واحد على كلا الجزأين. ما يجعل في الواقع عدم وجود أي تناقض مع النسبية العامة هو أن ميكانيكا الكم هي نظرية إحصائية ويمكن "فرضها" على أحد أجزاء النظام. يوضح هذا المتوسط ​​أن شخصًا ما في أحد طرفي النظام لا يتلقى أو يقدم معلومات إلى شخص ما في الطرف الآخر. لا أعلم إن كان ذلك قد بسط الأمر أم تعقيده.... على أية حال، أنا مثلك أزعم أنه لا يوجد تناقض بين النسبية الخاصة وميكانيكا الكم، وكذلك عالم الفيزياء بأكمله. كما تعلم على الأرجح، على مدار السبعين عامًا الماضية (تقريبًا) كانت النظرية السائدة هي نظرية المجال الكمي، والتي تبين مرارًا وتكرارًا أنها وصف دقيق وغير عادي للواقع (على الرغم من أننا نعرف على وجه اليقين أنها ليست وصفًا استثنائيًا). وصفًا كاملاً لأنه لا يستطيع وصف الجاذبية، وبالتالي فهو على الأكثر صورة ثلاثية الأبعاد للواقع). وهذه التوراة كمية ونسيبية صراحة (خاصة فقط) ولم يوجد فيها أي تناقض في هذا السياق. لذلك، حتى في المقال الذي قدمته إسرائيل، والذي يتحدى قدرة النظريين على العيش بسلام مع بعضهم البعض، فقد ورد صراحة أنه يوجد إجماع بين الفيزيائيين على عدم وجود مشكلة، والادعاءات التي يتم تقديمها هناك ( وهي، على عكس ما قالته إسرائيل، وأنا لا أضحك منهم ولا أقلل منهم) ادعاءات فلسفية، وكما يقول الكتاب صراحة، يمكن حلها من خلال قبول تفسير مختلف عن التفسير المقبول للواقع الذي نعيش فيه .

  139. وبالمناسبة، لم "أضحك" في أي وقت من الأوقات من المقالة المنشورة في مجلة Scientific. قلت إنه كان بالفعل أكثر فلسفية وليس علميًا، لكنني لم أسخر منه، بل وكتبت أنه مثير للاهتمام. إذا قرأته، سترى أن المؤلفين يتفقون معي أيضًا - فكل حججهم المتعلقة بإمكانية وجود توتر معين بين النظريات هي حجج فلسفية. إنهم أنفسهم لا يتظاهرون بإيجاد أي تناقض رياضي، بل ويتحدثون عن حقيقة أن علماء الرياضيات يجدون في السنوات الأخيرة طرقًا أكثر صرامة لإثبات عدم وجود تناقض (أكثر صرامة من الطرق التافهة المقترحة في ثلاثينيات وأربعينيات القرن العشرين، مثل فون نيومان).

  140. إسرائيل،

    لن أعتذر لك. وعلى عكس ما تقوله الآن، فإن الأشياء التي قلتها لك لم أقلها لك لأنك "تجرأت على التعبير عن رأي مختلف عن رأيي". لقد أخبرتك أنك أحمق عندما اعتقدت أنك تتصرف مثل الأحمق، وما إلى ذلك. لقد قلت لي أيضًا الكثير من الأشياء السيئة. لن أعتذر ولن أخوض في جدالات شخصية مرة أخرى. اعتقدت أنك لم تكن مهتما أيضا. إذا كنت مخطئًا - استمتع، يمكنك كتابة مخطوطات حول مدى كونك ضحية وأنني أتيت في يوم من الأيام وأسيء معاملتك لأنني لم أحب اسمك أو أي هراء آخر. لا أعتقد أنني سأرد على هذه الأشياء.

    فيما يتعلق بالفيزياء - 1. لقد تلقيت توضيحات مفصلة تمامًا عن سبب عدم ذكر المقال في مجلة Scientific ما تدعي أنه يقوله، وما الذي يقوله بالفعل. كل ما عليك قوله هو أنني لا أسميهم كاذبين وقحين... حسنًا، حسنًا. بالمناسبة، لم يكذبوا على حد علمي، لذلك لا أعرف لماذا أطلق عليهم هذا الاسم.

    2. أنت تكرر نفس الخطأ. أحضر اقتباسًا من ويكيبيديا لكنه لا يفهم ما يقوله على الإطلاق. أنت لا تعرف ما هو المراقب أو ما معنى التواصل (بالطبع أنت تعرف ماذا تعني الكلمات، لكنك لا تعرف ما الذي تحدده في النظرية الرياضية لنظرية المعلومات). لذا عليك أن تأخذ الجملة وتفسرها كيفما تشاء وفقًا لفهمك اليومي لتلك الكلمات. إذا كنت مخطئا وأنت تعرف ما تعنيه هذه المفاهيم، فيرجى أن تشرح لي على المستوى الرياضي ما هو خطأي.

    3. بالطبع، ساكوراي مجرد رجل قش هدفه - وأعتقد ربما دون وعي - هو عدم التعامل مع نقص معرفتك بنظرية المعلومات. كما تم شرحه لك عدة مرات من قبل، ساكوراي لا يتعامل مع المعلومات على الإطلاق. إنه ليس "أسطوريًا" والسبب الوحيد الذي يجعلك تعرف هذا الاسم على الإطلاق هو أنني، بطريقة غير مرتبطة تمامًا، حاولت أن أجعلك تفهم أن لديك فجوات معرفية في ميكانيكا الكم لا تسمح لك بفهم موضوع التشابك والمعلومات على الإطلاق، مثل شخص يحاول فهم التحليل المركب دون أن يعرف أي شيء. ولهذا السبب ظللت أحيلك إلى ساكوراي، وحاولت أن أجعلك تفهم أن هذا الكتاب، الذي يعكس معرفة الطالب في نهاية الدرجة الأولى، لا يزال أمامك طريقًا طويلًا. ولم أقل ذلك من أجل الإهانة، ولا الآن، فقط لأجعلك تفهم. بمعنى آخر، كل تصوراتك لكلمة أو أخرى (لاحظ أن الفيزياء النظرية هي نظام رياضي محدد جيدًا - الكلمات ليس لها معنى كبير فيها) في كتاب لا يتظاهر بقول أي شيء أو نصف شيء عن الكم المعلومات هي مجرد هروب من الواقع. انتقل إلى النصوص التي تتناول الموضوع، وانظر ما هي المعلومات، وانظر ما هو انتقال المعلومات، وانظر ما الذي تتوقعه. لذا يمكنك أن ترى أن ما تسميه "معلومات غير معروفة" ليس بالضرورة معلومات وفقًا لنظرية المعلومات، وبالتالي فإن "انتقالها" ليس بالضرورة نقل معلومات وفقًا لنظرية المعلومات. تعتمد حجتك بأكملها على حقيقة أنك تبدأ من افتراض يومي بديهي حيث يشير أي ارتباط بين موقفين إلى نقل المعلومات بينهما، لكن نظرية المعلومات تحدد بوضوح المعلومات والارتباط، والذي بموجبه لا يتضمن الارتباط النقل ممكن.

    كما قلت لك عدة مرات من قبل - إذا كانت لديك فكرة عن نظرية معلومات أفضل، مع تعريفات أخرى تنص على أن هناك انتقالًا غير معروف للمعلومات (والتي تستنسخ النجاحات المعروفة للتوراة القديمة)، فمرحبًا بك. سوف أستمتع بتعلم شيء جديد. لكن أن نأخذ التوراة القديمة بما فيها من جمل حسب تعريفات معينة، ثم نحاول تطبيقها مع تغيير التعريفات واختراع مفاهيم جديدة غير محددة جيدا على الإطلاق، فهي مزحة.

  141. إلبينتزو

    طلبت منك الاعتذار عن كل الألقاب المسيئة التي أطلقتها عليّ بسبب جرأتي على التعبير عن رأي مختلف عن رأيك. من المثير للاهتمام أن مجلة ساينتفيك أمريكان تدعي ما أدعيه حرفيًا تقريبًا، وعلى الرغم من أنك تحاول الاستهزاء بموقفهم باعتباره "فلسفيًا" وليس علميًا، على الأقل لا تصفهم بالكذابين الوقحين، والمتصيدين الأغبياء والمخرفين الذين ينتشرون على ما يبدو " "القمامة العلمية" الذين لا يستطيعون التعامل مع حقيقة أنهم لا يفهمون ما يفهمه الأطفال في سن العشرين بسهولة.

    وفي واقع الأمر، من ويكيبيديا:

    في الفيزياء، نظرية عدم الاتصال هي نظرية محظورة من نظرية المعلومات الكمومية التي تنص على أنه أثناء قياس الحالة الكمومية المتشابكة، لا يمكن لمراقب واحد إجراء قياس لنظام فرعي للحالة الإجمالية، لتوصيل المعلومات إلى مراقب آخر

    إذا قرأ أي شخص مثال العملات المعدنية وأجهزة الراديو التي أحضرتها، فيمكنك أن ترى بوضوح ما أدعيه: تشير نظرية عدم الاتصال فقط إلى الجزء الثاني - أي إرسال المعلومات باستخدام الجهاز الذي اقترحته مع أجهزة الراديو التي تعيد إنتاج المشهد، وليس إلى الجزء الأول، فإن كيفية تواصل أجهزة الراديو مع بعضها البعض، لا تشير نظرية عدم الاتصال إلى الموضوع على الإطلاق.

    فيما يتعلق بالمعلومات المجهولة: المراجع الأسطوري، رغم أنه يدعي أنه لا يتم تعلمها إلا قبل دراسة نظرية المعلومات، يتحدث عن نقل المعلومات غير المفيدة عن طريق التشابك (وهذا دون تعريف المعلومات أو المعلومات غير المفيدة).

    أعتقد أن "معلومات Skorai غير المفيدة" هي نفس "معلوماتي غير المعروفة". لن أكون قادرًا على تقديم دليل رياضي صارم، لكن Skorai لا يفعل ذلك أيضًا ويستخدم المفهوم بدون تعريف، لذلك ربما أكون بخير..

  142. ألبانزو
    لدي سؤالين.
    لنفترض أن زوجتي فقدت مفاتيحي في المنزل. طالما أنني لم أجدهم - فهناك احتمال معين أنهم موجودون في جميع أنواع الأماكن في المنزل. أعرف ما هي الاحتمالات: من خلال الخبرة، عادة ما تكون المفاتيح في الدرج، وأحيانًا في جيب البنطال، وأحيانًا بين وسائد الأريكة.
    بمجرد أن وجدتهم - تمت إعادة ضبط كل هذه الاحتمالات. هل يستشعر هذا المثال حالة التشابك؟

    السؤال الثاني يتعلق بما قلته لإسرائيل – هل النقطة المهمة هي عشوائية القياسات؟ صحيح أن هناك تشابكًا بين الحالات، لكن كل حالة في حد ذاتها تكون عشوائية (مثل استقطاب الفوتونات).
    أنظر إليه كما لو كان في الواقع فوتونًا واحدًا موجودًا في مكانين، ولا أرى أنه يتعارض مع النظرية النسبية (السببية، وما إلى ذلك).

  143. إسرائيل
    كيف تفهم الجملة التالية (من ويكيبيديا)
    "في هذه التجارب، تظهر نظرية عدم الاتصال أن فشل الواقعية المحلية لا يؤدي إلى ما يمكن الإشارة إليه باسم "الاتصال الشبحي عن بعد" (قياسًا على وصف أينشتاين للتشابك الكمي بأنه "فعل شبحي عن بعد" )"

  144. إسرائيل،

    أو ربما ينبغي لي أن أوجه الرد فعليا إلى نسيم والأشخاص الآخرين الذين قد يقرأون المناقشة ويريدون أن يفهموا، لأن إسرائيل لا تريد التحدث معي وهذا بالطبع حقه الكامل. هدفي ليس جره إلى نقاش ولكن تقديم المعلومات لأولئك المهتمين بالمعلومات الكمومية - ويمكن لأولئك غير المهتمين (أو المهتمين ولكنهم لا يريدون أن يسمعوا مني شخصيًا) أن يتجاهلوا ذلك.

    جمل عدم الاتصال هي جمل رياضية. أنها تثبت بعض الادعاءات حول نظرية المعلومات. وبالطبع فإن هذه المطالبات، مثل شروط وجود الجمل، تصاغ في لغة نظام منطقي على تعريفات معينة. أي أن هناك تعريفًا رياضيًا دقيقًا لماهية "المعلومات" في سياق الجملة، وما هو "نقل المعلومات"، وما إلى ذلك. من السخف محاولة تطبيق هذه النظريات، ولكن استبدال التعريفات بتعريفات أخرى (خاصة وأن تلك التعريفات الأخرى ليست محددة رياضيًا بشكل جيد). يمكن لأولئك المهتمين حقًا بالموضوع أن يتعلموا أساسيات نظرية المعلومات ويثبتوا لأنفسهم أنه بموجب تعريفاتها لا توجد "معلومات معروفة" أو "معلومات غير معروفة". المعلومات هي كمية كمية يتم تعريفها دون أي اعتماد على "المراقب" وعندما تثبت جملة عدم الاتصال أنه لا يمكن نقل المعلومات، فإن هذا ينطبق أيضًا على محادثة بين صديقين يقرران ما سيقولانه لبعضهما البعض، و أيضًا لإشارة الراديو التي ليس لها سيطرة على النظام ولكن يمكنها التواصل مع المستخدم النهائي وتتبع حالة عملته المعدنية وتنشيط الجهاز وفقًا لذلك.

    ومرة أخرى، في الختام - إن جمل عدم الاتصال لا تقول في الواقع ما تدعيه إسرائيل. ويستند ادعاؤه إلى سوء فهم ماهية المعلومات أو تمرير المعلومات تحت تعريفات التوراة الرياضية، واستبدال المفاهيم الصحيحة بالمفاهيم البديهية التي تناسب طريقة تفكير معظم الناس في الحياة اليومية.

  145. ויקי

    المفارقات التي لم يتم حلها في العلاقات، بما في ذلك مفارقة إهرنفيست، غير معروفة. لكنه بلا شك غريب وإذا قرأت المقال، فربما يتعارض مع الكميين، وإذا قرأت رابط العرض في سانتا مونيكا، مع بعض النظريات الأخرى.

  146. سكة مستديرة وترتفع.

    حدس؟ نحن لا نتعامل مع الكميات والنسبية، فما علاقة الحدس بهما؟

    أنا فقط أحاول إظهار مدى غرابة تقصير الطول (انظر مدى التعقيد الذي وصلنا إليه وتوصلنا أخيرًا إلى نتيجة خاطئة، انظر مدى تعقيد 4 أساتذة في مفارقة التوأم ووجد واحد فقط إجابة منطقية، كل والبعض الآخر أعطى إجابات متناقضة) ليبين أن حل الفوتون المتحرك بجميع السرعات يحل الكثير من المشاكل رغم غرابته.

    "إن حقيقة استحالة نقل المعلومات بهذه الطريقة تظهر أن شيئًا ما يحدث هنا وهو ليس "نقل معلومات" كلاسيكيًا.
    صحيح أن القطعتين تقعان دائمًا على نفس الجانب، لكن ليس لدينا طريقة للتأثير على أي جانب..."

    وهنا التوضيح الذي قدمته من قبل.

    النظر في العملات المعدنية في غرف مختلفة دون التواصل فيما بينها. هل هناك طريقة لجعل العملات المعدنية تقع دائمًا على نفس الجانب؟ سلبي.

    الآن قم بتوصيل العملات المعدنية بالراديو، وعندما تسقط إحدى العملات المعدنية على شجرة أو حقل، تنتقل هذه المعلومات إلى الغرفة الأخرى وتقوم آلية ميكانيكية على الفور بترتيب الأخرى في نفس الوضع.

    الآن بالطبع يمكنك جعل العملات المعدنية تسقط دائمًا على نفس الجانب. هل تم تبادل المعلومات بينهم؟ إيجابي. وأجهزة الراديو هي التي نقلت ذلك.

    هل من الممكن إرسال المعلومات باستخدام هذا الجهاز؟ سلبي. لكي نتمكن من إرسال المعلومات، يجب أن نكون قادرين على التأثير على الجانب الذي ستسقط فيه العملة التي سقطت أولاً وتحديد حالة النظام. وبما أننا لا نستطيع القيام بذلك، فإن الجانب الذي سقطت عليه العملة هو معلومات غير معروفة بالنسبة لنا، لذلك لا يمكننا إرسال المعلومات بهذه الطريقة.

    الجزء الثاني فقط، إرسال المعلومات، يتم الحديث عنه في نظريات عدم الاتصال. ما أتحدث عنه هو الجزء الأول، مع الراديو.

  147. إسرائيل
    وأنا أتفق معك أنه ليس بديهيا.
    ولكن - حقيقة أنه من المستحيل نقل المعلومات بهذه الطريقة، تظهر أن هناك شيئًا ما يحدث هنا، وهو ليس "نقل معلومات" كلاسيكيًا.
    صحيح أن القطعتين تقعان دائماً على نفس الجانب، لكن ليس لدينا طريقة للتأثير على أي جانب...

  148. يسير القطار على مسار دائري مثل لعبة القطار ويملأ المسار بأكمله وبالطبع تؤثر عليه قوة شعاعية وتسارع شعاعي.

    إذا كان نصف القطر صغيرًا والسرعة عالية، فسوف تطير خارج القضبان. لكن نصف القطر كبير جدًا لدرجة أنه يتحرك في خط مستقيم تقريبًا.

    ماذا سيحدث عندما يصل إلى عامل جاما 2؟ هذه هي نفس حالة سؤال العربة الذي تحدثنا عنه سابقًا.

    كنا نظن سابقًا أنه بين ساعتين على المسار في لحظة معينة سيكون هناك ضعف عدد السيارات بسبب قصر الطول، لكن هذا مستحيل لأنه يعني أن بعض السيارات ستتجاوز سرعة الضوء.

    ولكن هذا ليس ما سيحدث. سيكون هناك بالضبط نفس العدد من كرور (تحدثنا عن مليار) ولكن ستكون هناك مسافات بينهما وإلا فإنها سوف تتمزق، انظر مفارقة الجرس.

  149. أعتقد أنني أفهم ما يحدث مع العربات.

    لا (يحدث).

    الحل هو نسبية التزامن.

    لرؤية ذلك، دعونا نفكر في مسار دائري ضخم يوجد عليه قطار دائري ضخم بنفس القدر. لكل وحدة طول على المسار، تناسب x سيارة.

    في لحظة معينة، يبدأ القطار في التسارع ويصل إلى عامل جاما يساوي 2. وبسبب نصف القطر الكبير، فإن التسارع الشعاعي لا يكاد يذكر.

    يقول المنطق أنه بسبب قصر الطول، عندما يصل القطار إلى سرعة ثابتة، فإن سيارتين ستتسعان الآن لكل وحدة طول على المسار، لكن هذا مستحيل لأننا سنحصل على ضعف السيارات السابقة وهذا مستحيل.

    ما سيحدث هو أنه بسبب النسبية اللحظية سيتم تقصير القطارات. إذا كانت منفصلة وذاتية الدفع، فسوف تنفصل عن بعضها البعض وسيتم إنشاء مسافات بينهما، ولكن لا تزال x من السيارات مناسبة لكل وحدة طول على المسار.

    إذا كانوا متصلين ببعضهم البعض كما هو الحال في القطار العادي، فسوف ينقطعون ويتفككون، مثل الحبل في مفارقة سفينة بيل الفضائية.

  150. يبدو لي أنه ربما يكون التفسير هو النفور، أي أنه بسبب تقصير الطول، فإن المزيد من الجزيئات ذات الشحنة الموجبة تؤثر بقوة أكبر. أحاول معرفة ما إذا كان ذلك سيحل مشكلة النقل أيضًا، لكن في الوقت الحالي

  151. أنصحك بشرب الكثير من الماء.

    ولا أرى سببًا لزيادة عدد الإلكترونات، كما هو الحال في نظام مغلق من المضخات والأنابيب، لا تزيد كمية الماء.

    علاوة على ذلك، فإن التأثير الذي ذكرته ليس نسبيًا.

    في الفيديو، بالمناسبة، يمكنك حساب كمية الشحنات حرفيًا (12 من كل نوع عندما تكون القطة في حالة راحة، و16 موجبة و8 سالبة عندما تتحرك).

    وبما أنك سألت ما هو تفسيري (ليس لدي تفسير)، لكن يمكنني تخمين ما كان سيقوله ليساج.. (أين سيكون يودا حقًا؟).

  152. إسرائيل
    تقديم توصيات - سأكون هناك في غضون شهر.
    أريد أولاً أن أفهم ما كتبه ألبينزو... في ظاهر الأمر - يبدو لي أن الأمر لا يستحق مناقشة الموصلية الفائقة لأنها ظاهرة كمومية.
    في الحالة الكلاسيكية يجب أن يزيد عدد الإلكترونات، وإلا لن يكون هناك تيار (قانون أوم). لست متأكدًا من أنني على حق، لأنه بشكل عام من الممكن إنشاء تيار في السلك بواسطة مجال مغناطيسي آخر (على سبيل المثال، حلقتين بعيدتين، عندما تحرك مغناطيسًا بالقرب من حلقة واحدة، يتم حث تيار في السلك) حلقة أخرى أيضًا، وهنا لا تضيف إلكترونات حقًا.

    هناك الكثير لنفهمه هنا 🙂

  153. هاواي حبيبتي، هاواي.

    المعجزات

    يمكنك استخدام كيلومتر ضئيل، ولكن بالنسبة لك مميز - حلقة حول الصفر المطلق، لن يتوقف التيار لعدة أيام حتى بدون كيلومتر.

    ولم أفهم أمر الكثافة. بعد كل شيء، بيت القصيد هو أن الكثافة لكل وحدة حجم (أو طول) تزداد بسبب تقلص الطول، وهناك شحنات موجبة أكثر من السالبة لكل وحدة حجم، وبالتالي يتم إنشاء قوة. ترى ذلك بوضوح في الفيديو في الدقيقة الثانية تقريبًا، حيث يكون عدد الشحنات الموجبة لكل وحدة طول ضعف عدد الشحنات السالبة، بينما كانت قبل ذلك متوازنة.

    لذا، إذا كان هذا هو الحال في جميع أنحاء الدائرة المغلقة، فأين ظهرت فجأة العديد من الشحنات الموجبة الإضافية؟

  154. ألبانزو
    شكرا على الشرح مع أنني لم أفهم إلا البداية. أفهم ذلك من خلال القياس التالي: إذا كنت بالقرب من سكة قطار وأشعر بالجاذبية من القطار، فإن هذه الجاذبية لن تزيد إذا تحرك القطار بسرعة نسبية.
    هل حصلت عليها بشكل صحيح؟

  155. عدد الإلكترونات ليس كثافتها. عدد الإلكترونات هو تكامل الكثافة على الحجم (أو المساحة، أو الطول. ويعتمد ذلك على كثافته بالطبع). يمكن للمرء أن يرى بشكل عام - دون النظر بشكل خاص إلى حلقة أو قرص أو مكعب أو سلك - أن تحويل لورنتز على الكثافة وتحويل لورنتز على الفضاء يقابلان بعضهما البعض، بحيث يكون التكامل ثابتًا للتحويل. على المستوى الرياضي، يرجع ذلك إلى حقيقة أنه ليس فقط المتكامل يخضع للتحول، ولكن أيضًا درجة التكامل يجب أن يكون لها يعقوبي ويمكن ملاحظة بشكل عام أن مصفوفة يعقوبية هي معكوس مصفوفة التحويل . لهذا السبب، لا يهم حقًا ما إذا كان التحول المعين الذي يحول الدائرة إلى شكل ناقص أو المربع إلى مستطيل أو شريط Wattbar يبدو غريبًا عندما تنظر إلى كثافته. إذا كان متسقًا مع تحويل لورنتز، فلن يتغير عدد الجزيئات. وبالطبع، كل هذا لا يصدق إلا في العلاقة الخاصة حيث يكون الفضاء مسطحًا ويتصل جميع المشاهدين ببعضهم البعض بسرعة ثابتة. إذا لم يكن الفضاء مسطحًا أو إذا كان بعض المراقبين يتسارعون، عليك أن تنظر إلى النسبية العامة وهناك القصة مختلفة تمامًا (عدد الجسيمات ليس ثابتًا بالتأكيد، بل إنك تراه تجريبيًا).

  156. إسرائيل
    لإحداث تيار في سلك، تحتاج إلى جهد على طول السلك. هذا يعني أن هناك فائضًا من الإلكترونات في السلك (وهذا لا يختلف عن نفخ الهواء عبر أنبوب هوائي).

  157. على متن الطائرة، في الطريق إلى هاواي، قبل 5 دقائق من الإقلاع.

    لذلك من المستحيل أن تصل السيارة التي تبعد ساعة ضوئية إلى الصورة في غضون 10 ثوانٍ.

    وفيما يتعلق بالإلكترونات المزدحمة: إذا فكرت في حلقة يتدفق فيها تيار كهربائي، فكيف يمكن أن تزيد كثافة الإلكترون في نقطة ما على الحلقة دون أن تنخفض في نقطة أخرى؟ بعد كل شيء، عدد الإلكترونات ثابت على الحلقة..

    وكما قلت، غريب للغاية.

  158. اترك الفوتونات للحظة، لننتهي من مسألة العربات.

    إذا بدأت عندما كانت لديك سيارات أمام النقطتين 1 و 2 يفصل بينهما مليار كرونة، وعندما انتهيت كان لديك سيارات أمام نفس النقاط التي يفصل بينها 2 مليار كرونة، فمن أين أتت السيارات؟ أنه قبل التسارع كان على بعد ساعة ضوئية على الأقل من النقطتين. لا يمكنك الحصول على ذلك لا علاقة له بالعلاقات.

    إذا لم يكن الأمر كذلك - اشرح لي أين كانوا من قبل.

  159. إسرائيل
    دعونا نفكر في حالة بسيطة. هناك 3 نقاط على المسار، أ على يساري على بعد سنة ضوئية، ب أمامي، و ج على بعد سنة عن يميني. عند اللحظة 0، يخرج فوتون من M وأيضًا من M، يخرج فوتون باتجاه اليمين. وبعد عام سيصلني الفوتون Ma وسيصل الفوتون Mb إلى L. أي أن المسافة بين الفوتونات تبقى سنة ضوئية.

  160. إسرائيل
    كتبت لك أن تأخذ:
    "شاريل
    دعونا نتجاهل التسارع أولا. في الخطوة الأولى، أمام النقطة 1 0 كرونة وأمام النقطة 2 مليار كرونة.
    الآن - لنفترض أن قطارًا ثانيًا يتحرك بسرعة ثابتة تبلغ غاما = 2. في هذه الحالة، إذا كان هناك كرون x أمام النقطة 1، فسنرى أمام النقطة 2 كرون x زائد مليار مرة 2."

    المشكلة أنني لم أكتب "رقم السيارة x زائد مليار ضرب 2"؟ هل أنت في رياض الأطفال

  161. أنت مدعو لأمر المعجزات.

    ما هو الشيء الصعب جدًا في الإجابة على شيء مثل "ضريبة السيارة هي x + 2 مليار؟"

    لا يهم، أعتقد أن هذه هي إجابتك، ولكن هنا تكمن المشكلة: ضريبة السيارة x + 2 مليار كانت بعيدة عن الكاميرا XNUMX قبل حوالي ساعة ضوئية من بدء تسارع القطار.

    نظرًا لأن العملية بأكملها تستغرق 10 ثوانٍ فقط (من الوقت 0 إلى الوقت 10)، فليس لديه إمكانية الوصول إلى الكاميرا 2 في الوقت المناسب.

    ولا يهم كيفية اللعب بالبيانات، إذا وصلت إلى موقف حيث خلال 10 ثواني من بدء التسارع توجد سيارات أمام الكاميرتين في نفس اللحظة في كل منهما مع 2 مليار سيارة يفصل بينهما لهم، ثم وصل أحد الطرفين خلال 2 ثواني إلى سيارة كانت على بعد ساعة ضوئية منه.

    هل ترى خيارا آخر؟

  162. متوافق، متوافق

    ولكن لا علاقة لها بسؤالي. لا يوجد تسارع سواء. السؤال هو: ما هي السيارة التي سيتم تصويرها بالكاميرا رقم 2 في الساعة 10؟ سوف تلتقط صورة لبعض المقطورات، أليس كذلك؟

    لقد وصل القطار منذ فترة طويلة إلى التوازن وهو بالفعل نظام بالقصور الذاتي يتحرك بسرعة ثابتة تبلغ 0.87 درجة مئوية، أي أقل من الميونات الخاصة بك ويستغرق أيضًا وقتًا أطول بكثير لتسريعها.

    إذن ما هو رقم السيارة؟ ليس عليك أن تكون دقيقًا ولكن عليك أن تعطي تقديرًا معقولًا لأنه في نهاية المطاف، ما هي قيمة خمسين وستين مليون كرونة بين الأصدقاء؟

  163. لنجعل الأمر بسيطًا:

    في اللحظة 10 عند النقطة 1، تلتقط الكاميرا ذات الدقة العالية عند النقطة 1 صورة للسيارة x في القطار المقابل.

    ما رقم السيارة التي ستلتقطها الكاميرا عند النقطة 2 في اللحظة 10؟

    ولزيادة الطين بلة، لا تنسوا أنه وفقًا للتزامن، فإن القطار في الواقع يطول، ولا يقصر (هل تتذكر مفارقة بيل للسفن؟ لكن الإطالة لا تحل المشكلة المطروحة أيضًا).

    أياً كان الحل، ولا أجده حالياً، فهو غريب للغاية.

    وهذه هي النقطة التي ذكرتها فيما يتعلق بتقصير الطول، فهو غريب جداً.

    لذا حاول أن تقول ما سيكون رقم السيارة تقريبًا أمام الكاميرا 2 في اللحظة 10، وذلك دون مناقشة ما إذا كانت اللامكانية لا تنهار النسبية كما يدعي المقال في مجلة ساينتفيك أمريكان، وليس في الفيزياء البديلة.

  164. إسرائيل
    دعونا نتجاهل التسارع أولا. في الخطوة الأولى، أمام النقطة 1 0 كرونة وأمام النقطة 2 مليار كرونة.
    الآن - لنفترض أن قطارًا ثانيًا يتحرك بسرعة ثابتة تبلغ غاما = 2. في هذه الحالة، إذا كان هناك كرون x أمام النقطة 1، فسنرى أمام النقطة 2 كرون x زائد مليار مرة 2.

    وأنا أتفق مع ذلك.

  165. يا لها من معجزات حقا.

    1. في اللحظة 0 على ساعات السكة، عندما يكون القطار في حالة سكون، توجد السيارة رقم 1 أمام النقطة رقم 1 على السكة والسيارة رقم 2 مليار أمام النقطة رقم 1 على السكة وهي ساعة ضوئية واحدة بعيدا عن النقطة رقم XNUMX

    2. يتسارع القطار خلال خمس ثواني حسب ساعات القطار بحيث تكون غاما تساوي 2.

    3. والآن أخبرني: إذا كان هناك رقم سيارة x أمامها عند اللحظة 10 ثوانٍ وفقًا لنقطة الساعة 1 على المسار، فكم سيكون رقم السيارة التي تقف عند اللحظة 10 على المسار؟ المسار أمام النقطة 2؟ ليس س + 2 مليار؟

  166. إسرائيل
    حسنًا، أنت تقول أن القطار بدأ في حالة توقف تام، وتسارع خلال ثوانٍ إلى غاما = 2. أي من 0 إلى 260 مليون متر في الثانية. من وجهة نظر المراقب من الجانب، فإن طرفي القطارين يتحركان بسرعات مختلفة، والعربات الأخيرة أسرع بكثير من العربات الأمامية.

    مرة أخرى - أنا حقاً لا أرى أي تناقض هنا.

  167. المعجزات

    إنك لا تفهم الموضوع الرئيس.

    إذا كان في غضون ثوانٍ قليلة على ساعات المسار والتي هي أيضًا بضع ثوانٍ على ساعات القطار، فسيتم تقصير القطار بمقدار النصف، ثم إذا أخذت نقطتين على المسار وأخذت وقتًا معينًا على ساعات المسار (على سبيل المثال 0) ثم بين هاتين النقطتين عدد معين من السيارات (مثلا مليار) محصورين في البداية وبعد بضع ثوان في ساعات السكك الحديدية (مثلا 10 ثوان) 2 مليار.

    يمكنك رؤية ذلك بوضوح إذا كانت السيارات مرقمة: في اللحظة 0 توجد النقطة المقابلة 1 على المسار سيارة 1 وفي اللحظة 0 عند النقطة 2 على المسار مليار سيارة.

    في اللحظة 10 عند النقطة 1 على المسار ستكون هناك سيارة x أمامها وفي اللحظة 10 عند النقطة 2 على المسار ستكون هناك سيارة x + XNUMX مليار.

    من أين أتت كل السيارات في مثل هذا الوقت القصير؟ هذا فقط.

    إلبينتزو.

    ليس لدي أي نية للتجادل معك قبل أن تعتذر عن كل النقد اللاذع الذي وجهته إليّ في السنوات الأخيرة (تذكر؟ كاذب، أحمق، معتوه، جاهل، خرف، وما إلى ذلك). وهذا هو الشرط الأول فقط.

    التاريخ الحزين يثبت أنك تغضب فوراً عندما يختلف معك الرأي، فما فائدة النقاش؟ الدخول في معركة أخرى؟

    ولا، لم يكن أينشتاين مخطئًا في ورقة EPR، لكن ليس لدي أي نية للدخول في قتال مرة أخرى. غدا سنذهب إلى هاواي، آسف على الأعصاب.

  168. لقد استغرقت الآن بضع دقائق لقراءة المقالة من مجلة Scientific وإجراء القليل من البحث حول المصادر التي تشير إليها.

    بادئ ذي بدء، يجب أن نتذكر أن كل مقال في مجلة - حتى لو كانت علمية، وبالتأكيد في مجلة ساينتفيك أمريكان، التي تخاطب عامة الناس ولديها اهتمام كبير ببيع النسخ - يمكن أن يقع في ميل طبيعي للإثارة أو التبجح العناوين الرئيسية. ويحدث هذا أيضًا في المقالات العلمية. لذلك لا فائدة من النظر إلى العنوان فقط، وتحتاج إلى الرجوع إلى متن المقال.

    أول ما لفت انتباهي هو أن المؤلفين فيلسوف (فيلسوف علم بالتأكيد، ولكن على أي حال ليس عالم رياضيات أو فيزيائي) وكاتب. من الواضح منذ البداية أن هذا ليس بالضرورة مصدرًا جيدًا لتعلم الفيزياء منه. وسأستبق لاحقاً بأن أقول إنني لا أدعي أنهم لا يعرفون ما يتحدثون عنه، لأن معظم المقال يتعلق بالفلسفة وليس بالفيزياء، فلا مشكلة. لكن إذا كنا نتحدث عن الفيزياء، فلا ينبغي أن يكون هذا المقال أكثر من حافز لقراءة مقالات وأبحاث حقيقية لعلماء الفيزياء.

    والآن إلى المقال نفسه. الجزء الأول من المقال (حوالي 75%، لم أحص الصفحات) يقوم فقط بمراجعة تاريخية لخلفية التشابك والنسبية الخاصة وينتهي بالإجماع الكامل للفيزيائيين على أنه لا يوجد تناقض بين النسبية الخاصة وميكانيكا الكم . وبعد هذا الجزء يأتي الجزء الثاني الذي يتكون من ثلاثة أجزاء:

    1. ادعاء أن فيلسوفًا يُدعى مودلين كتب كتابًا في عام 94' أشار فيه إلى مشكلة (وليس تناقضًا كنقطة دقيقة) في الجمع بين النسبية الخاصة وميكانيكا الكم. وبصرف النظر عن حقيقة أن هذا الكتاب غير علمي ويخضع لمراجعة النظراء، فإن المشكلة لم يتم شرحها بدقة. يوجد شرح موجز لما لا تمثله المشكلة، ومن ثم تتم صياغة المشكلة بمتطلب "التزامن المطلق". أنا شخصيا لا أعرف كيف حدث ذلك، لكني لم أقرأ الكتاب.

    2. إشارة إلى عمل عالم رياضيات (الاقتباس الذي قدمه نيسيم سابقًا) والذي، بالنسبة للنظريات الحرة، يُظهر بشكل أكثر صرامة من فون نيومان أنه لا توجد مشكلة في الجمع بين التشابك والنسبية الخاصة.

    3. إشارة إلى عمل أحد المؤلفين. لم يتم شرح العمل بالتفصيل، ولكن من القليل الذي تم شرحه، يتضح أنه يسير على حبل مشدود رفيع وخطير للغاية، وهو إشارة إلى الهندسة التافهة للنسبية الخاصة. النسبية الخاصة هي نظرية هندسية وأي انحراف عنها، حتى ولو كان أصغرها، يتطلب استخدام النسبية العامة. النسبية العامة هي نظرية ليست كمية بشكل واضح (على عكس النسبية الخاصة). لذلك، لا يمكن تنفيذ الجمع بين ميكانيكا الكم والنسبية الخاصة إلا في ظل ظروف هندسية ثابتة لا تتفاعل أو تتقلب على الإطلاق. أعني أن هناك أشياء دقيقة جدًا هنا، والتي حتى العديد من الفيزيائيين الذين لا يدرسون الجاذبية يمكن أن يخطئوا فيها. ولكن حتى لو تم كل شيء بشكل صحيح، فإن المشكلة التي يطرحها هي مشكلة فلسفية بحتة يمكن حلها من خلال قبول مساحة التكوين كمساحة مادية. بمعنى آخر، أن نفهم أن تاريخنا موجود في فضاء هيلبرت وليس في الفضاء الزماني.

    في الختام، من الصعب جدًا بالنسبة لي أن أرى كيف يوجد في محتوى المقال أي خطر للجمع بين النسبية الخاصة وميكانيكا الكم. مزيج تم إنشاؤه بشكل واضح في نظريات المجال الكمي، والتي كانت متسقة رياضيًا (في هذا السياق) لمدة 60 عامًا تقريبًا والتي تصل دقتها التجريبية أحيانًا إلى ملايين بالمائة. لن آخذ العنوان على محمل الجد.

  169. إسرائيل
    لنفترض أن جميع عربات القطار بدأت في التسارع في الوقت t، وفقًا لساعة القطار (لا توجد مشكلة في مزامنة الساعات بهذه الطريقة). من وجهة نظر المراقب على الأرض - السيارات لا تبدأ بالتسارع في نفس الوقت. وعلى وجه الخصوص - سيرى المراقب على الأرض تسارع السيارة الأخيرة قبل تسارع السيارة الأولى.
    لذلك - غريب، ولكن لا يوجد تناقض هنا.

    سأحاول تبسيط القضية حتى تكون واضحة. لنتخيل أن هناك سيارتين متباعدتين فقط متصلتين بواسطة قضيب صلب، طوله ثانيتان ضوئيتان، والقطار يتحرك بسرعة v. يوجد في وسط العمود وميض - وكل سيارة تتسارع بمجرد رؤيتها للوميض. يتم تنشيط الفلاش في الوقت t. أما بالنسبة للسيارات - يصل الضوء إلى السيارتين في الزمن t+2، فيبدأان بالتحرك معًا وتبقى المسافة بينهما ثانيتين ضوئيتين.

    من وجهة نظر مراقب على الأرض. لنفترض أن المشاهد يرى الفلاش أيضًا في الوقت t (في حوالي....). بالنسبة له، يصل الضوء إلى السيارة الخلفية في الوقت (c+v)/1 وإلى السيارة الأمامية في الوقت (cv)/1. لذلك، تسارعت السيارة الخلفية أولاً وبالتالي يبدو القطار أقصر.

  170. إسرائيل،

    بالنسبة لسؤالك الأخير، لقد حصلت بالفعل على إجابة مني من قبل، ولكن كان ذلك منذ وقت طويل، لذا ربما لا تتذكر.

    على الرغم من أنك لا تريد التفكير في الأمر، إلا أن أينشتاين كان مخطئًا. هناك العديد من الأشياء غير المحلية في العالم، والعديد من الأشياء التي تتحرك بشكل أسرع من الضوء. لا يتناقض الجميع مع النسبية الخاصة. على سبيل المثال، طاقة النظام هي كمية غير محلية. من الأمثلة التي يتم تقديمها دائمًا في درجات البكالوريوس هو المصباح الذي يضيء على شاشة بعيدة جدًا عنه. ثم يتم إزاحة المصباح إلى اليسار بمقدار عشرين درجة، ويستغرق هذا التحول، على سبيل المثال، نصف ثانية. إذا كانت الشاشة بعيدة بما فيه الكفاية فإن بقعة الضوء ستتحرك مسافة تزيد عن نصف ثانية ضوئية، أي أنها ستتحرك بشكل أسرع من الضوء.

    اعتقد أينشتاين أن تغير الحالة الكمومية على مسافة ما ربما يكون بوساطة جسيم، وبالتالي فهو من عائلة الأشياء التي تتعارض مع النسبية. ما يجب القيام به، حتى أينشتاين مخطئ. وحتى الكثير. ولعلكم تعلمون أن أينشتاين كان ينشر مقالاً كل أسبوع أو أسبوعين، وكان فيما بعد ينشر لنصفها اعتذاراً وتصحيحاً لأنه اكتشف أنه كان مخطئاً. نحن نعلم اليوم أن ميكانيكا الكم في الواقع ليست دقيقة، والأشياء الأساسية في عالم الكم ليست جسيمات بل حقول. تسمح الحقول بالارتباط الذي لا تتوسطه الجسيمات ويمكن أن يكون غير محلي دون أن يتعارض مع أي شيء في النسبية الخاصة.

    لا أعتقد أن ذلك سيقنعك، ولكن إذا كنت تريد حقًا التعمق في الأمر - فلا يجب أن تتجاهل الإجماع الكامل الموجود اليوم (بعد أكثر من 80 عامًا من الأشخاص الأذكياء مثل أينشتاين الذين اكتشفوا الكثير من الأشياء). الأشياء التي لم يكن لديه أي فكرة عنها) فيما يتعلق بالموضوع.

  171. هههه.. يعني اهتميت بمقال هامشي وتخطيت الشيء الرئيسي؟

    يقول أينشتاين أننا إذا قمنا بتجربة معينة سنحصل على نتائج تتفق مع النسبية وبالتالي فإن نظرية الكم خاطئة.

    يتم تنفيذ التجربة. الكم صحيح. إذن ماذا عن العلاقة؟

    ودعونا نتجاوز حقيقة أنه لم يفهم معنى التجربة. إنه أينشتاين وهو من اخترعه.

  172. فكرت قليلاً في التفسير النسبي للتجاذب بين الكابلات الموصلة. من الواضح أن هذا أمر بالغ الأهمية، ولكن إذا فكرت بشكل أعمق قليلاً، فإن ثوابت الكهرباء والمغناطيسية تتجسد بالفعل في سرعة الضوء ولم تستخرجها المعالجة الرياضية النسبية إلا من هناك.

    وهذا على النقيض من نموذج ماكسويل الذي أخذ 3 ثوابت تبدو غير مرتبطة - ثوابت الكهرباء والمغناطيسية وسرعة الضوء - وربطها في صيغة أنيقة. إنها ضربة قاضية! (لا تنتقد الخردة!).

    لكن التفسير النسبي يوضح ما قلته عن تقصير الطول: وهو أمر غريب جداً.

    لرؤية ذلك، فكر في التفسير: بسبب حركة الإلكترونات، تزداد كثافة الشحنات.

    لذلك دعونا نفكر في قطار طويل جدًا يسير على مسار أطول. في لحظة معينة من ساعات القطار تتسارع جميع السيارات خلال ثوان معدودة حسب ساعات المسار بحيث يكون معامل جاما يساوي 2.

    وبحسب التفسير النسبي يتم تقصير القطار ومضاعفة كثافة السيارات.

    لكن هذا يعني أنه إذا أخذنا نقطتين على المسار على بعد ساعة ضوئية من النقطة التي يوجد بينها عدد معين من السيارات، ففي غضون ثوانٍ قليلة سيكون هناك ضعف عدد السيارات، أليس كذلك؟

    من أين أتت الكثير من العربات في مثل هذا الوقت القصير؟؟

    وكما قلت غريب جداً

  173. إسرائيل
    إذا لم تفهم، فأنا واحد من هؤلاء الأشخاص الذين يحاولون عدم التوصل إلى نظريات بديلة للأشياء التي لا أفهمها.

    هذه الأشياء التقنية اللطيفة هي بالضبط ما أدى إلى إسقاط العديد من النظريات البديلة - ومقالك عن نظام تحديد المواقع العالمي (GPS) هو مجرد مثال. يعد الاحتكاك في دفع الجاذبية تفصيلًا تقنيًا لطيفًا آخر، مما يبطل الفكرة برمتها.

    لا أفهم ما يكفي من الفيزياء أو علم النفس لشرح سبب إعطاء شخص ما عنوانًا معينًا لمقالة ما، خاصة عندما لا يتوافق محتوى المقالة مع العنوان بنسبة 100%.

    أنا لا أستبعد مناقشة هذه القضايا، ولا أقول أنه لا ينبغي عليك أن تتوصل إلى فكرة مجنونة. لكن هذا مفيد للتمرين الأكاديمي فقط ولا ينبغي مزجه بالعلم الحقيقي.

    من ناحية، فإنك تقدم جميع أنواع الأسباب لإبطال النظرية، ومن ناحية أخرى، في كل مرة تحاول فيها شرح سبب خطأ السبب - فهي مجرد تفاصيل فنية بسيطة. تعتبر المقالة المتعلقة بنظام تحديد المواقع العالمي (GPS) مثالًا جيدًا على ذلك - "لا تخلط بيني وبين الحقائق".

  174. المعجزات

    لقد كنت أبحث فقط على Google.. المقالة التي قمت بربطها سابقًا هي من مجلة Scientific American وهي بعنوان: تهديد كمي للنسبية الخاصة.

    إذن ما هي نظريتك حول سبب عدم الإشارة إلى عنوان المقالة ولكن التعامل مع التفاصيل الفنية اللطيفة؟

  175. إسرائيل
    المقالة التي قمت بربطها مثيرة للاهتمام. وهو مخطئ أيضًا. هناك سبب لقول الفيزياء البديلة هناك...

    وهذا يسلط الضوء على المشكلة في كل النظريات البديلة - نظريتك ونظرية يهودا وحتى نظريتي صديقي رافائيل: الافتقار إلى المعرفة. عندما يقول شخص يفهم شيئًا لا تفهمه (ألبانزو على سبيل المثال) فيمكنك التوصل إلى نظرية بديلة، أو يمكنك فتح كتاب والتعلم.

    إذا كان الأمر مثيرًا للاهتمام - يمكنني توضيح الخطأ الموجود في المقالة التي قمت بربطها بـ...

  176. رافائيل،

    1. حقيقة أننا نعرف بدرجة عالية جدًا من اليقين أن شيئًا ما لا يمكن أن يكون صحيحًا لأنه يتناقض مع ميكانيكا الكم (التي، على حد علمنا، وصف جيد جدًا للواقع)، لا تعني أننا نعرف ما هو. حقيقي. إذا كنت تريد الحصول على رؤى حول مسألة ما قد حدث في الكون المبكر جدًا وفقًا لنظريات الجاذبية الكمومية، فيمكنك العودة إلى الفيديو الذي أحضرته بنفسك إلى هنا قبل بضع سنوات حيث شرح فيزيائيون موهوبون للغاية ماهية أفكارهم وكيف تم يمكن اختبارها. اعتبارًا من اليوم ليس لدينا إجابة لا لبس فيها على سؤال ما حدث في بداية الكون بالنظر إلى ميكانيكا الكم.

    2. أعطيت مثالاً على نقطة المفرد لأنني شرحت لنسيم عن المفرد كموضوع عام في الرياضيات وليس في سياق الانفجار. ولم أقل أن هذا العنقاء يصف حجم الانفجار الكبير أو أنه مرتبط بالنموذج على الإطلاق. ولم أقل حتى أن مقياس الانفجار الكبير فردي. قلت إن النسخة المطابقة تكون مفردة عند النقطة a=0 لأنها تتحلل وتنسخ جميع نقاط الفضاء إلى نقطة واحدة. لا أريد أن أبدو متعاليًا أو متعاليًا، ولكن إذا كنت تريد فهم الجانب التقني لنموذج الانفجار الكبير (أو أي نموذج آخر للفيزياء الحديثة)، فعليك أن تدرسه. هناك العديد من الأشياء التي يمكن تفسيرها بشكل أو بآخر بلغة العلوم الشعبية، لكن الفهم التقني يتطلب معرفة الرياضيات وراء النموذج.

  177. ألبانزو

    آسف للعودة قليلاً ولكني ما زلت لا أفهم هذا:

    1. لقد كتبت "إن الكون بالحجم 0 ليس ضروريًا للنموذج ومن المحتمل أن يُفترض (في الواقع، مؤكد تمامًا) أنه لن يتم قبوله كحل لنظرية الكم لأسباب التوطين. "

    ماذا يعني ذلك في الواقع؟ أنه كان هناك دائمًا شيء ما، فقط أنه بدأ في التوسع في وقت معين؟

    2. فيما يتعلق بهذا "مثال تافه لطائر الفينيق المفرد حقًا هو أحد أجزاء z. ولها تفرد عند النقطة z=0."

    يجب أن يعطي التفرد مقياسًا صفرًا. 1/0 لا يعطي صفراً كيف يكون هناك تفرد هنا؟ (لقد حذفت موضوع اللانهاية لأنه في وقت سابق كان يحول كل الاهتمام عن النقطة الرئيسية)

  178. المعجزات

    هناك مليون نظرية، انظر على سبيل المثال

    http://www.alternativephysics.org/book/GPSmythology.htm

    خلاصة القول، ألا توافق على أن عدم المحلية يشكل مشكلة صعبة بالنسبة للنظرية النسبية؟

    لذا اشرح عنوان مقالة مجلة Scientific American (تظهر النسخة العبرية أيضًا في هيدان).

  179. إسرائيل
    لم أفهم ما كنت تقوله عن الميونات. تم اختبار تباطؤ الوقت في كلا الاتجاهين - باستخدام الأقمار الصناعية لنظام تحديد المواقع العالمي (GPS) على سبيل المثال.

    مرة أخرى، تقول مقالتك أن هناك تفسيرًا محتملاً لـ EPR لا يتعارض مع النسبية.

    أنا لا أقول أن هذه التوراة أو تلك صحيحة. وأنا أقول أن هناك دليلاً على قصر المسافة.

  180. المعجزات

    هل قرأت الفقرة الأخيرة في المقال؟

    حالة النسبية الخاصة، أكثر من مجرد
    بعد قرن من تقديمه للعالم، هو
    فجأة مفتوحة بشكل جذري وتتطور بسرعة
    سؤال. لقد حدث هذا الوضع بسبب
    وأخيراً تبعه الفيزيائيون والفلاسفة
    من خلال النهايات الفضفاضة التي أهملها أينشتاين لفترة طويلة
    حجة مع ميكانيكا الكم

    يمكنني أن أبدأ في محاولة شرح كل سؤال تطرحه - على سبيل المثال، تم تسريع الميونات التي ذكرتها وتتحرك مقابل إشعاع الخلفية، بينما وفقًا للنسبية فهي أيضًا في الوضع المعاكس، أي إذا كانت ثابتة بالنسبة لإشعاع الخلفية حتى فإن ساعتهم الداخلية ستكون أبطأ بالنسبة للنظام المتزامن الذي يكون في حركة نسبية للإشعاع، وهذا لم يتم إثباته في أي تجربة.

    ويمكنني أن أشير أيضًا إلى أن نظرية نيوتن تعمل أيضًا وتمت دراستها وثبت صحتها ودقتها منذ مئات السنين، ومع ذلك فهي ليست صحيحة بل هي حالة خاصة من النسبية عند السرعات المنخفضة.

    لكن الحقيقة هي كيف أعرف أنا أو أي شخص. ليس من المفترض أن نعرف أيضاً. أعتقد أننا نتفق على شيء واحد: أينشتاين يكتب ورقة بحثية - EPR - يحاول فيها إظهار أن ميكانيكا الكم خاطئة أو غير كاملة لأنها (وفقًا لـ Wiki على الأقل) تتعارض مع النسبية. كما يقدم تجربة فكرية لإثبات وجهة نظره.

    تم إجراء التجربة في الواقع (شيماوني، أسبكت)، وكانت النتائج متوافقة مع ميكانيكا الكم.

    إذن ماذا يقول هذا عن العلاقة؟

  181. إسرائيل
    اقتباس من المقال الذي قمت بربطه:
    وظهرت النتيجة الأولى بشكل مذهل
    ورقة بحثية عام 2006 بقلم رودريش تومولكا، شاب ألماني
    عالم الرياضيات الآن في روتجرز. تومولكا
    أظهر كيف أن جميع التنبؤات التجريبية ل
    ميكانيكا الكم لأزواج الجسيمات المتشابكة
    يمكن إعادة إنتاجها عن طريق تعديل ذكي
    لنظرية GRW (تذكر أن هذه النظرية
    يقترح طريقة واقعية فلسفيا للحصول على
    تنبؤات ميكانيكا الكم في ظل الكثير
    ظروف). التعديل غير محلي
    ومع ذلك فهو متوافق تمامًا مع الزمكان
    هندسة النسبية الخاصة.

    إقرأ الجملة الأخيرة مرة أخرى....

    لكن دعنا نذهب - من فضلك اشرح لي كيف يسافر الميون مسافة 6000 متر في 2 ميكرون، دون تقصير المسافة.

  182. إسرائيل
    اذا هيا بنا نبدأ
    1) فيما يتعلق بالتقصير. لنفترض أن حجم النجم هو بحيث يمكن أن يكون هناك كسوف دقيق للنجم بالقمر. وهكذا نقطة على محيط القمر والنقطة المقابلة لها على النجم البعيد. تقع هذه النقاط على خط مستقيم منا، مثلًا بزاوية مخروطية أ. عند أي سرعة v للراصد (باتجاه مركز القمر/النجم)، تكون السرعة إلى هاتين النقطتين هي v مرات جيب التمام a. وهذا يعني أن المسافة إلى النقطتين سيتم تقصيرها بنفس العامل، وبالتالي ستظلان على نفس الخط. لتوضيح ذلك - ارسم الوضع على سطح بلاستيكي، ثم قم بتمديد البلاستيك على طول محور واحد. الامتداد عبارة عن تحويل تقاربي، ومن خصائصه أن النقاط الواقعة على خط مستقيم ستبقى على خط مستقيم.

    2) ما الجديد؟ إذا كان لدينا تعليمان متعارضان، أحدهما بسيط والآخر معقد للغاية، فمن المرجح أن يكون التعليم المعقد خاطئًا. نأمل أن نتمكن من العثور على نظرية تجمع بين الملاحظات من كلا المجالين.

    3) يعتمد حساب ماكسويل لسرعة الضوء على ثابتين (epsilon-0 وmu-0) وليس لديه تفسير لهما. وفقًا لماكسويل، يجب أن تعتمد سرعة الضوء على السرعة "المطلقة" لمصدر الضوء، وهذا ليس له أساس رصدي.
    تعمل معادلات نيوتن أيضًا في ظل ظروف معينة. …

    4) نحن نعلم أن هناك إطار مرجعي مفضل للكون، وهذه هي بالضبط درجة الحرارة التي تتحدث عنها. وهذا لا يتعارض مع النظرية النسبية.

    5) من الناضج جدًا منك أن تستشهد كدليل بأقوال شخص لا تتفق مع رأيه 🙂 أينشتاين كان يؤمن بالمحلية، والملاحظات تظهر أنه كان مخطئًا.

    إسرائيل - هل لديك تفسير آخر لسبب انجذاب الأسلاك الموصلة لبعضها البعض؟ واسمحوا لي أن أعطيكم مثالاً آخر.

    يتم إنشاء الميونات عن طريق تصادمات الأشعة الكونية في الغلاف الجوي العلوي (حوالي 20,000 قدم)، ويبلغ متوسط ​​عمرها حوالي 2 ميكروثانية. سرعتها حوالي غاما = 10. بالنسبة لنا - تعيش 20 ميكرون، ويمكنها التحرك 6000 متر، مما يعني أنه يمكننا اكتشاف بعضها على الأرض.
    لكن - بالنسبة لهم، فإنهم يعيشون على مسافة 2 ميكرو فقط! كيف يقطعون مسافة 6000 متر؟؟ هممممم…. ماذا سيحدث إذا قسمنا المسافة على 10؟ سنحصل على 600 متر 🙂 وهنا - حوالي نصفهم سيصل بالفعل إلى الأرض المقدسة.

    إذا كنت لا توافق على التفسير - ما هو تفسيرك؟

  183. إسرائيل
    ومن الممكن أن يصل كل إلكترون إلى الإلكترون الموجود في السلك الآخر، وبالطبع لا يمكنه فعل ذلك إلا عندما يتحرك كلا الإلكترونين في نفس الاتجاه.

    إن التنبؤ النسبي لتقصير الطول في النسبية الخاصة هو نتيجة مباشرة لنظرية ناجحة للغاية، والتي لا تتعارض مع أي ملاحظة موجودة، بل إنها سهلة الفهم. وهناك أيضًا ملاحظات تؤكد هذا التوقع.

    لماذا لا تحصل عليه؟ تقصير الزمن وانحناء المكان نعم، وتقصير الطول بالتحديد لا؟

    نعلم جميعًا أن النسبية ونظرية الكم لا تتطابقان بشكل جيد مع بعضهما البعض، ولكن لماذا يزعجك تقصير الطول بالضبط؟

  184. QED هي نظرية نسبية وتتضمن في داخلها كل النسبية الخاصة، بما في ذلك الاختصارات/الإطالات المختلفة. ربما أنا في عداد المفقودين هذه النقطة.

    وفقط للمعرفة العامة - المعجزات، فإن إثبات مخطط العمود الطبيعي لناقص واحد مقسومًا على اثني عشر بمساعدة معالجة الأعمدة التي لا تتقارب بشكل صارم (مثل على سبيل المثال 1-1+1-1+...) هو إرشادي ولطيف ولكنه يعتبر بمثابة دليل على الألعاب في الرياضيات. وفقًا للمعايير الحديثة، لن يُطلق عليه دليلًا، لأنه يفترض أن جبر المتسلسلة (التي ليست متقاربة تمامًا) محدد جيدًا وهذا ببساطة غير صحيح. هناك بالطبع أدلة صارمة تؤدي إلى نفس الإجابة التي يمكن الوصول إليها ببساطة كما اقترحت. على سبيل المثال، من خلال متابعة العمود تحليليًا وإجراء التنظيم باستخدام دالة زيتا لريمان.

  185. قد تقصد هذا:

    http://galileo.phys.virginia.edu/classes/252/rel_el_mag.html

    جميلة ومثيرة للإعجاب. لكن سؤالنا كان: هل هذا هو التفسير الوحيد الذي يتبادر إلى ذهننا بعد ذلك؟

    وهنا من ويكي:

    الديناميكا الكهربائية الكمومية[عدل]
    أنظر أيضا: النموذج القياسي والديناميكا الكهربائية الكمومية
    في الفيزياء الحديثة، يُفهم المجال الكهرومغناطيسي على أنه ليس مجالًا كلاسيكيًا، بل هو حقل كمي؛ ولا يتم تمثيله كمتجه لثلاثة أرقام عند كل نقطة، ولكن كمتجه لثلاثة عوامل كمومية عند كل نقطة. الوصف الحديث الأكثر دقة للتفاعل الكهرومغناطيسي (وأشياء أخرى كثيرة) هو الديناميكا الكهربائية الكمومية (QED)،[37] والتي تم دمجها في نظرية أكثر اكتمالا تعرف باسم النموذج القياسي لفيزياء الجسيمات.
    في QED، يتم حساب حجم التفاعلات الكهرومغناطيسية بين الجسيمات المشحونة (والجسيمات المضادة لها) باستخدام نظرية الاضطراب. تنتج هذه الصيغ المعقدة نوعًا ما تمثيلًا تصويريًا رائعًا مثل مخططات فاينمان التي يتم فيها تبادل الفوتونات الافتراضية.

  186. الصيغ هي قلب المسألة.

    نحن نتحرك لاستقبال السبت، وفي هذه الأثناء حاول حساب عامل جاما عندما يكون v أقل من متر في الثانية (سرعة الإلكترونات في الموصل).

    إذا كنت تفتقد صفحات للأصفار بعد العلامة العشرية، فيمكنك العثور على بعض الغابات غير المستخدمة في كندا القريبة.

  187. إسرائيل
    النظر في مسألة المغناطيسية. هل يمكنك تفسير التجاذب بين الأسلاك المتوازية بطريقة أخرى؟
    تم شرحه بشكل جيد في كتاب بيركلي - وهنا مقطع فيديو قصير يشرح، بدون الصيغ...

  188. المعجزات

    أحضر التجربة المحددة، وسنرى ما إذا كان هذا هو الخيار الوحيد حقًا.

    وأيضاً تجربة الاختيار المؤجل لفايلر:

    https://www.hayadan.org.il/quantun-philospy-part-b-07121

    تم تفسيره على أنه تأثير على الماضي من المستقبل، على الرغم من أن التفسير الأكثر منطقية في رأيي هو التواصل السريع في أوريت.

  189. إسرائيل
    من ويكي:
    لا يمكن تفسير سلوك تصادم الأيونات الثقيلة إلا إذا تم أخذ كثافتها المتزايدة بسبب تقلص لورنتز في الاعتبار. ويؤدي الانكماش أيضًا إلى زيادة شدة حقل كولوم بشكل عمودي على اتجاه الحركة، وهو ما تمت ملاحظة آثاره بالفعل. وبالتالي، يجب أخذ كل من تمدد الزمن وانكماش الطول في الاعتبار عند إجراء التجارب في مسرعات الجسيمات.

    يفسر التقصير الطولي أيضًا سبب انجذاب موصلين متوازيين لبعضهما البعض.

  190. المعجزات

    حسب الرابط لتأثير سينياك

    https://en.wikipedia.org/wiki/Sagnac_effect#Relativistic_derivation_of_Sagnac_formula

    وكما هو الحال دائمًا، يمكن تفسير ذلك على أنه تقصير الطول وإطالة الأزمنة.

    أين هو التفسير الوحيد لمسارات الأيونات بعد الاصطدام؟ لم أرى الرابط الذي قدمته.

    إذا أحضرت مفهومًا رياضيًا مع تفسير منطقي، فسأقبله بكل سرور. لكن القول بأن 1+2+3+4+5+….. = 1/12-، ومن ثم إحضار رابط لا يوجد فيه أي شيء مكتوب، هو أمر مثير للسخرية. ربما حتى سحقت.

  191. إسرائيل
    لاحظ كلمة مباشر ليس لدينا إمكانية تقنية للتحقق من التغير في طول الجسم السريع. لكن هذا هو التفسير الوحيد لمسارات الأيونات بعد الاصطدام. هذا ما تقوله ويكيبيديا. بشكل عام، إذا فكرت في الأمر، فمن الصعب جدًا قياس الطول من نظام حركة آخر.

    الطائرات (على الأقل الطائرات المقاتلة) لديها نظام يسمى RLG. ويعتمد هذا النظام على ظاهرة سانياك وهي نتيجة تغير طول مسار الليزر بسبب الحركة الدائرية. في الصيغ التي تشرح الظاهرة يتم إدخال تغير الطول بسبب الحركة.

    لا أفهم لماذا تقلل من شأن المفهوم الرياضي. يتم أيضًا اقتطاع جذر ناقص 1، ولكنه يستخدم كثيرًا في الفيزياء، وحتى في الكهرباء.

  192. المعجزات

    وفي الرابط الخاص بالتجمع ليس هناك أي تفسير أو دليل، بل مجرد ادعاء والكثير من الادعاءات المضادة بأنه مجرد هراء.

    وفي الرابط أيضا عن تقصير الطول لم أجد شيئا عن الحمام أو عودتك ولا دليل على تقصير الطول. ومن جهة أخرى في الرابط التالي:

    http://math.ucr.edu/home/baez/physics/Relativity/SR/experiments.html#Length_Contraction

    ومكتوب في التفسير:

    في هذا الوقت لا توجد اختبارات مباشرة لانكماش الطول

  193. قرر، 1/12 - أم 1/12؟

    لا هذا ولا ذاك.

    الفكرة التي أفكر فيها، سيزارو قد تناثرت، إذا راهنت بالمال على العمود المتجمع إلى النصف، فسوف تخسر. حتى لو وضعت السلحفاة في منافسة جري مع أخيل أو روزان بير في مسابقة جمال مع ساندي بير.

    أين الرابط من الويكي عن الحمام؟

  194. إسرائيل
    لا داعي لإضافة أي شيء... العمود لا نهائي وقيمته سالب 1/12... والدليل بسيط لكنه يعتمد على التقارب الذي ذكرته (1-1+1-1+1-1+1-1+1 = 0.5). فكرة سيزارو هي النظر إلى حد القيمة المتوسطة.

    ونعم - هناك تجارب تظهر قصر الطول. إذا نظرت إلى تصادمات الأيونات الكبيرة، فإن الطريقة الوحيدة لشرح زوايا الجسيمات هي تقصير الأيونات. انظر ويكيبيديا….

  195. 1+2+3+4+5+….. = 1/12- فقط إذا أضفنا حداً موجباً على الجانب الأيسر من المعادلة، أو حداً سالباً على الجانب الأيمن حيث النقاط.

    النسبية - هل تعرف أي تأكيد قاطع على تقصير الطول مثل تجربة الطائرات من أقاصي الأرض التي أثبتت إطالة الزمن؟

  196. إسرائيل
    يستطيع ألبينزو أن يشرح لك السبب فيزيائياً 1+2+3+4+5+….. = 1/12-
    ليست بديهية ولكنها مكتوبة في كتاب فيزياء (لكاتبه بولتشينسكي) (اعتقدت أن الكلمة المناسبة في العبرية هي "السبب"، لكنك على حق في أن هذا ما قصدته).

    أستطيع أن أشرح لك رياضيا لماذا هذا صحيح ….

    وأما القطار، فبالنسبة للركاب لم يتم تقصير القطار. وهذا ينطبق فقط على مراقب خارجي.

  197. المعجزات

    هذا ما قاله لايبنتز وربما كان صحيحا فلسفيا. جسديًا، سوف تحصل دائمًا على واحد أو اثنين (كما هو الحال في الكميات).

    وفيما يتعلق بالقسمة على 0: عند الاقتراب من سرعة الضوء، تزداد كتلة الجسم وتتجه إلى ما لا نهاية حسب تحويل لورنتز. عندما تصل إلى سرعة الضوء و v تساوي c، تحصل على القسمة على 0، لكن في هذه الحالة يكون حاصل القسمة يساوي ما لا نهاية وليس سالب ما لا نهاية وبالتالي يتم تعريفه.

    وهذا على النقيض من الدالة واحد مقسومًا على x والتي تهدف أيضًا إلى سالب ما لا نهاية. من الممكن التمييز بين الرياضيات والفيزياء (كما يفعل نيسيمينو في مفارقة زينون)، لكن إذا كان همنا هو الفيزياء، ففيزيائيا يتم حل مفارقة زينون (اللانهاية)، ويتم تحديد كتلة الجسم عند سرعة الضوء (اللانهاية). ).

    وبما أننا عدنا إلى الفيزياء (بلا نهاية!)، فكما ذكرت فإن النسبية هي نظرية جميلة ومثالية في حد ذاتها، مع مشكلة هامشية وهي أنها لا تتفق مع ما نعرفه من نظريات الكم، والانفجار، والأوتار، ماكسويل...

    ولا يعني ذلك أن الأمر غير منطقي (كان نسيم يعني البديهة، على ما أعتقد)، ولكن مع ذلك، قطار يبلغ طوله كيلومترًا مضغوطًا إلى سنتيمتر واحد دون علم الركاب؟

  198. إسرائيل
    يمكن أن يكون مجموع العمود الذي قلته أيضًا 1.5، إذا قمت بتحديد المخطط بالطريقة المناسبة. يُسمى هذا المخطط مخطط سيزارو، وهذا المخطط محدد جيدًا ومفيد حتى في الفيزياء.

  199. رافائيل
    اللانهاية محددة جيدا. وكما كتبت، هناك عدة أنواع من اللانهاية، وهناك أيضًا تعريفات مختلفة لنفس "النوع" من اللانهاية.
    دعونا ننظر إلى الأعداد الطبيعية للحظة. إحدى طرق النظر إليها تسمى الأعداد الترتيبية. والمعنى أن الأعداد الطبيعية تشكل سلسلة، وهناك أول وثاني وثالث وهكذا. السلسلة عبارة عن مجموعات - 0 هي المجموعة الفارغة، 1 هي المجموعة التي تحتوي على المجموعة الفارغة (تحتوي على العضو 1). 2 هي المجموعة التي تحتوي على 0 و 1 (أي أنها تحتوي على عضوين - المجموعة الفارغة والمجموعة التي تحتوي على المجموعة الفارغة). وفي هذه الطريقة يوجد تعريف لللانهاية - ويسمى أوميغا. أوميغا هو رقم لكل شيء. ويمكنك إضافة أرقام إلى أوميغا وضربها وحتى زيادة قوتها. وأي إجراء من هذا القبيل محدد بشكل جيد.

    الطريقة الثانية هي من خلال أرقام المشاركة (وتسمى أيضًا أوتزما). الرقم 3، على سبيل المثال، يتم تعريفه من قبل جميع المجموعات التي لديها 3 أعضاء (أو - قوتها 3). يبدو هذا التعريف دائريًا، ولكن هذا ليس هو الحال: المجموعة "أ" والمجموعة "ب" لهما نفس عدد الأعضاء إذا وفقط إذا كان من الممكن العثور على دالة ذات قيمة واحدة ومن كل مجموعة إلى أخرى مجموعة.
    للتوضيح، إليك مثال لطيف - "X" هي مجموعة الأعداد الطبيعية، و"Y" هي مجموعة الأعداد الزوجية. قوة "X" هي A-0، ولكن ما هي قوة الاثنين؟ لكل x من X، يمكن تحديد y المقابل - قيمته هي 2x. لكل y في Y، يمكن تعريف x المقابل على أنه y مقسومًا على 2 (تذكر أن y بالضرورة زوجي). لذلك أثبتنا أن قوة الأعداد الزوجية هي أيضًا A-0.

    بنفس الطريقة من الممكن إظهار أن قوة الأعداد النسبية (الكسور التي يكون فيها البسط والمقام طبيعيين) هي a-0، وكذلك قوة الأعداد الجبرية (معادلات مثل 7x^4 + 3x – 19 = 0) – أيضًا أ-0.

    وإذا أردت، هناك أيضًا تعريف للأرقام B، ولا نعرف ما إذا كانت الأرقام B مختلفة عن الأرقام A.

  200. لقد ارتكبت فوضى في الفقرة قبل الأخيرة. كتبت أنني اخترت وضع x تربيع في المقام وإسرائيل x في المقام الرابع، ولكن بعد ذلك عندما تحققت مما يحدث عندما تضرب في x تربيع، قمت بعكس المولد وتعاملت مع المشكلة كما لو أنني اخترت x في الرابع و إسرائيل × في الساحة. آمل ألا يكون هذا مربكًا للغاية.

  201. سأحاول إلقاء بعض الضوء على موضوع القسمة على صفر إذا كان الأمر يهم الناس، على الرغم من أنني آمل أن يكون واضحًا أننا قد ابتعدنا تمامًا عن موضوع متفردة الانفجار الكبير.

    القسمة على الصفر غير محددة حسابيا للسبب الذي ذكره نسيم. رياضياً، يتم تعريف القسمة ببساطة على أنها الضرب في المعكوس - على سبيل المثال، القسمة على اثنين هي الضرب في رقم هو معكوس اثنين (الرقم الذي إذا ضربته في 2 تحصل على 1، وهو بالطبع النصف). . الصفر ليس له معاكس (لا يوجد رقم إذا ضربته في 0 تحصل على 1) وبالتالي فإن حساب العملية غير محدد.

    يمكنك إلقاء نظرة على النهج الوظيفي. بدلًا من القسمة على الصفر، سنقسم على رقم ما (على سبيل المثال المتغير x) ونأخذ الحد الذي يصل إليه الرقم صفر. مثال بسيط هو أخذ الدالة الخطية x نفسها. تقول إسرائيل (وهذا صحيح) إن الحدود ليست محددة بشكل جيد لأنها تعتمد على ما إذا كان المرء يسعى إليها من اليمين أو من اليسار. وإذا أخذنا النهاية على اليمين، نرى أنه كلما اقتربنا من 0، يزداد حاصل القسمة دون حد أعلى، أي لا توجد نهاية منتهية والتعبير يتباعد - وهذا هو سبب الحدس الذي يقول 1 مقسوما على 0 يعطي ما لا نهاية. إذا تطلعنا إلى النهاية الموجودة على اليسار، فسنحصل على سالب ما لا نهاية. ولكن ليس صحيحا أن الجرعة غير محددة بشكل جيد. بعد كل شيء، إذا أردنا، يمكننا ببساطة أن ننظر إلى نقطة مختلفة - على سبيل المثال، علامة X في المربع، ولا يهم إذا أخذنا الحدود من اليسار أو اليمين. المشكلة كانت في انقطاع الإشارة وكان من الممكن حلها بسهولة.

    إذن ما هي مشكلة النهج الوظيفي؟ المشكلة بالطبع هي الحساسية لاختيار الهاتف. تريد إسرائيل التحقق من 1 مقسومًا على 0، لذلك اختار وضع التعبير x في المقام والتحقق مما يحدث عندما يسعى للحصول على 0. اخترت أن أضع x في المربع. لا أحد منا على حق من الآخر، وكلانا سوف يهدف إلى التعبير 1 مقسومًا على 0، لكننا سنحصل على نتائج مختلفة. على سبيل المثال، ستواجه إسرائيل مشكلة الإشارة وأنا لن أفعل ذلك. وهذا ليس سوى غيض من فيض.

    لنفترض أنني اخترت وضع x في المربع الموجود في المقام واختارت إسرائيل x في المربع الرابع. لا يواجه أي منا مشكلة في الإشارة، وكلانا يحصل على تباعد حيث تميل x إلى 0. في ظاهر الأمر، يبدو أننا نتفق على أن الجزء لانهائي. لكن الترفيه ليس شيئًا ثنائيًا (يوجد ترفيه أو لا يوجد ترفيه)، بل هو سلسلة من السلوكيات (أي نوع من الترفيه، ما مدى سرعة الترفيه). على سبيل المثال، يمكنك أن تسأل ماذا يحدث إذا ضربنا هذه اللانهاية التي حصلنا عليها في 0. يمكن لإسرائيل أن تقول: "حسنًا، بدلاً من وضع 1 في البسط، سأضع الحرف x في المربع (لتجنب مشاكل الإشارة) وأخذ النهاية. الجواب يخرج 1 ". ومن ناحية أخرى، إذا فعلت الشيء نفسه وضربت في x مربع، سأحصل على أن الجواب لا يزال لا نهاية، لأن الترفيه الذي حصلت عليه أقوى من الترفيه الذي حصلت عليه إسرائيل.

    من الناحية الحسابية، العملية ببساطة غير محددة. وظيفة العمليات محددة جيدًا، ولكنها تتطلب المزيد من المعلومات ويمكن أن تعطي نتائج مختلفة تمامًا اعتمادًا على مدى سرعة ميل البسط في المقام إلى الصفر. في الواقع، هناك مجال كامل ومتطور في الرياضيات يتعامل بشكل حصري تقريبًا مع هذا السؤال ضمن إطار التحليل المركب.

  202. رافائيل

    يمكنك العثور على تعريف اللانهاية في الويكي.

    ما هي مشكلة اللانهاية؟ ما هو غير الواضح حول اللانهاية؟

    ومن ويكي القسمة على 0:

    "القسمة على الصفر هي العملية الرياضية لقسمة رقم على الرقم 0، وغالبا ما تكون نتيجتها غير محددة."

  203. 1 أو 2 (مسلسل الشيطان إليك..).

    رافائيل

    لماذا اللانهاية غير محددة؟ وأين الرقي؟ إذا قسمت على 0، فستحصل على رقم كبير جدًا (اللانهاية) وسالب هذا الرقم، وليس الرقم الأول فقط. ولهذا السبب لم يتم تعريف القسمة على 0.

  204. ألبانزو
    لقد فهمت ما تقوله. إذا أخذت، على سبيل المثال، مجالًا من درجات الحرارة وقمت بإجراء تحويل خطي عليه (على سبيل المثال، الضرب بـ x)، فمن المستحيل أن x لن تكون 0. على سبيل المثال، x هو جيب الزمن.

  205. إسرائيل
    أعني منطقيًا بالمعنى - الفطرة السليمة. بالنسبة للشخص العادي، ليس من المنطقي أن تتغير وتيرة الزمن، أو أن يتحرك الضوء للأمام. كما أنه ليس "منطقيًا" حيث لا تكون الزمانية شيئًا محددًا.

    ترجمة "لا معنى له" إلى أنه لا معنى له.

    رافائيل
    لقد قلت أن 1/0 يساوي اللانهاية. إنها مسألة تعريف، ولكنها ليست التعريف المقبول في الرياضيات. تعريف القسمة هو مقلوب الضرب. أي أن a/b = c إذا وفقط إذا كانت a = b*c. وهو غير موجود في التعريف الذي قدمته. يمكنك أن تقرر أن 0 مضروبًا في ما لا نهاية يساوي 1، ولكن تظهر مشاكل أخرى على الفور: اضرب طرفي المعادلة في 7، وستحصل على 1=7.

    لقد قلت أن اللانهاية غير محددة. هذا غير صحيح. على الرغم من أن الأمر يبدو غريبًا، إلا أنه تم تعريف عدة أنواع من اللانهاية. A-0 (نعم، باللغة العبرية) هي قوة الأعداد الطبيعية (عدد الأعضاء في مجموعة الأعداد الطبيعية). A-1 (والذي يسمى عادةً c، أو قوة التسلسل)، هو عدد الأعداد الحقيقية بين 0.0 و1.0 - وهو أكبر من a-0 (بالطبع تحتاج إلى تحديد ما هو "الأكبر"، و هناك تعريف لذلك).

    وينبغي للمرء أن يكون حذرا مع كلمة اللانهاية، وألا يخلط بين فكرة كبيرة جدا أو صغيرة جدا. عندي "معضلة":

    כמה זה 1+1-1+1-1+1-1+1-1+1-1…… עד אינסוף?

  206. رافائيل

    "حتى الآن كنت أعلم أن 1 مقسومًا على صفر يساوي ما لا نهاية."

    لماذا لا ناقص اللانهاية؟ لا يعتمد الأمر على الجانب الذي تهدف إلى القسمة على صفر؟

    انظر على سبيل المثال…….000000000000000000000000000000-/1

    كلما كان الرقم السالب أصغر، يميل حاصل القسمة إلى ناقص ما لا نهاية، بينما إذا كان الرقم موجبًا، فإنه يميل إلى ما لا نهاية.

    إذن كم يكون المبلغ عندما يكون القسمة 0 بالضبط؟ ما لا نهاية أو ناقص ما لا نهاية؟

    وهذا يعني: غير محدد.

  207. ألبانزو

    1. لقد كتبت "إن الكون بالحجم 0 ليس ضروريًا للنموذج ومن المحتمل أن يُفترض (في الواقع، مؤكد تمامًا) أنه لن يتم قبوله كحل لنظرية الكم لأسباب التوطين. "

    ماذا يعني ذلك في الواقع؟ أنه كان هناك دائمًا شيء ما، فقط أنه بدأ في التوسع في وقت معين؟

    2. فيما يتعلق بهذا "مثال تافه لطائر الفينيق المفرد حقًا هو أحد أجزاء z. ولها تفرد عند النقطة z=0."

    حتى الآن كنت أعلم أن 1 مقسومًا على صفر يساوي ما لا نهاية. إذًا ربما في النهاية هناك علاقة في الفيزياء أو الرياضيات بين الصفر المطلق واللانهاية؟

  208. في الواقع، طائر الفينيق الذي يمكنك أن تفعل به ما كتبته في ردك (تعيين قيمة لهم عند نقطة المفرد عن طريق أخذ حد من اليمين واليسار) عادة لا يعتبر طائر الفينيق المفرد حقًا، ولكن فقط طائر الفينيق الذي لم يتم محددة بشكل صحيح. مثال تافه على هاتف مفرد حقًا هو أحد الأجزاء z. ولها تفرد عند النقطة z=0.

  209. نعم. لا يعني ذلك أن هناك فرقًا بين الفيزياء والرياضيات، بل حتى في الرياضيات ما كتبته ليس دقيقًا. أولاً، أنت تتحدث عن نقطة انقطاع المقاصة، وهي حالة خاصة جدًا ومحددة لنقطة فريدة من Pon'، وليست الحالة العامة حقًا. ثانيًا، أنت تتحدث بشكل خاص عن تفرد بون. يمكن أن يظهر التفرد في phon وأيضًا في كائنات رياضية أخرى، خاصة في العوامل أو النسخ.

    في حالة الانفجار الكبير، يكون التفرد في النسخة المطابق. هناك طريقة لطيفة للتفكير في الأمر وهي أنك أخذت مساحة مسطحة (أي مقياسها ثابت 1 ومستقل عن الزمن)، وقمت بتطبيق بعض النسخ عليها مما أدى إلى تمديدها وإعطائك مساحة مختلفة. على وجه التحديد بالنسبة لـ a=0، يتم الحصول على انحطاط لأن الفضاء يتحول من الفضاء ذي الأبعاد d إلى الفضاء ذي الأبعاد 0، ويقوم النسخ فعليًا بنسخ جميع النقاط الموجودة في الفضاء إلى نفس النقطة.

    من حيث المبدأ، يمكن أيضًا تفسير تفرد الجاذبية على أنه نقطة مفردة في طائر الفينيق (على سبيل المثال، إذا تم كتابة الانحناء العددي على أنه طائر الفينيق لمعلمة معينة، فإن تفرد الجاذبية يمكن أن يكون انحرافًا في هذا العنقاء). ولكن كما قلت سابقًا، حتى في هذه الحالة لن يكون انقطاعًا للتخليص (أي يمكن إعادة تعريفه كحدود للفون في البيئة المحيطة بالنقطة). بالإضافة إلى ذلك، في بعض الأحيان تحتوي هذه النقاط على نقاط فردية لا تعزى إلى تفرد الجاذبية (على سبيل المثال، تلك التي تنشأ ببساطة من الفهم السيئ لنظام الإحداثيات).

  210. ألبانزو
    عادة في الرياضيات، يتم تعريف معنى قيمة الدالة عند نقطة مفردة على أنها نهاية الدالة المستمرة. على سبيل المثال، sin(x)/x يساوي 1 عندما يكون x=0، بمعنى أنه يمكننا أن نقرب x من 0 كما نريد. فهل المعنى في الفيزياء، وخاصة في النسبية العامة، مختلف؟

  211. إن الكون بحجم 0 ليس ضروريًا للنموذج ومن المحتمل (في الواقع، مؤكد تمامًا) أنه لن يتم قبوله كحل لنظرية الكم لأسباب التوطين. إن التغيير في المقياس في الفضاء هو تفسير بسيط ودقيق لأفضل ما لدينا من معلومات للملاحظات، وبالتالي فهو مدرج أيضًا في النماذج الكمومية. لكن الاستقراء حتى النقطة التي يكون فيها a=0 ينتج عن حل المعادلات الكلاسيكية التي لن تكون موجودة في نظرية الكم.

  212. ألبينتيزو,
    سأحاول صقل. لقد كتبت أنه - "وفقًا لنموذج الانفجار الكبير للنظرية النسبية، بدأ الكون من نقطة واحدة، أي أن كل الفضاء اللانهائي له مقياس من 0"
    وبما أن هذا لم يكن هو الحال على الأرجح، فهل التفسير الذي قدمته لا يزال قائمًا؟ هل الشرط الأولي: كون بحجم صفر هو تفصيل مهم للنموذج أم لا ولكن النموذج سوف ينجو من التغيير مع التصحيحات المطلوبة؟
    في ضوء إجابتك الأخيرة، أفهم أن نعم ونعم، وآمل أن أكون أكثر فهمًا الآن

  213. شموليك،

    لست متأكدًا مما تقصده بـ "شرح المقاييس". لقد قمت بسرعة بمراجعة تعليقاتي السابقة ويبدو لي أن كل ما كتبته مشترك في كل من علم الكونيات الكلاسيكي وما نعتقد أنه موجود كميًا، باستثناء الفيزياء القريبة من نقطة الانفجار بالطبع. ولكن إذا كنت تريد إجابة أكثر دقة أو حسمًا، فستحتاج إلى توضيح ما تعنيه بالضبط.

    يونيو،

    أولا وقبل كل شيء، Scalar هو شيء آخر تماما. على الرغم من التشابه في الكلمة، العددية لا علاقة لها بالسكالا. هذا العدد هو مصطلح من عالم الرياضيات ويعني الكمية التي تظل ثابتة بعد تحول أو آخر، عندما يكون الاستخدام الأكثر شيوعًا للمصطلح هو نحو تحويل دوراني (وبالتالي متجه له اتجاه وبالتالي سيتغير تحت اتجاه التحول الدوراني ليس عدديًا، لكن بعض الأرقام - مثل طاقة الجسيم - لن تتغير أثناء الدوران وبالتالي الحجم العددي).

    و. مقال يتحدث عن ماذا بالضبط؟ حول كيفية توسع الكون اللانهائي؟ المقالات المتعلقة بهذا تمت كتابتها ونشرها منذ عقود. من السهل اليوم العثور على هذه الأشياء في أي كتاب تمهيدي للنظرية النسبية، في الفصل الخاص بعلم الكونيات. على سبيل المثال، كتاب شون كارول هو كتاب واضح إلى حد ما ويغطي الموضوع بطريقة بسيطة للغاية.

    الزمكان والهندسة / شون كارول

    ب. يتم تحديد الشكل الصريح لفون المقياس (لاحظ، هذا هو فون الزمن: هذا هو بيت القصيد، إنه ليس حجمًا ثابتًا ولكنه شيء يتغير بمرور الوقت) يتم تحديده عن طريق حل معادلات أينشتاين، أو في الواقع معادلات فريدمان، وبالتالي يعتمد على ما هو موجود في الفضاء. فكرة النسبية العامة بأكملها هي أن الجاذبية هي التفاعل بين الفضاء نفسه وما يحتويه. ولذلك فإن حل الدالة العددية يعتمد على النموذج الدقيق ومحتويات الكون الخاص به. ما ننظر إليه عادة هو كون متجانس إلى حد ما وذو كثافة منخفضة (تقريب ممتاز لكوننا)، ومن ثم يمكن حل عامل القياس وإيجاده. لاحظ أنها دالة غير تافهة للزمن، وتتصرف بشكل مختلف سواء كان العامل المهيمن في الكون هو المادة أو الإشعاع مثلا. سامحني على عدم كتابة فون أو معادلات هنا، ولكن بالنسبة للقسم السابق - يمكن العثور على الصيغة الصريحة في أي كتاب تمهيدي للنسبية العامة وعلم الكونيات، وخاصة في الكتاب الذي ذكرته سابقًا.

    ثالث. وبحلول أربعة مليارات سنة، كان كل شيء أصغر. فيما يتعلق بالشمس على وجه الخصوص، لا أعرف كيف أجيب بوضوح لأنني لست خبيرًا في تطور النجوم (إنها فيزياء فلكية وأدرس نظرية الأوتار). ليس لدي أي فكرة عن شكل الشمس قبل أربعة مليارات سنة، ربما كانت أكبر بسبب طبيعة عمليات تطور النجوم. إذا كنت تريد معرفة عامل القياس في وقت معين، فنحن نرحب بك لوضع الأرقام بنفسك في الصيغ الموجودة في الكتب التي ذكرتها.

    رابع. ليس لدي أي فكرة عما تقصده أو ما الذي تتحدث عنه.

  214. لقد فهمت ما كتبته عن العددية.
    و. وسأكون ممتنًا لو تفضلتم بتوجيهي إلى مقال يتحدث عن ذلك.
    ب. سأكون ممتنًا لو تفضلتم بإعطائي قيمة العددية ومعدل تغيرها، لأن هذه هي الطريقة التي يتم بها قياس الوقت منذ الانفجار الأعظم.
    ثالث. بالنسبة لك، قبل حوالي أربعة مليارات سنة، كانت الشمس أصغر مما هي عليه اليوم لأن العدد كان أصغر، وإذا كان الأمر كذلك فبكم؟ (إذا أمكن يرجى ذكر المصدر العلمي).
    رابع. وبحسب المعرفة المحدثة فإن معدل التوسع يتزايد، ورغم ذلك فإن أبعد النجوم التي نرصدها لا تبتعد بسرعة نصف سرعة الضوء، ما علمنا يوما أن معدل تغير المقياس كان أقل، وإذا كان الأمر كذلك، فكيف تمكنت النجوم، رغم مرور نحو 13 مليار سنة، من الابتعاد عنا مسافة أكبر من ذلك؟

  215. ألبانزو
    نتجاهل التمديد الزمني في تمرين كرة البلياردو، لأن التمديد الزمني ليس له أي معنى في هذا السياق. ليس الأمر كذلك عند النقطة المفردة.

    أخشى أننا نتحدث عن أشياء مختلفة، وربما لم أشرح نفسي جيدًا. ليس لدي سبب لأجادلك حول الفيزياء نفسها 🙂

    أنا لا أناقش ما تقوله النظرية النسبية. أنا فقط أقول أنه من الغريب بالنسبة لي أن أتحدث عما تقوله النسبية حيث نعلم أنه غير صالح.

    أفهم (الآن) أنك تقول أن ادعاءات رافائيل خاطئة، أيضًا في سياق النسبية. شكرا على الشرح!

  216. 1. ماذا تعني عبارة "كيف يمكن للمرء أن يتجاهل"؟ عند حل مسألة كرة البلياردو في الميكانيكا الكلاسيكية، هل تأخذ في الاعتبار تباطؤ الزمن؟ بالنظر إلى قوة باولي بين الكرات؟ في الفيزياء، نبني نموذجًا ثم ننظر إلى الحلول التي يقدمها للمشكلات. في نموذج النسبية العامة لا يوجد شك ولا توجد ظاهرة كمومية، وبالتالي عند حل مشكلة بهذا النموذج تحصل على نتائج تتعارض مع هذه المبادئ.

    2. لا، لقد كان رافائيل مخطئًا عندما قال ذلك، وأنت أيضًا مخطئ الآن. من الممكن كتابة نماذج لنقطة (صفرية الأبعاد) تتوسع إلى فضاء ثنائي الأبعاد. لا توجد مشكلة هندسية في هذا كما اعتقد رافائيل (وكما أعتقد أنك تعتقد). هل هناك مشاكل في هذا تنشأ من التوطين الكمي؟ نعم من حيث المبدأ. لذلك، في النظرية التي لا تتضمن مبادئ الكم، سيكون مثل هذا الشيء ممكنًا، وفي نظرية الجاذبية الكمومية، ربما لا يكون كذلك، على الأقل ليس بنفس الطريقة (على الرغم من أننا لا نستطيع الجزم بذلك) لن يكون ذلك ممكنًا على الإطلاق لأننا ببساطة لا نعرف ذلك على وجه اليقين بعد).

    3. من المؤكد أن مناقشتنا تطرقت إلى ما تقوله النظرية النسبية، ومسألة ما إذا كانت تناسب الواقع أم لا. نموذج الانفجار الأعظم هو نموذج في إطار النسبية، وليس نظرية الأوتار أو جاذبية الحلقة الكمومية. يمكنك العودة ورؤية أنه من خلال تعليقاتي أميز بين ما يقوله نموذج الانفجار الأعظم وما هو صحيح حقًا، وأحاول توضيح متى تكون النسبية موثوقة ومتى لا تكون كذلك. عندما تكتب أن نموذج الانفجار العظيم لا يبدأ من نقطة ما، فأنت ببساطة مخطئ، لأنه يبدأ من نقطة ما. ربما لا يكون نموذجًا جيدًا لأنه ليس كميًا، لكنه يبدأ من نقطة ما.

    4. إن عدد المرات التي كتبت فيها أن النظرية النسبية لا تقدم تفسيراً كاملاً للمشكلة الكونية أكبر من أن أمتلك القوة لإحصائها والاقتباس منها هنا.

    5. مرة أخرى، اقرأ تعليقاتي وانظر كم مرة كتبت أن التفرد هو إسناد للنسبية وأن النظريات الحديثة تناقضه، لذلك لا ينبغي أن يؤخذ هذا الإسناد على أنه حقيقة. لا أفهم كيف لم يخرج من فمي. على سبيل المثال، الاقتباس

    "كما علقت، فإن حقيقة أنه وفقًا للنسبية العامة صحيح لا يعني أنه صحيح من حيث المبدأ لأن هذه الفترة الزمنية (حيث كان المقياس صغيرًا جدًا) هي على وجه التحديد نقطة الضعف في النظرية النسبية حيث أنها ليست موثوقة. ولهذا السبب ينظر الباحثون المعاصرون إلى تعاليم أخرى ويتوصلون إلى استنتاجات أخرى."

    6. كان لدى رافائيل أسئلة بخصوص النموذج النظري، حاولت توضيحها. نحن نعلم أن الميكانيكا الكلاسيكية خاطئة، لكن إذا جاءني شخص وادعى أن الميكانيكا الكلاسيكية غير منطقية أو أن معادلات نيوتن تخلق حالة لا يمكن أن توجد فيها الحركة النسبية، فسوف أثبت له خطأ وأحاول أن أشرح له المسألة. هذا لا يعني أنني أدعي أن الكون موصوف بالميكانيكا الكلاسيكية. فكما أنه من الجميل أن نفهم الميكانيكا الكلاسيكية (رغم أنها خاطئة) وهناك فائدة من فهمها لأنك تستطيع أن تستمد منها المعرفة والحدس لنظريات أكثر تقدما ودقة، كذلك هناك الكثير من الجمال والكثير. من نقطة في فهم النماذج الكلاسيكية لعلم الكونيات. ولذلك، عندما طرح رافائيل الأسئلة، أجبت عليها مع التوضيح بشكل لا لبس فيه أنني كنت أشرح له كيف يعمل النموذج على الرغم من أن النموذج ليس دقيقا وليس في الواقع ما يعتقد الفيزيائيون أنه حدث في الكون المبكر.

    لقد كتبت هذا عدة مرات خلال المناقشة، يمكنك البحث بنفسك.

  217. ابي،
    ماذا كتبت بالفعل أن الرد ينتظر التأكيد؟
    بالإضافة إلى ذلك، أوافق على طلب الآخرين بإعادة صفحة التعليقات الأخيرة. جزء كبير من متعة هذا الموقع هو القدرة على الوصول بسهولة إلى أحدث التعليقات.

  218. فهل سيكون تفسير المقاييس صحيحا حتى لو لم تكن هناك نقطة مفردة بسبب التأثيرات الكمومية؟
    بمعنى آخر، هل سيكون تفسير هال صحيحًا أيضًا في ظل نظرية الجاذبية الكمومية؟

  219. ألبانزو
    انا افهم ما تقول.
    لكنني لا أفهم كيف يمكنك تجاهل نظرية الكم عند النقطة المفردة. يقدم رافائيل حجة منطقية - لن يؤدي أي تضخم بالحجم 0 إلى زيادة النقطة، وبالتأكيد ليس إلى شيء لا نهائي. مناقشتنا ليست حول ما تقوله النظرية النسبية في هذه المرحلة، ولكن حول ما يحدث بالفعل هناك (أو بعد ذلك..).

    كل ما أحاول قوله هو أنني لا أعتقد أنه من الضروري الادعاء بأن النظرية النسبية تعطي تفسيرًا كاملاً لما حدث، وحسب فهمي، فإن هذا ما تكتبه يعني ضمنيًا. إذا كان التفرد نتيجة للنسبية، في مكان/زمان تقول أنت بنفسك إنه غير صحيح، فلماذا تشرح كيف توسع الكون من تلك النقطة؟ يبدو لي أن هذا ليس ضروريا.

  220. معجزات,

    ما أشرحه مرارًا وتكرارًا هو أن النظرية النسبية بأكملها تتناقض مع نظرية الكم. إنها نظرية كلاسيكية وليست كمومية. ليست هناك حاجة للبحث عن نقاط منفردة في الانفجار الأعظم، انظر إلى حركة جسيم حر في الفضاء الفارغ في النظرية النسبية وسوف ترى أن مبدأ عدم اليقين مكسور.

    ما كتبته في العديد من تعليقاتي الأخيرة هو أن النظرية النسبية تحتوي بالفعل على نقطة مفردة بحجم 0، ولكنها نظرية كلاسيكية. ولهذا السبب نعلم أنها ليست دقيقة. حتى نقطة معينة، يعد هذا تقريبًا جيدًا للواقع، ولكن بدءًا من نقطة معينة لم يعد الأمر كذلك، ومن ثم تكون هناك حاجة إلى الجاذبية الكمومية. وعلى أية حال، لا يمكنك استخدام حجج عدم اليقين في إطار النسبية العامة لأنها لا تحترمها، تمامًا كما لا يمكنك أن تأخذ حلاً لمشكلة في الفيزياء النيوتونية وتقول إنها جيدة أو سيئة اعتمادًا على مبدأ عدم اليقين. التوراة بأكملها تتناقض مع ميكانيكا الكم، لذلك فإن هذه الحجج ليست ذات صلة.

  221. رافائيل
    لقد فاجأتني اعتقدت أنك بدأت تؤمن لأنه كان منطقيًا بالنسبة لك. لقد سمعت منك العديد من الحجج "المنطقية" لوجود الخالق، مثل وجود القوانين الطبيعية.

    لا بد أنني كنت مخطئا.

  222. رافائيل
    إن ما نسميه "المنطق" لا يتعامل مع مشاكل الصغير جدًا أو الكبير جدًا. المنطق يقول أن الأرض مسطحة والسماء فوقنا. المنطق هو ما يرشد الأطفال في مرحلة الروضة (اطلب من طفل صغير أن يرسم - أين يضع السماء؟ بعد ذلك نقتنع بأنه لا يعمل خارج نطاق حياتنا اليومية.

    المنطق شيء مختلف تمامًا، وقد استغرق الأمر سنوات عديدة حتى تعلمنا قوانين المنطق (أرسطو، بول، فريجه، راسل، فيتجنشتاين وما إلى ذلك).

    رافائيل: أعتقد أنك توافق أيضًا على أن المعتقد الديني يأتي من المنطق، أليس كذلك؟

  223. ألبانزو
    ألا يتعارض هذا مع نظرية الكم القائلة بأن كل المادة ثابتة عند نقطة واحدة؟ لا أعرف إذا كان هذا يتعارض مع مبدأ عدم اليقين، أو مبدأ باولي، لكنه يجب أن ينجح، أليس كذلك؟

  224. معجزات,

    هذا غير صحيح. أولًا، عدم اليقين ليس له أي صلة لأن النسبية ليست كمية. لذلك في التفسير ليس هناك شك في ذلك. ثانيًا، هناك بالتأكيد نقطة فريدة حيث يصبح عامل المقياس صفرًا. كما أن هناك نقطة مفردة داخل الثقب الأسود الكلاسيكي، والتي من المحتمل أن تنتشر الكم ولا تكون مفردة.

  225. رافائيل / ألبانزو
    لا أعتقد أن جميع نماذج الانفجار الكبير تبدأ عند نقطة مطلقة. وهذا يتناقض مع مبدأ عدم اليقين، كما أنني لا أعتقد أنه من الضروري أن تكون البداية نقطة. "التفرد" هو النقطة التي لا تصمد فيها النظرية النسبية، وبالتحديد أن x/x لم يتم تعريفها عند 0. تسمح لنا الرياضيات بتعيين القيمة عند 0 كحد أقصى، لكن هذا لا يعني أن القيمة عند 0 هي 1!
    إنه يذكرني قليلاً بتوراة بيرمينديز، ومفارقات زينون. رياضياً - تم حل المفارقات، لكن فلسفياً - لا يوجد حل حتى الآن 🙂

    رافائيل - فيما يتعلق بالزمن، هناك مفهوم يسمى "الوقت المناسب" وهو الزمن الذي سيقيسه الراصد في تلك المرحلة، وهذا الوقت محدد بشكل جيد.

    لا تحاول استخدام المنطق لفهم الانفجار الكبير. النسبية تتعارض مع المنطق (أعني الفطرة السليمة، وليس المنطق الفلسفي)، ونظرية الكم تتعارض مع المنطق. لذلك لا تتوقع أنه إذا وصلت إلى الحد الأقصى فستحصل على شيء معقول 🙂

  226. رافائيل،

    لا أعرف ماذا أقول لك، هذا ليس صحيحًا. نحن "نشغل المساحة بأكملها" كما تقول إذا وفقط إذا كان عامل القياس هو 0. أما بالنسبة لأي رقم آخر، فلا نفعل ذلك. انظر إلى المثال الذي قدمته، ضع أي رقم باستثناء a=0، وسوف ترى أننا نبتعد عن بعضنا البعض. إنها عملية حسابية بسيطة جدًا، إذا أردت، يمكنك محاولة النظر إليها - إذا فهمت ما هي المساحة المترية، فكل ما تحتاج إلى النظر إليه هو مصفوفة FRW. كل هذا الحديث لا معنى له إلى حد ما لأنه يمكنك أن ترى أنك مخطئ من خلال حساب ثانية واحدة (إذا كنت تعرف ما هو المقياس).

  227. ألبانزو

    إذا لم ننمو أنا وأنت بالنسبة إلى الميزان، فستنشأ بيننا مسافة تسمح بالحركة في الفضاء، ولكن إذا كنت أنا وأنت ننمو معًا مع الميزان، فكما في البداية احتلنا المساحة بأكملها (والتي كانت ثم صفر)، لذلك سنشغل دائمًا المساحة بأكملها، مهما زاد المقياس، وفي هذه الحالة لن يكون هناك مجال للتحرك.

  228. وكانت المسافة 0 فقط في لحظة واحدة من الزمن. يتم تعريف الحركة دائمًا على مدى فترة من الزمن. لنترك المقياس متلاشيًا، لنسميه أ.

    لنفترض أن المسافة بيني وبينك شريط واحد، أي مسافة 1*a (كما في المثال السابق - إذا كان المقياس 1 فهي مسافة متر واحد، وإذا كان نصفاً فهي نصف متر) متر، الخ). لكن هذه لقطة للحظة واحدة. فكر في الصورة التي كانت فيها المسافة بيننا في لحظة البداية هي 2a، وبعد ثانية أصبحت 3a، وبعد ثانيتين أصبحت 0a، وما إلى ذلك. أنت تقول أنه إذا قمنا بتعيين = 0 فسنحصل على أنه لا توجد حركة، وهذا صحيح من الناحية الفنية، ولكن بالطبع لا معنى له لأن بيت القصيد هو أن المقياس يتغير بمرور الوقت. في لحظة الانفجار تساوي 0، لكن بعد جزء من الثانية لم تعد XNUMX. لذلك لا توجد مشكلة في الحركة النسبية طالما أنك لا تنظر فقط إلى لحظة الانفجار نفسها (و كما قلت، بدون أي علاقة بالنسبية عند الحديث عن الحركة، لا يمكنك مجرد النظر إلى لحظة واحدة).

  229. ألبينزا,

    1. شكرا لك.

    2. إذا كانت المسافة بيني وبينك في البداية صفراً وكل شيء يتوسع بنفس النسبة، فإن المسافة بيننا ستظل دائماً صفراً لأنني وأنا أيضاً ننمو بنفس النسبة وحينها لن يكون هناك أي حركة ممكنة...

    3. شكرًا لك على توضيح أن 13.7 مليار سنة مرتبطة بالإطار المرجعي للمقياس النظري (الذي استقر في مكانه لحظة الانفجار في الفضاء الفارغ ولم يتحرك أو يمر في مجال جاذبية كبير منذ ذلك الحين). المشكلة هي أنه ليس الجميع على علم بذلك ويخلق الكثير من الاحتكاكات والصعوبات التي ليس لها مكان في الواقع.

  230. 1. وفقا لنموذج الانفجار الكبير للنظرية النسبية، بدأ الكون من نقطة واحدة، أي أن كل الفضاء اللانهائي كان له مقياس 0، وبالتالي طوبولوجيًا فهو يساوي نقطة واحدة. أنت تسأل "كيف" ولكن هذا بالضبط ما شرحته لك. إذا لم يكن الشرح شاملاً بما فيه الكفاية بالنسبة لك، فنحن نرحب بك لقراءة المزيد من الأدبيات المهنية، ولكن هذا سيتطلب تطوير قدرات رياضية غير تافهة. وكما علقت، فإن حقيقة أنه وفقًا للنسبية العامة صحيح لا يعني أنه صحيح من حيث المبدأ لأن هذه الفترة الزمنية (حيث كان المقياس صغيرًا جدًا) هي بالضبط نقطة الضعف في النسبية حيث لا يمكن الاعتماد عليها. ولهذا السبب ينظر الباحثون المعاصرون إلى تعاليم أخرى ويتوصلون إلى استنتاجات أخرى.

    2. حقيقة أن كل شيء يتمدد معًا لا تأتي على حساب الحركة النسبية. إذا كان هناك نملتان على بالون، وقفت إحداهما ساكنة والأخرى ابتعدت عنه، وانتفخ البالون، فإن كلا التأثيرين يحدثان في وقت واحد - يبتعد النملان عن بعضهما نتيجة للتضخم وتكون هناك حركة نسبية بينهما. لا أفهم ما هي المشكلة. وأن التغيير في الحجم ليس تغييراً، لأن كل شيء يتغير ليس صحيحاً، لأن هناك وقت. في أي وقت من الأوقات إذا قمت بتجميد الإطار ونظرت إلى الكون، فلن تتمكن من معرفة المقياس. ولكن هناك أساليب وقياسات يمكن أن تحدد حجم المقياس اليوم مقارنة بما كان عليه بالأمس.

    3. إن تدفق الوقت نسبي بالفعل ويمكن اللعب به. الأرقام التي تعطى عادة للوقت الذي انقضى منذ حدث معين تشير إلى إطار مرجعي معين (مراقب يتحرك مع الكون). وفي هذا الصدد، يمكنك القول إن هذا هو الوقت الذي كان يمكن قياسه من قبل شخص استقر في مكانه لحظة الانفجار في الفضاء الفارغ، ومنذ ذلك الحين لم يتحرك أو يمر عبر مجال جاذبية كبير. قد يرى المشاهدون الآخرون وقتًا أقصر اعتمادًا على الإطار المرجعي الخاص بهم.

  231. ألبينزا,

    لقد تحدثنا كثيرًا في الماضي حول هذا الموضوع. ولكن الآن أريد أن أوضح مسألة مبدأ. إذا بدأ الكون من نقطة مفردة هي الصفر حرفيًا (وليس كما أوضح نسيم)، فإننا لا نزال بحاجة إلى شرح بسيط لكيفية أن يكون الكون صفرًا ولانهائيًا في نفس الوقت. وإذا كان الأمر كذلك فما الفرق بين الصفر واللانهاية؟

    شيء آخر أريد أن أسأله بسبب مثلك عن المسافة بيني وبينك التي تقلصت إلى النصف. أنت تقول أنه في هذه الحالة تقلص كل شيء بمقدار النصف. وهذا يعني أنه في الواقع لم يكن هناك أي تغيير لأننا تقلصنا أنا وأنت أيضًا. لكن في الكون لا ترى أن كل شيء يتوسع معًا لأنه بخلاف ذلك لن نكون قادرين على ملاحظة أي حركة، على سبيل المثال - لن نتمكن من ملاحظة تحرك المجرات بعيدًا عن بعضها البعض لأن المجرات نفسها كما تنمو بنفس النسبة.

    شيء أخير - يرجى الرجوع إلى مسألة 13.7 مليار سنة. هل ستوافق المخلوقات التي تعيش على كوكب آخر يتمتع بجاذبية أكبر بكثير من كوكبنا على أن 13.7 مليار سنة قد مرت منذ أن بدأ الكون في التوسع؟

  232. رافائيل،

    أنا متأكد من أننا أجرينا هذه المحادثة من قبل وقد شرحت لك كل هذا بالفعل.

    عليك أولاً أن تفهم ما هو سكالا. إذا كنت اليوم واقفا بجانب بعضنا البعض، ويفصل بيننا مسطرة مكتوب عليها "1 متر"، فإن المسافة بيننا هي متر. ويمكن إدخال 100 من هذه المساطر على طول ملعب كرة قدم، بحيث يبلغ طوله 100 متر. ويوجد بين حيفا وتل أبيب حوالي 80000 ألفاً من هؤلاء الحكام، بحيث تبلغ المسافة بينهما 80 كيلومتراً.

    كان حجم الأمس مختلفًا. كما هو الحال على جهاز الكمبيوتر، يمكنك أحيانًا تمديد الصورة أو تصغيرها دون تغيير النسب (أمسك الزاوية واسحب)، وكذلك الحال بالنسبة للكون. لذا، إذا كان المقياس بالأمس 1/2 مقارنة باليوم، فأنا وأنت - على الرغم من أننا لا نزال مفصولين بنفس الشريط - سنكون على مسافة نصف متر. تقلص الشريط أيضًا، وتقلص كل شيء لأن الفضاء نفسه تقلص. في ملعب كرة القدم يدخل 100 مسطرة لكن طوله 50 مترا، وبين حيفا وتل أبيب يدخل 80000 ألف مسطرة لكن المسافة 40 كم. بالأمس كان الوضع أسوأ، وكان المقياس 0.1. أي أن المسافة بين حيفا وتل أبيب كانت 8 كيلومترات، مع أن 80000 ألف حاكم ما زالوا يسيرون بينهما، الخ. لاحظ أنه في كل يوم، على الرغم من اختلاف المقياس، إلا أن الكون لا يزال لا نهائيًا. لا يزال بإمكانك المضي قدمًا في أي اتجاه تختاره وفي أي مسافة تختارها. إذا كان موشيه متهماً بالرشوة ويريد الهروب مسافة 3000 كيلومتر من دولة إسرائيل، فيمكنه اليوم الفرار إلى سويسرا. بالأمس كان علينا أن نركض إلى الساحل الشرقي لأمريكا. بالأمس كان عليه أن يهرب إلى الفضاء الخارجي ليبتعد مسافة 3000 كيلومتر عن سلطات إنفاذ القانون، لكن ذلك ممكن على أية حال. الكون ليس له نهاية في كلتا الحالتين.

    يمكنك أن تسأل ماذا يحدث إذا عدت بالزمن إلى النقطة التي يكون فيها المقياس 0 حرفيًا. وبالتالي فإن المسافة بيني وبينك هي 0، على الرغم من أنه يفصل بيننا شريط. طول القطعة هو 0 والمسافة بين حيفا وتل أبيب هي 0. وبما أن المسافة بين أي نقطتين في الفضاء هي 0، فإن الفضاء هو في الواقع نقطة (هو الفضاء 0 البعد يعادل نقطة) . وهذا هو المقصود بنقطة المفرد – الزمن الذي كان فيه المقياس 0 بالضبط وبالتالي كان كل مكانين في الكون يفصلان عن بعضهما البعض بمسافة 0 وكانا في الواقع نفس المكان. مصطلح النقطة المفردة مشتق من التعبير الرياضي الذي يعني أن التحويل المطابق (الذي يغير المقياس) هو مفرد - غير محدد بالوحدات. فهو يعين جميع النقاط في الكون إلى نفس المكان.

    وخلافا لما كتبته، وفقا للنظرية النسبية فإن الكون بدأ من هذه النقطة. من الممكن العودة بعد تضخم كوننا إلى الوقت الذي كان فيه المقياس صفرًا حرفيًا. ولكن كما كتبت لكم من قبل، قبل أن نصل إلى هذا الوقت، نصل إلى وقت تصبح فيه التأثيرات الكمومية مساوية في الأهمية لتأثيرات الجاذبية في تطور الكون، وبالتالي لا يمكن إهمال ميكانيكا الكم ويجب وصف العملية مع بمساعدة نظرية تجمع بين الجاذبية والكم. النسبية العامة ليست كذلك، وبالتالي فإن توقعها (أن الكون كان حرفيًا نقطة) ليس بالضرورة موثوقًا به. ما هو موثوق؟ إنه طويل بعض الشيء وقد تحدثنا عنه كثيرًا في الماضي (أتذكر أنك عرضت مقطع فيديو لباحثين من معهد المحيط يتحدثون عما كان موجودًا بالفعل في الانفجار وما سبقه، من خلال النظر في نظريات الكم للجاذبية بدلاً من النسبية العامة وهي قريبة من الانفجار).

  233. المعجزات

    بمعنى آخر، أنت تقول إن الكون، كما يراه الفيزيائيون اليوم، لم يبدأ من نقطة، لأن النقطة لا تشغل حيزاً على الإطلاق، بل من جسم أولي صغير جداً ثلاثي الأبعاد انتفخ. لذا فإن 13.7 مليار سنة ليست المدة الزمنية التي انقضت منذ خلق الكون، ولكنها الوقت الذي بدأ فيه التضخم لسبب ما.

    وأذكرك أيضًا أنه لا يزال من غير المفهوم كيفية حساب 13.7 مليار سنة حيث أن الزمن ليس مسألة عالمية بل مسألة شخصية للغاية ويختلف من جسم إلى آخر حسب السرعة التي يتحرك بها الجسم وبحسب قوة الجاذبية المبذولة. عليه.

  234. رافائيل
    فكر في الأمر على هذا النحو: أنت عند النقطة المفردة وفي الوقت 0 تبدأ في التحرك. النقطة "تتضخم" بشكل أسرع بكثير مما تستطيع تحريكه. لذلك، بالنسبة لك، ليس هناك "نهاية" لهذا الكون.

    النقطة ليست بالحجم 0، ولكن بحجم صغير جدًا، صغير جدًا.

  235. يونيو،

    قرأت تعليقاتك مرارًا وتكرارًا ولا أستطيع أن أفهم أين ترى التناقض. هل يمكنك كتابة عبارتين بشكل صريح وإظهار أنهما يؤديان إلى استنتاجات متعارضة، أي أن هناك تناقضا بينهما؟

    وبالمناسبة، لا يهم إذا كتبت "راجع ويكيبيديا"، فلن يجعل ذلك أي شيء تقوله صحيحًا إلى حد ما. لقد كان الكون دائمًا لانهائيًا، والهدف الأساسي هو فقط تغيير حجم الفضاء. لم يكن للكون حجم ولم يكن له حجم أبدًا (وفقًا للنظريات الكونية المقبولة اليوم).

  236. هذا هو التناقض تمامًا، فمن ناحية وفقًا لنظرية الانفجار الكبير، بدأ كل شيء عند نقطة تفرد انظر ويكيبيديا، ثم حدث التضخم الذي زاد الكون إلى حجم مجرة ​​متوسطة انظر ويكيبيديا، ومنذ ذلك الحين أصبح كل شيء وفقًا لنظرية الانفجار العظيم. الأحجام المادية التي نعرفها اليوم. ومن ناحية أخرى، يزعمون أن هناك نجوم تبعد عنا أكثر من 13 مليار سنة ضوئية، أو كما يعرفها "الكون المرئي".
    وأوافق أيضًا على أنه عندما غادر الضوء النجم كان أقرب، لكن نظرًا لتمدد الفضاء نفسه، استغرق وصوله وقتًا أطول. لكننا نعلم أن المسافة بيننا وبين أبعد الكوازارات تمتد بحوالي خمس سرعة الضوء، وهذا ما يسبب الانزياح نحو الأحمر، لذلك يمكننا الحساب بتقدير تقريبي أنه إذا استغرق الضوء 13 مليار سنة للوصول إلى لنا، فإن هذا الكوازار كان على مسافة حوالي خمس مسافته اليوم، أي حوالي 10 مليار سنة ضوئية، مما يعني أنه تمكن من الابتعاد منذ الانفجار الأعظم في 690 مليون سنة، حوالي 10 مليار سنة سنة ضوئية.

  237. يونيو،

    ما تكتبه هو الخطأ. إن الكون (كما نعلم اليوم وبحسب النظريات الكونية المقبولة) كان دائمًا لانهائيًا. ما يتغير هو فقط المقياس النسبي للمسافة. لم يكن الكون أبدًا بحجم مجرة، أو أي حجم آخر. ربما تخلط بينك وبين الكون المرئي، وهو شيء آخر تمامًا؟ وفي كلتا الحالتين، السؤال ذو الصلة هو كم من الوقت يستغرق جسمين للتفاعل (في هذا الصدد، تبادل الضوء بينهما). قبل التضخم، كان المقياس صغيرًا جدًا بحيث يمكن لأي جسمين أن يتفاعلا بشكل فوري تقريبًا وكانت السماء متجانسة. وبعدها لم يعد صحيحا.

    الكون يتوسع، وبالتالي فإن الضوء الذي سافر 13 مليار سنة ويصل إلينا اليوم، لم يبدأ على مسافة 13 مليار سنة ضوئية منا بل أقرب، والجسم الذي أطلقه اليوم ليس على بعد 13 مليار سنة ضوئية منا بل أكثر. هذا هو بالضبط ما يعنيه التغيير في الحجم. لكنني لا أفهم ما الذي يزعجك كل هذا.

  238. יוני
    أنت تنظر إلى الانفجار الكبير باعتباره انفجارًا في عالم ثلاثي الأبعاد. في مثل هذا العالم، سيستغرق وصول الضوء وقتًا قصيرًا نسبيًا.
    لكن الكون رباعي الأبعاد، وما يحدث ليس مثل الانفجار العادي الذي نعرفه. مثل هذا الانفجار له مركز، نقطة في الفضاء حدث فيها الانفجار.
    أما في الانفجار الكبير فالوضع مختلف. الفضاء نفسه يتوسع. والنتيجة هي أن الضوء يستغرق وقتًا أطول للوصول. بل إن هناك نطاقًا لن يصل إلينا منه الضوء أبدًا ولن نعرف أبدًا ما حدث هناك (أو بعد ذلك....).

    وبالمناسبة - حسب فهمي، فإن نفس الثقب الأسود الموصوف في المقالة أبعد بكثير، حوالي 45 مليار سنة ضوئية، أو أقل بكثير من 13 مليار سنة...

  239. صحيح أنه بحسب نظرية الانفجار الكبير فإن الكون خلق قبل 13.7 لكن يظهر في الموقع أنهم رأوا ثقبا أسود من وقت كان عمر الكون 690 مليون سنة، أي قبل 13 مليار سنة.
    المهم: بحسب نظرية التضخم فإن الكون نما في أجزاء قليلة من الثانية ليصل إلى حجم مجرة ​​متوسطة الحجم، مما يعني أن الضوء كان يجب أن يصل إلينا في وقت قصير (على الأكثر مائة ألف سنة). ) ، حتى لو استمر في التحرك بعيدًا بمعدل مرتفع جدًا، حتى بالنسبة لهذا الأمر بسرعة خمس سرعة الضوء (وهي سرعة هائلة تمامًا) لمدة 13 مليار سنة، فإن الضوء سيستغرق خمس ذلك الوقت للوصول إلينا.

  240. יוני
    لقد حدث الانفجار الكبير قبل 13.7 مليار سنة، وليس 13. وكما أوضح لك ألبينزو، كانت هناك مرحلة من التضخم توسع فيها الكون بسرعة أكبر بكثير من سرعة الضوء، ولكن الوقت الذي استغرقه (إذا كان من الممكن تفسير مفهوم الزمن) تستخدم على الإطلاق) هو جزء صغير من الثانية.

    فقرتك الأخيرة ليست ما كتبه ألبينزو على الإطلاق. أعد قرائتها

  241. لذا، إذا فهمت بشكل صحيح، فأنت تدعي أنه كان على بعد 13 مليار سنة ضوئية منا في أقل من بضع ثوان. قرأت عن الانفجار الكبير كثيرًا وفهمت أنهم توصلوا إلى نتيجة مفادها أن كل شيء كان عند نقطة واحدة لأننا نرى النجوم والمجرات تبتعد عنا، فإذا رجعنا الزمن نصل إلى حقيقة أن 13 قبل مليار سنة كان كل شيء في نفس النقطة، ولكن وفقًا لادعائك، قبل 13 مليار سنة كانت هناك بالفعل نجوم كانت بعيدة عنا مسافة 13 مليار سنة ضوئية.

  242. عاشق,

    حجمها هو نفسه تقريبا. تتناسب كمية الإشعاع الصادرة عن الثقب الأسود عكسيا مع كتلته. أي أن الثقوب الكبيرة لا تشع تقريبًا. حتى الثقب الأسود بحجم كتلة الشمس يكون إشعاعه صغيرًا جدًا. إن حجم الوقت الذي سيستغرقه مثل هذا الثقب الأسود الكبير لتجميع ما يكفي من الطاقة حتى نتمكن من ملاحظة انخفاض في الكتلة أكبر بكثير من عمر الكون.

    يونيو،

    صحيح أنه عندما كان الكون صغيرًا بما فيه الكفاية، كان كل شيء قريبًا جدًا وكان من الممكن لأي جسمين أن يتفاعلا في وقت قصير، ولكن هذه الفترة هي ما قبل التضخم. أدى التضخم الكوني إلى تضخيم الكون بعدة مراتب أسرع من سرعة الضوء، وبعد ذلك لم يعد هناك مثل هذا التجانس. إن الثقب الأسود المعني هو في الواقع بعيد جدًا وقديم جدًا، لكن هذه ليست ثوابت زمنية ذات صلة بالتضخم. وبعبارة أخرى، عندما تم إنشاء الكون كان بالفعل كبيرًا جدًا وفي مرحلة من التوسع البطيء، على غرار الصورة التي نراها في اليوم عندما ننظر إلى الكون.

  243. من الواضح أن هناك تناقضًا: فمن ناحية، يزعمون أن الضوء خرج قبل أكثر من 13 مليار سنة قبل أن يصل إلينا، مما يعني أنه بحلول وقت خروجه كان بالفعل بعيدًا جدًا عنا (لا أعتقد ذلك). أعرف كم، لأنه من الواضح أننا ابتعدنا بينما "سافر" الضوء نحونا)، ومن ناحية أخرى وفقًا لنظريات الانفجار الكبير الكبيرة، فإن الكون كله صغير جدًا في البداية لذا يجب أن يصل الضوء لنا بسرعة كبيرة. سأكون ممتنًا إذا تمكن شخص ما من حل حسابي.

ترك الرد

لن يتم نشر البريد الإلكتروني. الحقول الإلزامية مشار إليها *

يستخدم هذا الموقع Akismat لمنع الرسائل غير المرغوب فيها. انقر هنا لمعرفة كيفية معالجة بيانات الرد الخاصة بك.